You are on page 1of 202

केंद्रीय विद्यालय संगठन

Kendriya Vidyalaya Sangathan

सत्र/ Session 2023-24


अध्ययन सामग्री गणित
Study Material Maths
कक्षा - बारहिी
Class - XII

Kendriya Vidyalaya Sangathan (Jabalpur Region)


Regional Office, Jabalpur
Behind Science College,
South Civil Lines, Jabalpur (M.P.)- 482011
Website - https://rojabalpur.kvs.gov.in
केंद्रीय विद्यालय संगठन
Kendriya Vidyalaya Sangathan

हमारे संरक्षक
OUR PATRONS

श्री सोममत श्रीिास्ति


उपायक्
ु त, के.वि.स. क्षेत्रीय कायाालय जबलपरु

श्री हीरालाल श्रीमती सरोज डबास श्रीमती ककरण शमाा


सहायक आयुक्त सहायक आयुक्त सहायक आयुक्त

केंद्रीय विद्यालय संगठन क्षेत्रीय कायाालय जबलपुर


JABALPUR REGION

अध्ययन सामग्री/STUDY MATERIAL


सत्र/ SESSION: 2023-24
कक्षा/CLASS- बारहवीीं/TWELVE(XII)
गणित/MATHEMATICS
णवषय कोड/Subject Code - 041

SUBJECT COORDINATER:-

MR. PANKAJ KUMAR JAIN


PRINCIPAL,
PM SHRI KENDRIYA VIDYALAYA BALAGHAT

Page 1 of 199
SUBJECT COORDINATER’S MESSAGE

It is with profound delight and utmost pride that I announce the publication of our
study material for class XII (MATHEMATICS) for the session 2023-24. It’s my
firm belief that access to quality education should know no boundaries,
transcending social and economic constraints. Our collective vision is to
empower all students with the tools for success and intellectual growth.

With their steadfast dedication PGT-MATHEMATICS of Jabalpur Region of


Kendriya Vidyalaya Sangathan have invested their knowledge, expertise,
and passion into meticulously crafting these study materials to complement the
classroom learning experience of the students. These materials serve as
invaluable aids for self-study since they are comprehensive, well-structured,
and presented in a manner that is easy to comprehend.

It is with pleasure that I place on record my commendation for the commitment


and dedication of the team of all PGT Mathematics of Kendriya Vidyalaya
Sangathan Jabalpur Region.

Wishing you all the very best in your academic journey!

PANKAJ KUMAR JAIN

PRINCIPAL
PM SHRI KENDRIYA VIDYALAYA BALAGHAT

ii

Page 2 of 199
CONTENT DEVELOPMENT TEAM OF PGT MATHEMATICS

S.NO. NAME OF CHAPTER PREPARED BY KENDRIYA VIDYALAYA

1. Relations and functions


DR SAIYED SHAKEEL
1 SHAHADOL
AHMAD
2. Inverse Trigonometric Functions

3. Matrices
SHRI AVANI PRAKASH
2 JABALPUR VF S-1
SHRIVASTAVA
4. Determinants

5. Continuity and Differentiability

3 SHRI Anand Kumar Tiwari JABALPUR STC(S1)

6. Application of Derivatives

4 7. Integrals SHRI Mukesh Nigam JABALPUR GCF NO.II

8. Application of Integrals
SHRI UMESH KUMAR
5 REWA NO.I
TRIPATHI
9. Differential Equations

10. Vector Algebra


JAMUNA COLLIERY
6 SHRI ATUL KUMAR GUPTA
SECL
11. Three Dimensional Geometry

12. Linear Prpgramming Problem


SMT. SHRADDHA
7 CHAURAI
AGRAWAL
13. Probability

REVIEW , COMPILATION AND


8 MR. RITESH AGRAWAL BALAGHAT
ADDITION OF SAMPLE PAPERS

Page 3 of 199
INDEX

S.NO. NAME OF CHAPTER PAGE NUMBER

12-15
1 1. Relations and functions

2 2. Inverse Trigonometric Functions


16-19

3 3. Matrices 20-29

4 4. Determinants 30-39

5 5. Continuity and Differentiability 40-44

6 6. Application of Derivatives 45-49

7 7. Integrals 50-60

8 8. Application of Integrals 61-65

9 9. Differential Equations 66-71

10 10. Vector Algebra 72-75

11 11. Three Dimensional Geometry 76-79

12 12. Linear Prpgramming Problem 80-89

13 13. Probability 90-97

14 SAMPLE PAPERS 98-199

Page 4 of 199
MATHEMATICS (XI-XII)
(Code No. 041)
Session – 2023-24

The Syllabus in the subject of Mathematics has undergone changes from time to time in
accordance with growth of the subject and emerging needs of the society. Senior Secondary stage
is a launching stage from where the students go either for higher academic education in
Mathematics or for professional courses like Engineering, Physical and Biological science,
Commerce or Computer Applications. The present revised syllabus has been designed in
accordance with National Curriculum Framework 2005 and as per guidelines given in Focus Group
on Teaching of Mathematics 2005 which is to meet the emerging needs of all categories of
students. Motivating the topics from real life situations and other subject areas, greater emphasis
has been laid on application of various concepts.

Objectives

The broad objectives of teaching Mathematics at senior school stage intend to help the students:

 to acquire knowledge and critical understanding, particularly by way of motivation and


visualization, of basic concepts, terms, principles, symbols and mastery of underlying
processes and skills.
 to feel the flow of reasons while proving a result or solving a problem.
 to apply the knowledge and skills acquired to solve problems and wherever possible, by
more than one method.
 to develop positive attitude to think, analyze and articulate logically.
 to develop interest in the subject by participating in related competitions.
 to acquaint students with different aspects of Mathematics used in daily life.
 to develop an interest in students to study Mathematics as a discipline.
 to develop awareness of the need for national integration, protection of environment,
observance of small family norms, removal of social barriers, elimination of gender
biases.
 to develop reverence and respect towards great Mathematicians for their contributions
to the field of Mathematics.

Page 5 of 199
CLASS-XII
(2023-24)
One Paper Max Marks: 80
No. Units No. of Periods Marks
I. Relations and Functions 30 08
II. Algebra 50
10
III. Calculus 80
35
IV. Vectors and Three - Dimensional Geometry 30
14
V. Linear Programming 20
05
VI. Probability 30
08
Total 240
80
Internal Assessment
20

Unit-I: Relations and Functions

1. Relations and Functions 15 Periods

Types of relations: reflexive, symmetric, transitive and equivalence relations. One to one and onto
functions.

2. Inverse Trigonometric Functions 15 Periods

Definition, range, domain, principal value branch. Graphs of inverse trigonometric functions.

Unit-II: Algebra

1. Matrices 25 Periods

Concept, notation, order, equality, types of matrices, zero and identity matrix, transpose of a matrix,
symmetric and skew symmetric matrices. Operations on matrices: Addition and multiplication and
multiplication with a scalar. Simple properties of addition, multiplication and scalar multiplication. Non-
commutativity of multiplication of matrices and existence of non-zero matrices whose product is the
zero matrix (restrict to square matrices of order 2). Invertible matrices and proof of the uniqueness of
inverse, if it exists; (Here all matrices will have real entries).

2. Determinants 25 Periods

Page 6 of 199
Determinant of a square matrix (up to 3 x 3 matrices), minors, co-factors and applications of
determinants in finding the area of a triangle. Adjoint and inverse of a square matrix. Consistency,
inconsistency and number of solutions of system of linear equations by examples, solving system of
linear equations in two or three variables (having unique solution) using inverse of a matrix.

Unit-III: Calculus
1. Continuity and Differentiability 20 Periods

Continuity and differentiability, chain rule, derivative of inverse trigonometric functions,


𝑙𝑖𝑘𝑒 sin−1 𝑥 , cos −1 𝑥 and tan−1 𝑥, derivative of implicit functions. Concept of exponential and logarithmic
functions.
Derivatives of logarithmic and exponential functions. Logarithmic differentiation, derivative of functions
expressed in parametric forms. Second order derivatives.
2. Applications of Derivatives 10 Periods
Applications of derivatives: rate of change of quantities, increasing/decreasing functions, maxima and
minima (first derivative test motivated geometrically and second derivative test given as a provable
tool). Simple problems (that illustrate basic principles and understanding of the subject as well as real-
life situations).
3. Integrals 20 Periods
Integration as inverse process of differentiation. Integration of a variety of functions by substitution, by
partial fractions and by parts, Evaluation of simple integrals of the following types and problems based
on them.
dx dx dx dx dx
∫ ∫ ,∫ ,∫ 2 ,∫
x2 ±a 2,
√x 2 ± a2 2
√a − x 2 ax + bx + c 2
√ax + 𝑏𝑥 + 𝑐
px + q px + q
∫ dx, ∫ dx, ∫ √a2 ± x 2 dx, ∫ √x 2 − a2 dx
ax 2 + bx + c √ax 2+ bx + c
∫ √𝑎𝑥 2 + 𝑏𝑥 + 𝑐 𝑑𝑥,
Fundamental Theorem of Calculus (without proof). Basic properties of definite integrals and evaluation
of definite integrals.
4. Applications of the Integrals 15 Periods

Applications in finding the area under simple curves, especially lines, circles/ parabolas/ellipses (in
standard form only)

5. Differential Equations 15 Periods


Definition, order and degree, general and particular solutions of a differential equation. Solution of
differential equations by method of separation of variables, solutions of homogeneous differential
equations of first order and first degree. Solutions of linear differential equation of the type:

Page 7 of 199
dy
+ py = q, where p and q are functions of x or constants.
dx
d𝑥
+ px = q, where p and q are functions of y or constants.
d𝑦
Unit-IV: Vectors and Three-Dimensional Geometry
1. Vectors 15 Periods
Vectors and scalars, magnitude and direction of a vector. Direction cosines and direction ratios of a
vector. Types of vectors (equal, unit, zero, parallel and collinear vectors), position vector of a point,
negative of a vector, components of a vector, addition of vectors, multiplication of a vector by a scalar,
position vector of a point dividing a line segment in a given ratio. Definition, Geometrical Interpretation,
properties and application of scalar (dot) product of vectors, vector (cross) product of vectors.
2. Three - dimensional Geometry 15 Periods
Direction cosines and direction ratios of a line joining two points. Cartesian equation and vector equation
of a line, skew lines, shortest distance between two lines. Angle between two lines.

Unit-V: Linear Programming

1. Linear Programming 20 Periods

Introduction, related terminology such as constraints, objective function, optimization, graphical method
of solution for problems in two variables, feasible and infeasible regions (bounded or unbounded),
feasible and infeasible solutions, optimal feasible solutions (up to three non-trivial constraints).

Unit-VI: Probability

1. Probability 30 Periods
Conditional probability, multiplication theorem on probability, independent events, total probability,
Bayes’ theorem, Random variable and its probability distribution, mean of random variable.

Page 8 of 199
MATHEMATICS (Code No. - 041)
QUESTION PAPER DESIGN CLASS - XII
(2023-24)
Time: 3 hours Max. Marks: 80

%
S. Total
Typology of Questions Weightage
No. Marks

Remembering: Exhibit memory of previously learned material


by recalling facts, terms, basic concepts, and answers.
55
1 Understanding: Demonstrate understanding of facts and 44
ideas by organizing, comparing, translating, interpreting, giving
descriptions, and stating main ideas

Applying: Solve problems to new situations by applying


2 acquired knowledge, facts, techniques and rules in a different 20 25
way.
Analysing :
Examine and break information into parts by identifying
motives or causes. Make inferences and find evidence to
support generalizations

Evaluating:
Present and defend opinions by making judgments about
3 16 20
information, validity of ideas, or quality of work based on a set
of criteria.

Creating:
Compile information together in a different way by combining
elements in a new pattern or proposing alternative solutions

Total 80 100

1. No chapter wise weightage. Care to be taken to cover all the chapters


2. Suitable internal variations may be made for generating various templates keeping the overall
weightage to different form of questions and typology of questions same.

Choice(s):

There will be no overall choice in the question paper.


However, 33% internal choices will be given in all the sections

INTERNAL ASSESSMENT 20 MARKS


Periodic Tests ( Best 2 out of 3 tests conducted) 10 Marks
Mathematics Activities 10 Marks
Note: For activities NCERT Lab Manual may be referred.

Page 9 of 199
Conduct of Periodic Tests:

Periodic Test is a Pen and Paper assessment which is to be conducted by the respective
subject teacher. The format of periodic test must have questions items with a balance mix,
such as, very short answer (VSA), short answer (SA) and long answer (LA) to effectively
assess the knowledge, understanding, application, skills, analysis, evaluation and synthesis.
Depending on the nature of subject, the subject teacher will have the liberty of incorporating
any other types of questions too. The modalities of the PT are as follows:

a) Mode: The periodic test is to be taken in the form of pen-paper test.

b) Schedule: In the entire Academic Year, three Periodic Tests in each subject may be
conducted as follows:

Test Pre Mid-term (PT-I) Mid-Term (PT-II) Post Mid-Term (PT-III)


Tentative Month July-August November December-January

This is only a suggestive schedule and schools may conduct periodic tests as per their
convenience. The winter bound schools would develop their own schedule with similar time
gaps between two consecutive tests.

c) Average of Marks: Once schools complete the conduct of all the three periodic tests,
they will convert the weightage of each of the three tests into ten marks each for identifying
best two tests. The best two will be taken into consideration and the average of the two
shall be taken as the final marks for PT.
d) The school will ensure simple documentation to keep a record of performance as
suggested in detail circular no.Acad-05/2017.
e) Sharing of Feedback/Performance: The students’ achievement in each test must be
shared with the students and their parents to give them an overview of the level of learning
that has taken place during different periods. Feedback will help parents formulate
interventions (conducive ambience, support materials, motivation and morale-boosting)
to further enhance learning. A teacher, while sharing the feedback with student or parent,
should be empathetic, non- judgmental and motivating. It is recommended that the
teacher share best examples/performances of IA with the class to motivate all learners.

Page 10 of 199
Assessment of Activity Work:

Throughout the year any 10 activities shall be performed by the student from the activities
given in the NCERT Laboratory Manual for the respective class (XI or XII) which is
available on the link: http://www.ncert.nic.in/exemplar/labmanuals.htmla record of the
same may be kept by the student. An year end test on the activity may be conducted

The weightage are as under:


 The activities performed by the student throughout the year and record keeping
: 5 marks
 Assessment of the activity performed during the year end test: 3 marks
 Viva-voce: 2 marks

Prescribed Books:

1) Mathematics Textbook for Class XI, NCERT Publications


2) Mathematics Part I - Textbook for Class XII, NCERT Publication
3) Mathematics Part II - Textbook for Class XII, NCERT Publication
4) Mathematics Exemplar Problem for Class XI, Published by NCERT
5) Mathematics Exemplar Problem for Class XII, Published by NCERT
6) Mathematics Lab Manual class XI, published by NCERT
7) Mathematics Lab Manual class XII, published by NCERT

Page 11 of 199
1. RELATIONS AND FUNCTIONS
M.C.Q. (1 Mark each)
Direction: (Q.1 - Q.5) are multiple choice type questions. Select the correct option.

Q.1. Let 𝐴 = {1, 2, 3} and a relation R is defined in 𝐴 as: 𝑅 = {(1, 3), (2, 2), (3, 2)}, then minimum
ordered
pairs which should be added in relation R to make it reflexive and symmetric are:

(a) {(1, 1), (2, 3), (1, 2)}


(b) {(3, 3), (3, 1), (1, 2)}
(c) {(1, 1), (3, 3), (3, 1), (2, 3)}
(d) {(1, 1), (3, 3), (3, 1), (1, 2)}
Q.2. Let the relation R in the set 𝐴 = {𝑥 ∈ 𝑍 ∶ 0 ≤ 𝑥 ≤ 12}, given by
𝑅 = {(𝑎, 𝑏): |𝑎 − 𝑏| 𝑖𝑠 𝑎 𝑚𝑢𝑙𝑡𝑖𝑝𝑙𝑒 𝑜𝑓 4}. Then [1], the equivalence class of 1, is:
(a) {1, 5, 9} (b) {0, 1, 2, 5} (c) 𝜙 (d) 𝐴
Q.3. The number of elements in set 𝐴 is 4. The number of possible symmetric relations that can be defined on 𝐴
is: 16 (b) 32 (c) 512 (d) 1024
Q.4. The function 𝑓: 𝑋 ⟶ 𝑌 defined by 𝑓(𝑥) = sin 𝑥 is one - one but not onto, if 𝑋 and 𝑌 are respectively:

𝜋 𝜋
(a) [0, 𝜋] and [0, 1] (b) [− 2 , 2 ] and [−1, 1]
𝜋
(c) [0, 2 ] and [−1, 1] (d) 𝑅 and 𝑅

Q.5. Let 𝐴 = {1, 2, 3, … … , 𝑛} and 𝐵 = {𝑎, 𝑏}. Then the number of surjections from 𝐴 to 𝐵 is:

(a) 𝑛𝑃2 (b) 2𝑛 − 2 (c) 2𝑛 − 1 (d) 2𝑛

ASSERTION - REASONING TYPE QUESTIONS (1 Mark each)


Direction: (Q.6 - Q.7) In the following questions, a statement of Assertion (A) is followed by a statement
of Reason (R). Choose the correct answer out of the following choices:

(a) Both A and R are true and R is the correct explanation of A.


(b) Both A and R are true but R is not the correct explanation of A.
(c) A is true but R is false.
(d) A is false but R is true.
Q.6. Assertion (A): A relation 𝑅 = {(1, 1), (1, 3), (1, 5), (3, 1), (3, 3), (3, 5)} defined on the set 𝐴 =
{1, 3, 5} is
transitive.

Page 12 of 199
Reason (R): A relation 𝑅 on the set 𝐴 transitive iff (a, b) ∈ R and (a, c) ∈ R ⟹ (b, c) ∈ R.
𝑥
Q.7. Assertion (A): Consider the function 𝑓: 𝑅 ⟶ 𝑅 defined by 𝑓(𝑥) = 1+ 𝑥 2 . Then 𝑓 is one – one.
4 1 4
Reason (R): 𝑓(4) = 17 and 𝑓 (4) = 17 .

VERY SHORT ANSWER TYPE - QUESTIONS (VSA) (2 Marks each)

Direction (Q.8 - Q.12) - This section comprises of very short answer type-questions (VSA) of 2 marks
each.

Q.8. Let 𝑅 be the relation in the set Z of all integers given by 𝑅 = {(𝑎, 𝑏) ∶ 2 𝑑𝑖𝑣𝑖𝑑𝑒𝑠 𝑎 − 𝑏}. Is the relation 𝑅
transitive? Write the equivalence class [0].

Q.9. The relation 𝑅 defined by (𝑎, 𝑏) R (𝑐, 𝑑) ⇔ 𝑎 + 𝑑 = 𝑏 + 𝑐 on the set 𝐴 × 𝐴. Where

𝐴 = {1, 2, 3, … . . , 10} is an equivalence relation. Find the equivalence class of the element (3, 4).

1
Q.10. Is the function 𝑓 ∶ 𝑁 ⟶ 𝑅0 defined by 𝑓(𝑥) = one - one and onto? Where 𝑅0 is the set of all non
𝑥

- zero real numbers. Justify your answer.

𝑥−2
Q.11. Let 𝐴 = 𝑅 − {1}. If 𝑓 ∶ 𝐴 ⟶ 𝐴 be a function defined by (𝑥) = 𝑥−1 , show that 𝑓 is onto.

2 4𝑥 + 3
Q.12. Let 𝐴 = 𝑅 − {3}. If 𝑓 ∶ 𝐴 ⟶ 𝐴 be a function defined by (𝑥) = 6𝑥 − 4 , show that 𝑓 is one - one.

SHORT ANSWER TYPE - QUESTIONS (SA) (3 Marks each)

Direction (Q.13 - Q.17) - This section comprises of short answer type-questions (SA) of 3 marks each.

Q.13. Check whether the relation R in the set 𝑅 of real numbers, defined by:

R = {(𝑎, 𝑏): 1 + 𝑎𝑏 > 0}, is reflexive, symmetric or transitive.

Q.14. Check whether the relation R defined in the set 𝐴 = {1, 2, 3, 4, 5, 6} as 𝑅 = {(𝑎, 𝑏): 𝑏 = 𝑎 + 1; 𝑎, 𝑏 ∈ 𝐴}
is reflexive, symmetric or transitive.

Q.15. Show that the relation 𝑅 in the set 𝑁 × 𝑁 defined by (𝑎, 𝑏) R (𝑐, 𝑑) ⇔ 𝑎2 + 𝑑 2 = 𝑏 2 + 𝑐 2 ∀ 𝑎, 𝑏, 𝑐, 𝑑 ∈
𝑁 is an equivalence relation.

Q.16. Prove that the function 𝑓 ∶ 𝑁 → 𝑁 be defined by 𝑓 (𝑥) = 𝑥 2 + 𝑥 + 1, ∀𝑥 ∈𝑵

is injective but not surjective.

𝑥 + 1, 𝑖𝑓 𝑥 𝑖𝑠 𝑜𝑑𝑑
Q.17. Show that 𝑓 ∶ 𝑁 → 𝑁, given by 𝑓(𝑥) = { is a bijection.
𝑥 − 1, 𝑖𝑓 𝑥 𝑖𝑠 𝑒𝑣𝑒𝑛

LONG ANSWER TYPE - QUESTIONS (LA) (5 Marks each)

Page 13 of 199
Direction (Q.18 - Q.19) - This section comprises of long answer type - questions (LA) of 5 marks each.

Q.18. Let 𝑁 denotes the set of all natural numbers and 𝑅 be the relation on 𝑁 × 𝑁 defined by (𝑎, 𝑏) R
(𝑐, 𝑑) ⇔ 𝑎𝑑(𝑏 + 𝑐) = 𝑏𝑐(𝑎 + 𝑑). Show that 𝑅 is an equivalence relation.
𝑥
Q.19. Show that the function 𝑓 ∶ 𝑅 ⟶ {𝑥 ∈ 𝑅: − 1 < 𝑥 < 1} defined by 𝑓(𝑥) = 1 + |𝑥| , 𝑥 ∈ 𝑅 is one -
one and onto function.

COMPETENCY BASED QUESTIONS (4 Marks each)


Direction (Q.20 - Q.21) - This section comprises of 2 case-study / passage-based questions of 4 marks
each.Q.20. Case-Study: Read the Case study given below and attempt the questions given
below:Students of Grade 12, planned to plant saplings along straight lines, parallel to each other to one side
of the playground ensuring that they had enough play area. Let us assume that they planted one of the rows
of the saplings along the line 𝑦 = 𝑥 − 4. Let L be the set of all lines which are parallel on the ground and R
be a relation on L.

Based on the above information answer the following questions: (1 + 1 + 2)

Let 𝑅 = {(𝑙1 , 𝑙2 ): 𝑙1 ⊥ 𝑙2 , 𝑤ℎ𝑒𝑟𝑒 𝑙1 , 𝑙2 ∈ 𝐿}. What type of relation is ?

Let 𝑅 = {(𝑙1 , 𝑙2 ): 𝑙1 ∥ 𝑙2 , 𝑤ℎ𝑒𝑟𝑒 𝑙1 , 𝑙2 ∈ 𝐿}. What type of relation is ?

Check whether the function 𝑓:𝑅 → 𝑅 𝑑𝑒𝑓𝑖𝑛𝑒𝑑 𝑏𝑦 𝑓(𝑥) = 𝑥 − 4 is one-one and onto.

OR (Only for (iii))

Find the range of the function 𝑓.

Q.21. Case-Study: Read the Case study given below and attempt the questions given below:

Sherlin and Danju are playing Ludo at home during Covid-19. While rolling the dice, Sherlin’s sister Raji
observed and noted the possible outcomes of the throw every time belongs to set {1,2,3,4,5,6}. Let A be the

Page 14 of 199
set of players while B be the set of all possible outcomes.

𝐴 = {𝑆, 𝐷}, 𝐵 = {1,2,3,4,5,6}

Based on the above information, answer the following questions: (1 + 1 + 2)


(i) Raji wants to know the number of functions from A to B. How many numbers of functions are
possible?
(ii) Raji wants to know the number of relations possible from A to B. How many numbers relations
are possible?
(iii) Let R be the relation on B defined by 𝑅 = {(1, 2), (2, 2), (1, 3), (3, 4), (3, 1), (4, 3), (5, 5)}.
Is R an equivalence relation?

OR (Only for (iii))

Show that the relation 𝑅 ∶ 𝐵 ⟶ 𝐵 be defined by 𝑅 = {(𝑥, 𝑦) ∶ 𝑦 𝑖𝑠 𝑑𝑖𝑣𝑖𝑠𝑖𝑏𝑙𝑒 𝑏𝑦 𝑥} is a


reflexive
and transitive but not symmetric relation.

Page 15 of 199
2. INVERSE TRIGONOMETRIC FUNCTIONS
M.C.Q. (1 Mark each)
Direction: (Q.1 - Q.5) are multiple choice type questions. Select the correct option.

√3 𝜋
Q.1. The value of cos [cos −1 (− ) + 6 ] is:
2

1
(a) 1 (b) −1 (c) − 2 (d) 0

Q.2. The value of sin(tan−1 𝑥), where |𝑥| < 1 is:


𝑥 1 𝑥 1
(a) √1 − 𝑥 2
(b) √1 (c) √1 (d) √1
− 𝑥2 + 𝑥2 + 𝑥2

3𝜋
Q.3. The value of sin−1 (cos ) is:
5

𝜋 3𝜋 𝜋 3𝜋
(a) (b) (c) − 10 (d) −
10 5 5

Q.4. The principal value of tan−1 √3 − sec −1(−2) is:


𝜋 𝜋 2𝜋 2𝜋
(a) − 3 (b) 3 (c) (d) −
3 3

3𝜋
Q.5. If sin–1x + sin–1y + sin–1z = , then the value of x + y2 + z3 is
2

(a) 1 (b) 3 (c) 2 (d) 5

ASSERTION - REASONING TYPE QUESTIONS (1 Mark each)


Direction: (Q.6 - Q.7) In the following questions, a statement of Assertion (A) is followed by a statement
of Reason (R). Choose the correct answer out of the following choices:

(a) Both A and R are true and R is the correct explanation of A.


(b) Both A and R are true but R is not the correct explanation of A.
(c) A is true but R is false.
(d) A is false but R is true.

1 1
Q.6. Assertion (A): The domain of the function sec −1 2𝑥 is (−∞, − 2 ] ∪ [ 2 , ∞)
𝝅
Reason (R): 𝐬𝐞𝐜 −𝟏 (−𝟐) = − 𝟒

Q.7. Assertion (A): The function 𝑓(𝑥) = sin 𝑥 does not possess inverse if 𝑥 ∈ 𝑅.

Page 16 of 199
Reason (R): The function 𝑓(𝑥) = sin 𝑥 is not one - one and onto if 𝑥 ∈ 𝑅.

VERY SHORT ANSWER TYPE - QUESTIONS (VSA) (2 Marks each)

Direction (Q.8 - Q.12) - This section comprises of very short answer type-questions (VSA) of 2 marks
each.

Q.8. Find the domain of the function: cos−1 (3𝑥 − 2).

sin 𝑥+ cos 𝑥 𝜋 𝜋
Q.9. Express sin−1 ( ), where − < 𝑥 < in the simplest form.
√2 4 4

1 1
Q.10. Prove that: 3 sin−1 𝑥 = sin−1(3𝑥 − 4𝑥 3 ), 𝑥 ∈ [− 2 , 2].

3𝜋
Q.11. Evaluate: sin−1 (sin ) + cos−1 (cos 𝜋) + tan−1 1.
4

1−𝑥 1
Q.12. Show that: tan−1 √1+𝑥 = cos −1 𝑥.
2

SHORT ANSWER TYPE - QUESTIONS (SA) (3 Marks each)

Direction (Q.13 - Q.17) - This section comprises of short answer type-questions (SA) of 3 marks each.

𝜋
Q.13. Solve for x: sin−1(1 − 𝑥) − 2 sin−1 𝑥 = 2

1
Q.14. Find the value of sin (2 tan−1 4) + cos(tan−1 2√2).

𝑥 √3−3𝑥 2 𝜋
Q.15. Prove that: cos−1 𝑥 + cos −1 (2 + ) = 3.
2

Q.16. If cos −1 𝑥 + cos −1 𝑦 + cos−1 𝑧 = 𝜋 , show that: 𝑥 2 + 𝑦 2 + 𝑧 2 + 2𝑥𝑦𝑧 = 1.

1 1+√1+ 𝑥 2
Q.17. Prove that: tan−1 𝑥 = cos−1 (√ ).
2 2 √1+ 𝑥 2

Page 17 of 199
LONG ANSWER TYPE - QUESTIONS (LA) (5 Marks each)
Direction (Q.18 - Q.19) - This section comprises of long answer type - questions (LA) of 5 marks each.

√1+ 𝑥 2 + √1− 𝑥 2 𝜋 1
Q.18. Prove that: tan−1 (√1+ )= + cos −1 𝑥 2 ; −1 < 𝑥 < 1.
𝑥 2 − √1− 𝑥 2 4 2

√1+ sin 𝑥 + √1− sin 𝑥 𝜋 𝑥 𝜋


Q.19. Prove that: cot −1 ( ) = 2 − 2 ; 𝑥 ∈ ( 2 , 𝜋).
√1+ sin 𝑥 − √1− sin 𝑥

COMPETENCY BASED QUESTIONS (4 Marks each)


Direction (Q.20 - Q.21) - This section comprises of 2 case-study / passage-based questions of 4 marks
each.

Q.20. Case-Study: Read the Case study given below and attempt the questions given below:

The Government of India is planning to


fix a hoarding board at the face of a
building on
the road of a busy market for awareness
on COVID-19 protocol. Aditya, Sarita and
Rahim are the three engineers who are
working on this project. “A” is considered
to be a person viewing the hoarding
board 20 metres away from the building, standing at the edge of a pathway nearby. Aditya,
Sarita and Rahim suggested to the firm to place the hoarding board at three different
locations namely C, D and E. “C” is at the height of 10 metres from the ground level.
For the viewer, A, the angle of elevation of “D” is double the angle of elevation of “C”. The
angle of elevation of “E” is triple the angle of elevation of “C” for the same viewer. Look at
the figure given and based on the above information answer the following:

Based on the above information answer the following questions: (1 + 1 + 2)

(i) Find the measure of ∠𝐶𝐴𝐵.

(ii) Find the measure of ∠𝐷𝐴𝐵.

Page 18 of 199
(iii) Find the measure of ∠𝐸𝐴𝐵.

OR (Only for (iii))


A’ is another viewer standing on the same line of observation across the road. If the width of the
road is 5 meters, then what is the difference between ∠CAB and ∠CA’B.

Q.21. Case-Study: Read the Case study given below and attempt the questions given below:

Today in the class of mathematics, Mr


Bhaskar is explaining the inverse
trigonometric functions. He draws the graph
of the sin−1 𝑥 and write down the following
about the principal value branch of the
function y = sin−1 𝑥. Principal value
branch of the function y = sin−1 𝑥:
It is a function with domain [−1, 1] and range
3𝜋 𝜋 𝜋 𝜋 𝜋 3𝜋
[− , − 2 ] , [− 2 , 2 ] or [ 2 , ] and so on.
2 2

Corresponding to each interval, we get a


branch of the function sin−1 𝑥 . The branch
𝜋 𝜋
with range [− 2 , 2 ] is called the Principal

value branch. Thus


𝜋 𝜋
sin−1: [−1, 1] ⟶ [− 2 , 2 ].

Based on the above information, answer the following questions: (1 + 1 + 2)


(iv) Find the domain of sin−1 √𝑥 − 1
(v) Find the domain of sin−1 [𝑥], where [𝑥] is the greatest integer function.
𝜋 1
(vi) Find the value of sin (3 − sin−1 (− 2))

OR (Only for (iii))


2𝜋 2𝜋
Find the principal value of: cos −1 (cos ) + sin−1 (sin ).
3 3

Page 19 of 199
3. Matrices
MCQ’s
1 1, 𝑤ℎ𝑒𝑛 𝑖 ≠ 𝑗
𝐼𝑓. 𝐴 = [𝑎𝑖𝑗 ] 𝑖𝑠 𝑎 𝑠𝑞𝑢𝑎𝑟𝑒 𝑚𝑎𝑡𝑟𝑖𝑥 𝑜𝑓 𝑜𝑟𝑑𝑒𝑟 2, 𝑠𝑢𝑐ℎ 𝑡ℎ𝑎𝑡 𝑎𝑖𝑗 = { }
0, 𝑤ℎ𝑒𝑛 𝑖 = 𝑗

𝑡ℎ𝑒𝑛 𝐴2 =

(𝑎) [1 0] (𝑏) [1 1] (𝑐) [1 1 (𝑑) 1 0


] [ ]
1 0 0 0 1 0 0 1

0 1 1 0
Solution 𝐴 = [ ] , 𝐴2 = [ ],
1 0 0 1

2 𝐼𝑓 𝑡ℎ𝑒 𝑜𝑟𝑑𝑒𝑟 𝑜𝑓 𝑚𝑎𝑡𝑟𝑖𝑥 𝐴 𝑖𝑠 4 × 3, 𝑡ℎ𝑒 𝑜𝑟𝑑𝑒𝑟 𝑜𝑓 𝑚𝑎𝑡𝑟𝑖𝑥 𝐵 𝑖𝑠 4 × 5 𝑎𝑛𝑑 𝑡ℎ𝑒


𝑜𝑟𝑑𝑒𝑟 𝑜𝑓 𝑚𝑎𝑡𝑟𝑖𝑥 𝐶 𝑖𝑠 7 × 3, 𝑡ℎ𝑒𝑛 𝑡ℎ𝑒 𝑜𝑟𝑑𝑒𝑟 𝑜𝑓 𝑡ℎ𝑒 𝑚𝑎𝑡𝑟𝑖𝑥 (𝐴′ × 𝐵)′ × 𝐶 ′ 𝑖𝑠 :

(a) 4 × 5 (𝑏) 3 × 7 (𝑐)4 × 3 (𝑑)5 × 7

Solution – correct option is – option (𝑑) 5 × 7

Explanation : 𝑜𝑟𝑑𝑒𝑟 𝑜𝑓 𝑚𝑎𝑡𝑟𝑖𝑥 𝐴′ 𝑖𝑠 3 × 4 𝑎𝑛𝑑 𝑜𝑟𝑑𝑒𝑟 𝑜𝑓 𝑚𝑎𝑡𝑟𝑖𝑥 𝐵 𝑖𝑠 4 × 5


⇒ 𝑜𝑟𝑑𝑒𝑟 𝑜𝑓 𝑚𝑎𝑡𝑟𝑖𝑥 (𝐴′ × 𝐵) 𝑖𝑠 3 × 5
⇒ 𝑜𝑟𝑑𝑒𝑟 𝑜𝑓 𝑚𝑎𝑡𝑟𝑖𝑥 (𝐴′ × 𝐵)′ 𝑖𝑠 5 × 3
𝑁𝑜𝑤 𝑜𝑟𝑑𝑒𝑟 𝑜𝑓 𝑚𝑎𝑡𝑟𝑖𝑥 𝐶 𝑖𝑠 7 × 3
⇒ 𝑜𝑟𝑑𝑒𝑟 𝑜𝑓 𝑚𝑎𝑡𝑟𝑖𝑥 𝐶 ′ 𝑖𝑠 3 × 7
⇒ order of matrix (𝐴′ × 𝐵)′ × 𝐶 ′ 𝑖𝑠 5 × 7
3
4 2
𝐼𝑓, 𝐴 = [ ] , 𝑡ℎ𝑒𝑛 𝑣𝑎𝑙𝑢𝑒 𝑜𝑓 (𝐴 − 2𝐼)(𝐴 − 3𝐼)𝑖𝑠
−1 1

(𝑎)0 (𝑏)𝐼 (𝑐)6𝐼 (𝑑)5𝐼

Solution – correct option is – option (𝑎) 0

4 2
Explanation : we have 𝐴 = [ ]
−1 1

4 2 1 0 4 2 1 0
𝐿𝐻𝑆 = (𝐴 − 2𝐼)(𝐴 − 3𝐼) = {[ ]− 2[ ]} {[ ] − 3[ ]}
−1 1 0 1 −1 1 0 1

2 2 1 2 2−2 4−4 0 0
=[ ][ ]=[ ]= [ ]=0
−1 −1 −1 −2 −1 + 1 −2 + 2 0 0

4 𝐴 = [𝑎𝑖𝑗 ]𝑚×𝑛 is a square matrix if :


(a) 𝑚 = 𝑛 (𝑏)𝑚 < 𝑛 (𝑐)𝑚 > 𝑛 (𝑑) 𝑛𝑜𝑛𝑒 𝑜𝑓 𝑡ℎ𝑒𝑠𝑒
Solution (a) m = n

Page 20 of 199
5
𝑥 5 3 −4 7 6
𝐼𝑓, 2 [ ]+ [ ]= [ ] . 𝑡ℎ𝑒𝑛 𝑣𝑎𝑙𝑢𝑒 𝑜𝑓 𝑥 𝑖𝑠 ∶
7 𝑦−3 1 2 15 14

( a ) 1 ( b) 2 (c ) 3 (d) 4

Solution : 2x+ 3 = 7 hence x = 2 correct option is ( b)


6
1 0 0
𝑀𝑎𝑡𝑟𝑖𝑥 [0 1 0] 𝑐𝑎𝑛𝑛′ 𝑡 𝑏𝑒 𝑡𝑒𝑟𝑚𝑒𝑑 𝑎𝑠 :
0 0 1

(𝑎)𝑆𝑞𝑢𝑎𝑟𝑒 𝑚𝑎𝑡𝑟𝑖𝑥
(𝑏)𝑈𝑛𝑖𝑡 𝑚𝑎𝑡𝑟𝑖𝑥
(𝑐) 𝐷𝑖𝑎𝑔𝑜𝑛𝑎𝑙 𝑚𝑎𝑡𝑟𝑖𝑥
(𝑑)𝑆𝑐𝑎𝑙𝑎𝑟 𝑚𝑎𝑡𝑟𝑖𝑥
(𝑒) 𝐼𝑑𝑒𝑛𝑡𝑖𝑡𝑦 𝑚𝑎𝑡𝑟𝑖𝑥
(𝑓) 𝐼𝑑𝑒𝑚𝑝𝑜𝑡𝑒𝑛𝑡 𝑚𝑎𝑡𝑟𝑖𝑥
(𝑔) 𝑁𝑢𝑙𝑙 𝑀𝑎𝑡𝑟𝑖𝑥

1 0 0
Solution: correct option is ( g) : 𝑀𝑎𝑡𝑟𝑖𝑥 [0 1 0] 𝑐𝑎𝑛𝑛′ 𝑡 𝑏𝑒 𝑡𝑒𝑟𝑚𝑒𝑑 𝑎𝑠 𝑁𝑢𝑙𝑙 𝑀𝑎𝑡𝑟𝑖𝑥.
0 0 1

ASSERTION – REASON QUESTION - 1


Assertion (A) : Matrices of any order can be added
Reason(R ): Two matrices can be added, if they are of same order.
Now pick the correct option :
(a) Both A and R are individually true and R is the correct explanation of A.
(b) Both A and R are individually true and R is NOT the correct explanation of A.
(c) A is true but R is false
(d) A is false but R is true
(e) Both A and R are false

Solution : Correct option is – option ( d) A is false but R is true, Two matrices can be added, only if they
are of same order.
ASSERTION – REASON QUESTION – 2
Assertion (A) : Matrix (A.A’ ) is always a symmetric matrix for any matrix A
Reason(R ): Matrix A is symmetric is, A’ = A
Now pick the correct option :
(a) Both A and R are individually true and R is the correct explanation of A.
(b) Both A and R are individually true and R is NOT the correct explanation of A.
(c) A is true but R is false
(d) A is false but R is true
(e) Both A and R are false

Solution: Correct option is – option ( a ) Both A and R are individually true and R is the correct
explanation of A.
(A.A’)’ = (A’)’A’ = A.A’: Hence matrix (A.A’) is symmetric.

Page 21 of 199
02 MARKS QUESTIONS
1 (𝑖 − 𝑗)2
𝐶𝑜𝑛𝑠𝑡𝑟𝑢𝑐𝑡 2 × 2 𝑚𝑎𝑡𝑟𝑖𝑥, 𝐴 = [𝑎𝑖𝑗 ] 𝑤ℎ𝑜𝑠𝑒 𝑒𝑙𝑒𝑚𝑒𝑛𝑡 𝑖𝑠 𝑔𝑖𝑣𝑒𝑛 𝑏𝑦, 𝑎𝑖𝑗 =
2
Solution :
(𝑖 − 𝑗)2
𝑎𝑖𝑗 =
2
(1 − 1)2
𝑁𝑜𝑤 𝑎11 = =0
2
2
(1 − 2) 1
𝑎12 = =
2 2
(2 − 1)2 1
𝑎21 = =
2 2
(2 − 2)2
𝑎22 = = 0
2
1
𝑎11 𝑎12 0 2
Now the matrix , 𝐴 = [𝑎 ] = [1 ]
21 𝑎22 0 2
2
𝑥𝑦 4 8 𝑤
𝐼𝑓, [ ]= [ ] , 𝑡ℎ𝑒𝑛 𝑓𝑖𝑛𝑑 𝑡ℎ𝑒 𝑣𝑎𝑙𝑢𝑒 𝑜𝑓 (𝑥 + 𝑦 + 𝑧).
𝑧+6 𝑥+𝑦 0 6
Solution : Comparing the corresponding elements , we get
𝑥𝑦 = 8, 𝑤 = 4, 𝑧 + 6 = 0, 𝑥 + 𝑦 = 6
Consider 𝑥 + 𝑦 = 6
⇒𝑥+𝑦+𝑧 =6+𝑧=0
𝐻𝑒𝑛𝑐𝑒 𝑣𝑎𝑙𝑢𝑒 𝑜𝑓 𝑥 + 𝑦 + 𝑧 𝑖𝑠 0.

3
𝐼𝑓 𝑎 𝑚𝑎𝑡𝑟𝑖𝑥 ℎ𝑎𝑠 18 𝑒𝑙𝑒𝑚𝑒𝑛𝑡𝑠, 𝑡ℎ𝑒𝑛 𝑤𝑟𝑖𝑡𝑒 𝑡ℎ𝑒 𝑝𝑜𝑠𝑠𝑖𝑏𝑙𝑒 𝑜𝑟𝑑𝑒𝑟𝑠.

Solution : 𝑇ℎ𝑒 𝑝𝑜𝑠𝑠𝑖𝑏𝑙𝑒 𝑜𝑟𝑑𝑒𝑟𝑠 𝑎𝑟𝑒 1 × 18, 2 × 9, 3 × 6, 6 × 3, 9 × 2, 18 × 1

4
1 2 𝑥
𝐼𝑓, [2𝑥 3] [ ] [ ] = 0, 𝑓𝑖𝑛𝑑 𝑥.
−3 0 3

1 2 𝑥
Solution : [2𝑥 3] [ ][ ] = 0
−3 0 3
𝑥
⇒[2𝑥 − 9 4𝑥] [3] = 0

⇒ [2𝑥 2 − 9𝑥 + 12𝑥] = 0

⇒ [2𝑥 2 + 3𝑥] = 0

⇒ 2𝑥 2 + 3𝑥 = 0
3
⇒ 𝑥 = 0 𝑜𝑟 𝑥 = −
2

Page 22 of 199
0 2𝑏 −2
𝑀𝑎𝑡𝑟𝑖𝑥 𝐴 = [ 3 1 3 ] 𝑖𝑠 𝑔𝑖𝑣𝑒𝑛 𝑡𝑜 𝑏𝑒 𝑠𝑦𝑚𝑚𝑒𝑡𝑟𝑖𝑐 ,
3𝑎 3 −1

𝑡ℎ𝑒𝑛 𝑓𝑖𝑛𝑑 𝑡ℎ𝑒 𝑣𝑎𝑙𝑢𝑒𝑠 𝑜𝑓 𝑎 𝑎𝑛𝑑 𝑏.

Solution : Matrix A is symmetric hence A = A’

0 2𝑏 −2 0 3 3𝑎
⇒[3 1 3 ] = [ 2𝑏 1 3]
3𝑎 3 −1 −2 3 −1

Comparing the corresponding elements

2b= 3 and – 2 = 3a

3 3
𝑏= 𝑎𝑛𝑑 𝑎 = −
2 2

03 MARKS QUESTIONS
1 𝐼𝑓, 𝐵 = [𝑏𝑖𝑗 ] 𝑖𝑠 𝑎 𝑠𝑞𝑢𝑎𝑟𝑒 𝑚𝑎𝑡𝑟𝑖𝑥 𝑜𝑓 𝑜𝑟𝑑𝑒𝑟 2 × 2 𝑠𝑢𝑐ℎ 𝑡ℎ𝑎𝑡 𝑏𝑖𝑗 = 𝑖 3 − 𝑗 3 , 𝑡ℎ𝑒𝑛 𝑐ℎ𝑒𝑐𝑘
𝑤ℎ𝑒𝑡ℎ𝑒𝑟 𝐵 𝑖𝑠 𝑆𝑦𝑚𝑚𝑒𝑡𝑟𝑖𝑐 𝑜𝑟 𝑆𝑘𝑒𝑤 𝑠𝑦𝑚𝑚𝑒𝑡𝑟𝑖𝑐 𝑚𝑎𝑡𝑟𝑖𝑐.

Solution : we have 𝑏𝑖𝑗 = 𝑖 3 − 𝑗 3


𝑏11 = 13 − 13 = 0
𝑏12 = 13 − 23 = −7
𝑏21 = 23 − 13 = 7
𝑏22 = 23 − 23 = 0
0 −7
𝑁𝑜𝑤, 𝐵 = [ ]
7 0

0 7
𝐴𝑛𝑑, 𝐵′ = [ ]
−7 0

0 −7
=− [ ]
7 0

=−𝐵

𝐻𝑒𝑛𝑐𝑒 , 𝐵 ′ = −𝐵, 𝑡ℎ𝑒𝑟𝑒𝑓𝑜𝑟𝑒 𝐵 𝑖𝑠 𝑠𝑘𝑒𝑤 𝑠𝑦𝑚𝑚𝑒𝑡𝑟𝑖𝑐 𝑚𝑎𝑡𝑟𝑖𝑥

2
7 0 2 −3
𝐼𝑓, 𝐴 = [3 2] 𝑎𝑛𝑑 𝐵 = [ 4 5 ] , 𝑓𝑖𝑛𝑑 𝑡ℎ𝑒 𝑚𝑎𝑡𝑟𝑖𝑥 𝑋 𝑠𝑢𝑐ℎ 𝑡ℎ𝑎𝑡 2𝐴 + 3𝑋 = 8𝐵.
5 1 −6 5

Solution : 𝑊𝑒 ℎ𝑎𝑣𝑒, 2𝐴 + 3𝑋 = 8𝐵
⇒ 3𝑋 = 8𝐵 − 2𝐴
2 −3 7 0
1
⇒ 𝑋 = 3 (8 [ 4 5 ] − 2 [3 2])
−6 5 5 1

Page 23 of 199
16 −24 14 0
1
⇒𝑋 = ([ 32 40 ] − [ 6 4])
3
−48 40 10 2

16 − 14 −24
1
⇒𝑋 = ([ 32 − 6 40 − 4])
3
−48 − 10 40 − 2

2 −24
1
⇒ 𝑋 = 3 ([ 26 36 ])
−58 38

3
2 4 −6
𝐸𝑥𝑝𝑟𝑒𝑠𝑠 𝑡ℎ𝑒 𝑚𝑎𝑡𝑟𝑖𝑥 𝐴 = [7 3 5 ] 𝑎𝑠 𝑡ℎ𝑒 𝑠𝑢𝑚 𝑜𝑓 𝑎
1 −2 4
𝑠𝑦𝑚𝑚𝑒𝑡𝑟𝑖𝑐 𝑎𝑛𝑑 𝑠𝑘𝑒𝑤 𝑠𝑦𝑚𝑚𝑒𝑡𝑟𝑖𝑐 𝑚𝑎𝑡𝑟𝑖𝑥.

2 7 1
Solution : We have 𝐴′ = [ 4 3 −2]
−6 5 4

For expressing the matrix A as the sum of symmetric and skew symmetric matrix, we can write
A = P+Q, where
1
𝑃 = (𝐴 + 𝐴′ ), 𝑤ℎ𝑖𝑐ℎ 𝑖𝑠 𝑠𝑦𝑚𝑚𝑒𝑡𝑟𝑖𝑐 𝑝𝑎𝑟𝑡
2
And
1
𝑄 = (𝐴 − 𝐴′ ), 𝑤ℎ𝑖𝑐ℎ 𝑖𝑠 𝑠𝑦𝑚𝑚𝑒𝑡𝑟𝑖𝑐 𝑝𝑎𝑟𝑡
2

2 4 −6 2 7 1 4 11 −5
1 1
Now 𝑃 = 2 ([7 3 5 ]+[ 4 3 −2]) = [ 11 6 3]
2
1 −2 4 −6 5 4 −5 3 8

1 2 4 −6 2 7 1 1 0 −3 −7
𝑄 = ([7 3 5 ]−[ 4 3 −2]) = [3 0 7]
2 2
1 −2 4 −6 5 4 7 −7 0

Hence the matrix A can be written as the sum of symmetric and skew symmetric matrix

4 11 −5 0 −3 −7
1 1
A = 2 [ 11 6 3 ] + 2 [3 0 7]
−5 3 8 7 −7 0

4
𝐹𝑜𝑟 𝑡ℎ𝑒 𝑓𝑜𝑙𝑙𝑜𝑤𝑖𝑛𝑔 𝑚𝑎𝑡𝑟𝑖𝑐𝑒𝑠 𝐴 𝑎𝑛𝑑 𝐵, 𝑣𝑒𝑟𝑖𝑓𝑦 𝑡ℎ𝑎𝑡 (𝐴𝐵)′ = 𝐵 ′ 𝐴′

1
𝐴 = [−4] , 𝐵 = [−1 2 1]
3

Solution :

Page 24 of 199
1 −1 2 1
𝑊𝑒 ℎ𝑎𝑣𝑒, 𝐴𝐵 = [−4] [−1 2 1] = [ 4 −8 −4]
3 −3 6 3

−1 2 1 ′
𝑁𝑜𝑤, 𝐿𝐻𝑆 = (𝐴𝐵)′ = [ 4 −8 −4]
−3 6 3

−1 4 −3
= [ 2 −8 6 ] … . (𝑖)
1 −4 3

1 ′ −1
𝑎𝑛𝑑 , 𝑅𝐻𝑆 = 𝐵 ′ 𝐴′ = [−1 ′
2 1] [−4] = [ 2 ] [1 −4 3]
3 1

−1 4 −3
= [2 −8 6 ] … . (𝑖𝑖)
1 −4 3

LHS = RHS , Hence verified 𝑡ℎ𝑎𝑡 (𝐴𝐵)′ = 𝐵 ′ 𝐴′ .

5
2 −1 5 2 2 5
𝐿𝑒𝑡 , 𝐴 = [ ],𝐵 = [ ],𝐶 = [ ]
3 4 7 4 3 8

Find a matrix D such that CD-AB=0

𝑥 𝑦
Solution : 𝐿𝑒𝑡 𝑚𝑎𝑡𝑟𝑖𝑥 𝐷 = [ ]
𝑧 𝑤

Given that CD-AB=0

2 5 𝑥 𝑦 2 −1 5 2
⇒[ ][ ]− [ ][ ]=0
3 8 𝑧 𝑤 3 4 7 4

2𝑥 + 5𝑧 2𝑦 + 5𝑤 10 − 7 4−4
⇒[ ]−[ ]=0
3𝑥 + 8𝑧 3𝑦 + 8𝑤 15 + 28 6 + 16

2𝑥 + 5𝑧 2𝑦 + 5𝑤 3 0
⇒[ ]=[ ]
3𝑥 + 8𝑧 3𝑦 + 8𝑤 43 22

Comparing corresponding elements of both sides, we get


2𝑥 + 5𝑧 = 3, 3𝑥 + 8𝑧 = 43 𝑎𝑛𝑑 2𝑦 + 5𝑤 = 0, 3𝑦 + 8𝑤 = 22

𝑆𝑜𝑙𝑣𝑖𝑛𝑔 𝑡ℎ𝑒 𝑒𝑞𝑢𝑎𝑡𝑖𝑜𝑛𝑠 𝑤𝑒 𝑔𝑒𝑡

𝑥 = −191, 𝑦 = −110, 𝑧 = 77 𝑎𝑛𝑑 𝑤 = 44

𝑥 𝑦 −191 −110
𝑇ℎ𝑒𝑟𝑒𝑓𝑜𝑟𝑒 𝑚𝑎𝑡𝑟𝑖𝑥 𝐷 = [ ]= [ ]
𝑧 𝑤 77 44

05 MARKS QUESTIONS
1

Page 25 of 199
1 −1 2
𝐼𝑓, 𝑓(𝑥) = 2𝑥 3 + 5𝑥 2 − 𝑥, 𝑓𝑖𝑛𝑑 𝑓(𝐴) 𝑤ℎ𝑒𝑟𝑒 𝐴 = [2 0 1]
1 −1 0

Solution: 𝑊𝑒 ℎ𝑎𝑣𝑒 𝑓(𝑥) = 2𝑥 3 + 5𝑥 2 − 𝑥

⇒ 𝑓(𝐴) = 2𝐴3 + 5𝐴2 − 𝐴


1 −1 2 1 −1 2
2
𝑁𝑜𝑤 𝐴 = [2 0 1] [2 0 1]
1 −1 0 1 −1 0

1−2+2 −1 + 0 − 2 2−1+0
= [2 + 0 + 1 −2 + 0 − 1 4 + 0 + 0]
1−2+0 −1 + 0 + 0 2−1+0

1 −3 1
= [ 3 −3 4]
−1 −1 1

1 −3 1 1 −1 2
3 2
𝐴𝑛𝑑 𝐴 = 𝐴 . 𝐴 = [ 3 −3 4] [2 0 1]
−1 −1 1 1 −1 0

1−6+1 −1 + 0 − 1 2−3+0
= [ 3−6+4 −3 + 0 − 4 6−3+0 ]
−1 − 2 + 1 1+0−1 −2 − 1 + 0

−4 −2 −1
= [ 1 −7 3 ]
−2 0 −3

−4 −2 −1 1 −3 1 1 −1 2
𝐻𝑒𝑛𝑐𝑒, 𝑓(𝐴) = 2 [ 1 −7 3 ] + 5 [ 3 −3 4] − [2 0 1]
−2 0 −3 −1 −1 1 1 −1 0

−8 −4 −2 5 −15 5 1 −1 2
= [ 2 −14 6 ] + [ 15 −15 20] − [2 0 1]
−4 0 −6 −5 −5 5 1 −1 0

−4 −18 1
= [ 15 −29 25 ]
−10 −4 −1

2

0 − tan
𝐼𝑓, 𝐴 = [ 2 ] 𝑎𝑛𝑑 𝐼 𝑖𝑠 𝑡ℎ𝑒 𝑖𝑑𝑒𝑛𝑡𝑖𝑡𝑦 𝑚𝑎𝑡𝑟𝑖𝑥 𝑜𝑓 𝑜𝑟𝑑𝑒𝑟 2,

tan 0
2

𝑐𝑜𝑠 ∝ −𝑠𝑖𝑛 ∝
𝑡ℎ𝑒𝑛 𝑠ℎ𝑜𝑤 𝑡ℎ𝑎𝑡 𝐼 + 𝐴 = (𝐼 − 𝐴) [ ]
𝑠𝑖𝑛 ∝ 𝑐𝑜𝑠 ∝

Page 26 of 199

1 0 0 − tan 2
Solution : 𝐿. 𝐻. 𝑆 = 𝐼 + 𝐴 = [ ]+[ ∝ ]
0 1 tan 2 0


1 − tan
= [ 2]

tan 1
2
∝ ∝
∝ 1− 𝑡𝑎𝑛2 2𝑡𝑎𝑛
2 2
𝑃𝑢𝑡𝑡𝑖𝑛𝑔 tan 2 = 𝑡 𝑎𝑛𝑑 𝑎𝑝𝑝𝑙𝑦𝑖𝑛𝑔 cos ∝ = ∝ & 𝑠𝑖𝑛 ∝ = ∝
1+ 𝑡𝑎𝑛2 1+ 𝑡𝑎𝑛2
2 2

1 −𝑡
𝐿. 𝐻. 𝑆 = 𝐼 + 𝐴 = [ ]
𝑡 1

𝑐𝑜𝑠 ∝ −𝑠𝑖𝑛 ∝
𝑅𝐻𝑆 = (𝐼 − 𝐴) [ ]
𝑠𝑖𝑛 ∝ 𝑐𝑜𝑠 ∝
∝ ∝
1 − 𝑡𝑎𝑛2 2 2𝑡𝑎𝑛 2
∝ ∝ − ∝
0 − tan 1 + 𝑡𝑎𝑛2 2 1 + 𝑡𝑎𝑛2 2
1 0 2
= ([ ]−[ ∝ ]) ∝ ∝
0 1 2𝑡𝑎𝑛 2 1 − 𝑡𝑎𝑛2 2
tan 0
2
2∝ ∝
1 + 𝑡𝑎𝑛2 2 ]
[ 1 + 𝑡𝑎𝑛 2

1 − 𝑡2 2𝑡

1 𝑡 1 + 𝑡2 1 + 𝑡2
=[ ]
−𝑡 1 2𝑡 1 − 𝑡2
[ 1 + 𝑡2 1 + 𝑡2 ]

1 − 𝑡2 2𝑡 2 2𝑡 𝑡(1 − 𝑡 2 )
+ − +
= 1 + 𝑡2 1 + 𝑡2 1 + 𝑡2 1 + 𝑡2
𝑡(1 − 𝑡 2 ) 2𝑡 2𝑡 2
1 − 𝑡2
[ − + + ]
1 + 𝑡2 1 + 𝑡2 1 + 𝑡2 1 + 𝑡2

1 + 𝑡2 −2𝑡 + 𝑡 − 𝑡 3
= 1 + 𝑡2 1 + 𝑡2
– 𝑡 + 𝑡 2 + 2𝑡 1 + 𝑡2
[ 1 + 𝑡2 1 + 𝑡2 ]

(1 + 𝑡 2 )
1 −𝑡
= 1 + 𝑡2
(1 + 𝑡 2 )
[𝑡 1 + 𝑡2
1 ]

1 −𝑡
= [ ]
𝑡 1

Page 27 of 199

1 − tan
= [ 2 ] = 𝐿𝐻𝑆

tan 1
2

CASE BASED QUESTION – 1


On her birthday, Seema decided to donate some money to children of an orphanage home .If there
were 8 children less, everyone would have got Rs.10 more. However if there were 16 children more
everyone would have got Rs.10 less. Let the number of children be x and the amount distributed by
Seema on one child be y (in Rs.).
Based on the above information answer the following:
1. Formulate the equations between x and y.
2. Formulate the matrix equations representing the relation between x and y.
3. Find the number of children who were given some money by Seema.
Solution 1.
Here the number of children is x and the amount distributed by Seema on one child is y.
Total money distributed = xy
𝐴𝑐𝑐𝑜𝑟𝑑𝑖𝑛𝑔 𝑡𝑜 𝑡ℎ𝑒 𝑔𝑖𝑣𝑒𝑛 𝑞𝑢𝑒𝑠𝑡𝑖𝑜𝑛, (𝑥 − 8)(𝑦 + 10) = 𝑥𝑦
⇒𝑥𝑦 − 8𝑦 + 10𝑥 − 80 = 𝑥𝑦
⇒10𝑥 − 8𝑦 = 80
⇒5𝑥 − 4𝑦 = 40 … . (𝑖)
𝑎𝑛𝑑 (𝑥 + 16)(𝑦 − 10) = 𝑥𝑦
⇒𝑥𝑦 + 16𝑦 − 10𝑥 − 160 = 𝑥𝑦
⇒−10𝑥 + 16𝑦 = 160
⇒5𝑥 − 8𝑦 = −80 … . (𝑖𝑖)
Hence from (i) and (ii) the equations are
5𝑥 − 4𝑦 = 40 & 5𝑥 − 8𝑦 = −80
Solution 2.
5 −4 𝑥 40
The matrix form is [ ] [𝑦 ] = [ ]
5 −8 −80
Solution 3.
Solving the equations
5𝑥 − 4𝑦 = 40 & 5𝑥 − 8𝑦 = −80
We get x = 32
CASE BASED QUESTION – 2
A manufacture produces products P,Q and R which he sells in two markets A and B. Annual sales are
indicated below :
Market Products
P Q R
A 5000 3000 6000
B 7000 9000 5000

The unit sales price of P,Q and R are Rs.3, Rs.2 and Rs.1 respectively
(A) Calculate the total revenue ( 𝑅1 ) 𝑖𝑛 𝑚𝑎𝑟𝑘𝑒𝑡 𝐴 ?
(B) Find the total revenue ( 𝑅2 ) 𝑖𝑛 𝑚𝑎𝑟𝑘𝑒𝑡 𝐵 ?
(C) If the unit costs of the above products P, Q and R are Rs.2.50, Rs.1 and Rs.0.50 respectively,
then write the matrix that present the total cost C of all the products produced?
3
Solution A. [5000 3000 6000] [2]
1
= [15000 + 6000 + 6000] 𝑖. 𝑒. 𝑅𝑠. 27,000

Page 28 of 199
3
Solution B. [7000 9000 5000] [2]
1

= [21000 + 18000 + 5000] 𝑖. 𝑒. 𝑅𝑠. 44,000

2.50
5000 3000 6000
Solution C. [ ][ 1 ]
7000 9000 5000
0.50

CASE BASED QUESTION – 3


All the shops were closed during lockdown due to corona pandemic. When lockdown ended, Anjali And
her parents decided to go to the jewellery shop to do the shopping for Anjali’s wedding. A shopkeeper
sells 6 rings, 10 bracelets and 12 necklaces in the first week and 12 rings, 06 bracelets and 08 necklaces
in the next week.
1. Represent the information in matrix form.
2. Represent the matrix A formed in part 1 of this question as a sum of symmetric and skew
symmetric matrix.
3. For matrix A verify the property that (A’)’ = A.
Solution 1. The above information can be written as :
Rings Bracelets Necklaces
First week 6 10 12
Second week 12 6 8

6 10 12
Hence the required matrix is [ ]
12 6 8
Solution 2.
For expressing the matrix A as the sum of symmetric and skew symmetric matrix, we can write
A = P+Q, where
1
𝑃 = (𝐴 + 𝐴′ ), 𝑤ℎ𝑖𝑐ℎ 𝑖𝑠 𝑠𝑦𝑚𝑚𝑒𝑡𝑟𝑖𝑐 𝑝𝑎𝑟𝑡
2
And
1
𝑄 = (𝐴 − 𝐴′ ), 𝑤ℎ𝑖𝑐ℎ 𝑖𝑠 𝑠𝑦𝑚𝑚𝑒𝑡𝑟𝑖𝑐 𝑝𝑎𝑟𝑡
2
6 8
6 10 12
𝑁𝑜𝑤 , 𝐴 + 𝐴′ = [ ] + [10 6]
12 6 8
12 8
𝑤ℎ𝑖𝑐ℎ 𝑖𝑠 𝑢𝑛𝑑𝑒𝑓𝑖𝑛𝑒𝑑 𝑏𝑒𝑐𝑎𝑢𝑠𝑒 𝑓𝑜𝑟 𝑎𝑑𝑑𝑖𝑡𝑖𝑜𝑛 𝑏𝑜𝑡ℎ 𝑡ℎ𝑒 𝑚𝑎𝑡𝑟𝑖𝑐𝑒𝑠 𝑠ℎ𝑜𝑢𝑙𝑑 𝑏𝑒 𝑜𝑓 𝑠𝑎𝑚𝑒 𝑜𝑟𝑑𝑒𝑟.
Similarly 𝐴 − 𝐴′ 𝑖𝑠 𝑎𝑙𝑠𝑜 𝑢𝑛𝑑𝑒𝑓𝑖𝑛𝑒𝑑.
Hence we can’t’ represent the matrix A as the sum of a symmetric and skew symmetric matrix.
6 10 12
Solution 3. 𝐴 = [ ]
12 6 8
6 8
⇒ A’ = [10 6]
12 8
6 10 12
⇒ (A’)’ =[ ]=𝐴
12 6 8
⇒ (A’)’ = 𝐴

Page 29 of 199
4. Determinants
MCQ’s

1
𝐼𝑓, 𝐴 = [𝑎𝑖𝑗 ] 𝑖𝑠 𝑎 𝑚𝑎𝑡𝑟𝑖𝑥 𝑜𝑓 𝑜𝑟𝑑𝑒𝑟 2 × 2, 𝑠𝑢𝑐ℎ 𝑡ℎ𝑎𝑡 |𝐴| = −15 𝑎𝑛𝑑 𝑐𝑖𝑗 𝑟𝑒𝑝𝑟𝑒𝑠𝑒𝑛𝑡𝑠 𝑡ℎ𝑒
𝑐𝑜𝑓𝑎𝑐𝑡𝑜𝑟 𝑜𝑓 𝑎𝑖𝑗 𝑡ℎ𝑒𝑛, 𝑎21 𝑐21 + 𝑎22 𝑐22 =
(a) 15 (b) - 15 (c) 0 (d) -1

Solution (b) – 15
2
2𝑥 5 6 −2
𝐼𝑓 | |= | | , 𝑡ℎ𝑒𝑛 𝑡ℎ𝑒 𝑣𝑎𝑙𝑢𝑒 𝑜𝑓 𝑥 𝑖𝑠
8 𝑥 7 3

(𝑎) 3 (𝑏 ) ± 3 (𝑐) ± 6 (𝑑) 6

Solution : Correct option is – option ( c )

2𝑥 5 6 −2
| |= | |
8 𝑥 7 3

⇒ 2 𝑥 2 − 40 = 18 + 40

⇒ 2 𝑥 2 = 18 + 40

⇒ 𝑥 2 = 36

⇒𝑥 = ±6

3 If A is a skew – symmetric matrix of order 3, then the value of |𝐴| 𝑖𝑠


(a) 0 (b) 2 (c) 3 (d) 6

Correct option – (a) 0


Explanation : 𝑆𝑖𝑛𝑐𝑒 𝐴 𝑖𝑠 𝑠𝑘𝑒𝑤 𝑠𝑦𝑚𝑚𝑒𝑡𝑟𝑖𝑐 ℎ𝑒𝑛𝑐𝑒
, 𝐴′ = −𝐴

⇒ |𝐴′| = ⌈− 𝐴⌉

⇒ |𝐴| = (−1)3 ⌈ 𝐴⌉ [ |𝑘𝐴| = 𝑘 𝑛 |𝐴| & |𝐴′ | = |𝐴|]

⇒ |𝐴| = − ⌈ 𝐴⌉

⇒ 2|𝐴| = 0

⇒ |𝐴| = 0
4 −1 2 3
The cofactor of the element – 5 in the determinant | 4 −7 −1| 𝑖𝑠
−2 −5 6
(a) 11 (b) - 12 (c) - 11 (d) 12
Correct option – (a) 11
−1 3
Explanation : 𝐶𝑜𝑓𝑎𝑐𝑡𝑜𝑟 𝑜𝑓 𝑎𝑛 𝑒𝑙𝑒𝑚𝑒𝑛𝑡 − 5 𝑖𝑠 (−1)3+2 | | = −(1 − 12) = 11
4 −1

Page 30 of 199
5 10 0
For any 2 × 2 matrix, if A (adjA) = [ ],
0 10
then value of |A| is:
(a) 20 (b) 100 (c) 10 (d) 0
Correct option – (c) 10
Explanation :𝑊𝑒 𝑘𝑛𝑜𝑤 𝑡ℎ𝑎𝑡, 𝑨 (𝒂𝒅𝒋𝑨) = |A|I

𝟏𝟎 𝟎
⇒[ ] = |A|I
𝟎 𝟏𝟎

𝟏 𝟎
⇒ 10 [ ] = |A|I
𝟎 𝟏𝟏

⇒ 10 𝐼 = |A|I

⇒ |A| = 10

6 For any n × n matrix which one of the following is false ∶


𝑎𝑑𝑗 𝐴
(𝑎) |𝑎𝑑𝑗𝐴| = |𝐴|𝑛−1 (𝑏) 𝐴 (𝑎𝑑𝑗 𝐴) = |𝐴|𝐼 (𝑐) |𝑘𝐴| = 𝑘 𝑛 |𝐴| (𝑑) 𝐴−1 = 2 |𝐴|
−1 𝑎𝑑𝑗 𝐴
Correct option – (𝑑) 𝐴 = |𝐴|2

ASSERTION – REASONING QUESTION - 1


Assertion (A) : The points (𝑎 + 5, 𝑎 − 4 ), (𝑎 − 2 , 𝑎 + 3) 𝑎𝑛𝑑 (𝑎, 𝑎)
𝑑𝑜 𝑛𝑜𝑡 𝑙𝑖𝑒𝑠 𝑜𝑛 𝑎 𝑠𝑡𝑟𝑎𝑖𝑔ℎ𝑡 𝑙𝑖𝑛𝑒 𝑓𝑜𝑟 𝑎𝑛𝑦 𝑣𝑎𝑙𝑢𝑒 𝑜𝑓 𝑎.

7
Reason(R ): 𝑇ℎ𝑒 𝑣𝑎𝑙𝑢𝑒 𝑜𝑓 𝑎𝑟𝑒𝑎 𝑜𝑓 ∆ = ≠0
2

Now pick the correct option :


(a) Both A and R are individually true and R is the correct explanation of A.
(b) Both A and R are individually true and R is NOT the correct explanation of A.
(c) A is true but R is false
(d) A is false but R is true
(e) Both A and R are false

Solution : Correct option is – option ( a) Both A and R are individually true and R is the correct
explanation of A.
1 𝑎+5 𝑎−4 1 7
∆ = |𝑎 − 2 𝑎 + 3 1| = (𝑑𝑜𝑒𝑠 𝑛𝑜𝑡 𝑐𝑜𝑛𝑡𝑎𝑖𝑛 a) ≠ 0
2 2
𝑎 𝑎 1

ASSERTION – REASONING QUESTION - 2


Assertion (A) : The system of equations, x+2y=2 and 2x+3y=3 are consistent.

Reason(R ): 𝑆𝑦𝑠𝑡𝑒𝑚 𝑜𝑓 𝑒𝑞𝑢𝑎𝑡𝑖𝑜𝑛𝑠, 𝐴𝑋 = 𝐵 𝑖𝑠 𝑐𝑜𝑛𝑠𝑖𝑠𝑡𝑒𝑛𝑡 𝑖𝑓 |A| = 0


Now pick the correct option :
(f) Both A and R are individually true and R is the correct explanation of A.
(g) Both A and R are individually true and R is NOT the correct explanation of A.
(h) A is true but R is false
(i) A is false but R is true

Page 31 of 199
(j) Both A and R are false

Solution : Correct option is – option ( h) A is true but R is false

Explanation : 𝑆𝑦𝑠𝑡𝑒𝑚 𝑜𝑓 𝑒𝑞𝑢𝑎𝑡𝑖𝑜𝑛𝑠, 𝐴𝑋 = 𝐵 𝑖𝑠 𝑐𝑜𝑛𝑠𝑖𝑠𝑡𝑒𝑛𝑡 𝑖𝑓 |A| ≠ 0


1 2
Here |A| = | | = 3 − 4 = −1 ≠ 0 hence system of equations is consistent.
2 3

02 MARKS QUESTIONS
1 4 −1
𝐹𝑖𝑛𝑑 , |𝑎𝑑𝑗(𝑎𝑑𝑗𝐴)|, 𝑖𝑓 𝐴 = [ ]
3 2
4 −1
Solution : 𝐴 = | |
3 2
2 1
𝑡ℎ𝑒𝑛 𝑎𝑑𝑗 𝐴 = [ ]
−3 4

4 −1
𝑡ℎ𝑒𝑛 𝑎𝑑𝑗(𝑎𝑑𝑗 𝐴) = [ ]
3 2

4 −1
𝑁𝑜𝑤, |𝑎𝑑𝑗(𝑎𝑑𝑗 𝐴)| = | | = 8 + 3 = 11
3 2

2 If, 𝐴2 − 𝐴 + 𝐼 = 0, 𝑡ℎ𝑒𝑛 𝑓𝑖𝑛𝑑 𝐴−1 .

𝑆𝑜𝑙𝑢𝑡𝑖𝑜𝑛 ∶ 𝑤𝑒 ℎ𝑎𝑣𝑒 𝐴2 − 𝐴 + 𝐼 = 0,

⇒ 𝐴−1 (𝐴2 − 𝐴 + 𝐼) = 𝐴−1 0

⇒ 𝐴 − 𝐼 + 𝐴−1 = 0

⇒ 𝐴−1 = 𝐼 − 𝐴
3 𝑃 2
𝐼𝑓, 𝐴 = [ ] 𝑎𝑛𝑑 |𝐴3 | = 125, 𝑡ℎ𝑒𝑛 𝑓𝑖𝑛𝑑 𝑡ℎ𝑒 𝑣𝑎𝑙𝑢𝑒 𝑜𝑓 𝑝.
2 𝑃

𝑃 2
Solution : |𝐴| = | | = 𝑝2 − 4
2 𝑃
𝑁𝑜𝑤 𝑤𝑒 ℎ𝑎𝑣𝑒 |𝐴3 | = 125
⇒ |𝐴|3 = 125
⇒ ( 𝑝2 − 4)3 = 125
⇒ 𝑝2 − 4 = 5
⇒ 𝑝2 = 9
⇒ 𝑝 = ±3

4 𝐹𝑖𝑛𝑑 𝑡ℎ𝑒 𝑣𝑎𝑙𝑢𝑒 𝑜𝑓 𝛼, 𝑖𝑓 𝑎𝑟𝑒𝑎 𝑜𝑓 𝑡𝑟𝑖𝑎𝑛𝑔𝑙𝑒 𝑖𝑠 4 𝑠𝑞 𝑢𝑛𝑖𝑡𝑠, 𝑎𝑛𝑑 𝑤ℎ𝑜𝑠𝑒 𝑣𝑒𝑟𝑡𝑖𝑐𝑒𝑠 𝑎𝑟𝑒
( −2,0), (0,4) 𝑎𝑛𝑑 (0, 𝛼 ).
1 −2 0 1
𝑆𝑜𝑙𝑢𝑡𝑖𝑜𝑛 ∶ | 0 4 1| = ±4
2
0 𝛼 1
1
⇒ [ −2 (4 − 𝛼) + 0 + 1(0) = ±4
2
⇒ −8 + 2𝛼 = ±8
Taking minus sign
−8 + 2𝛼 = − 8 ⇒ 𝛼 = 0
Taking positive sign

Page 32 of 199
−8 + 2𝛼 = 8 ⇒ 𝛼 = 8
𝐻𝑒𝑛𝑐𝑒 , 𝛼 = 0 𝑜𝑟 8
5 2 3
𝐼𝑓, 𝐴 = [ ] , 𝑡ℎ𝑒𝑛 𝑤𝑟𝑖𝑡𝑒 𝐴−1 𝑖𝑛 𝑡𝑒𝑟𝑚𝑠 𝑜𝑓 𝐴.
5 −2
2 3
Solution : , 𝐴 = [ ]
5 −2

−2 −3
⇒ 𝑎𝑑𝑗𝐴 = [ ]
−5 2

2 3
𝑎𝑛𝑑 |𝐴| = | | = −4 − 15 = −19
5 −2

𝑎𝑑𝑗 𝐴
𝑁𝑜𝑤 𝐴−1 =
|𝐴|

1 −2 −3
= − [ ]
19 −5 2

1 2 3
= [ ]
19 5 −2

1
= 𝐴
19

03 MARKS QUESTIONS
1 𝑆𝑜𝑙𝑣𝑒 𝑡ℎ𝑒 𝑒𝑞𝑢𝑎𝑡𝑖𝑜𝑛𝑠, 5𝑥 − 7𝑦 − 2 = 0 𝑎𝑛𝑑 7𝑥 − 5𝑦 − 3 = 0 𝑏𝑦 𝑚𝑎𝑡𝑟𝑖𝑥 𝑚𝑒𝑡ℎ𝑜𝑑.
Solution : We have 5𝑥 − 7𝑦 − 2 = 0 𝑎𝑛𝑑 7𝑥 − 5𝑦 − 3
5 −7 𝑥 2
⇒[ ] [ ] = [ ] … . (𝑖)
7 −5 𝑦 3

5 −7 𝑥 2
𝑁𝑜𝑤 𝑐𝑜𝑛𝑠𝑖𝑑𝑒𝑟𝑖𝑛𝑔 𝐴 = [ ] , 𝑋 = [𝑦] 𝑎𝑛𝑑 𝐵 = [ ]
7 −5 3
The equation (i) becomes AX = B

𝑤𝑒 𝑐𝑎𝑛 𝑤𝑟𝑖𝑡𝑒 , 𝑋 = 𝐴−1 𝐵

5 −7
𝐻𝑒𝑟𝑒 |𝐴| = | | = −25 + 49 = 24 ≠ 0, 𝑠𝑜 𝐴−1 𝑒𝑥𝑖𝑠𝑡𝑠.
7 −5

Now we have to calculate cofactors

𝐴11 = +(−5) = −5

𝐴12 = −(7) = −7

𝐴21 = −(−7) = 7

𝐴22 = +5 = 5

−5 7
𝑇ℎ𝑒𝑟𝑒𝑓𝑜𝑟𝑒 , 𝑎𝑑𝑗𝐴 = [ ]
−7 5

Page 33 of 199
𝑎𝑑𝑗 𝐴
𝐴−1 =
|𝐴|

1 −5 7
= [ ]
24 −7 5

𝑁𝑜𝑤 𝑤𝑒 ℎ𝑎𝑣𝑒, 𝑋 = 𝐴−1 𝐵

𝑥 1 −5 7 2
⇒ [𝑦 ] = [ ][ ]
24 −7 5 3

𝑥 1 −10 + 21
⇒ [𝑦 ] = [ ]
24 −14 + 15

𝑥 1 11
⇒ [𝑦 ] = [ ]
24 1

11
𝑥
⇒ [𝑦] = [24]
1
24

11 1
𝐻𝑒𝑛𝑐𝑒 , 𝑥 = 𝑎𝑛𝑑 𝑦 =
24 24

2 𝐹𝑖𝑛𝑑 𝑎 𝑚𝑎𝑡𝑟𝑖𝑥 𝑃 𝑜𝑓 𝑡ℎ𝑒 𝑜𝑟𝑑𝑒𝑟 2 × 2 𝑜𝑓 𝑡ℎ𝑒 𝑓𝑜𝑙𝑙𝑜𝑤𝑖𝑛𝑔 𝑒𝑞𝑢𝑎𝑡𝑖𝑜𝑛 ∶

1 −2 6 0
𝑃[ ]= [ ]
1 4 0 6

1 −2 1 0
Solution : Let 𝑄 = [ ], then 𝑃𝑄 = 6 [ ]
1 4 0 1

⇒ 𝑃𝑄 = 6𝐼

⇒ 𝑃 = 6𝑄 −1 … . . (𝑖)

1 −2
𝑁𝑜𝑤, |𝑄| = | | = 4 + 2 = 6 ≠ 0, 𝑠𝑜 𝑄 −1 𝑒𝑥𝑖𝑠𝑡𝑠.
1 4
𝑎𝑑𝑗 𝑄
𝑄 −1 =
|𝑄|
4 2
𝑁𝑜𝑤, 𝑎𝑑𝑗 𝑄 = [ ]
−1 1
1 4 2
𝑓𝑟𝑜𝑚 𝑒𝑞𝑢𝑎𝑡𝑖𝑜𝑛 (𝑖), 𝑃 = 6𝑄 −1 = 6 × [ ]
6 −1 1
4 2
𝐻𝑒𝑛𝑐𝑒 , 𝑃 = [ ]
−1 1

3 𝐺𝑖𝑣𝑒𝑛 𝑡ℎ𝑎𝑡 𝐴 𝑖𝑠 𝑎 𝑠𝑞𝑢𝑎𝑟𝑒 𝑚𝑎𝑡𝑟𝑖𝑥 𝑜𝑓 𝑜𝑟𝑑𝑒𝑟 3 𝑎𝑛𝑑 |𝐴| = −2 , 𝑡ℎ𝑒𝑛 𝑓𝑖𝑛𝑑 𝑣𝑎𝑙𝑢𝑒 𝑜𝑓 |𝑎𝑑𝑗(2𝐴)|

Solution : |𝑎𝑑𝑗(2𝐴)| = |(2𝐴)|2 [ 𝑎𝑑𝑗𝐴 = |𝐴|𝑛−1 ]


= (23 |𝐴|)2 [ |𝑘𝐴| = 𝑘 𝑛 |𝐴|]
= 26 |𝐴|2

Page 34 of 199
= 26 × (−2)2 = 28
4 Given that A is a non-singular matrix of order 3 such that 𝐴2 = 2A, then find the value of |2A|.
Solution :
𝐺𝑖𝑣𝑒𝑛 𝑡ℎ𝑎𝑡 𝐴2 = 2𝐴
⇒ |𝐴2 | = |2𝐴|
⇒ |𝐴|2 =23 |𝐴| as |𝑘𝐴| = 𝑘 𝑛 |𝐴| 𝑓𝑜𝑟 𝑎 square 𝑚𝑎𝑡𝑟𝑖𝑥 𝑜𝑓 order 𝑛
⇒ either |𝐴| = 0 𝑜𝑟 |𝐴| = 8 But A is non-singular matrix
∴|𝐴| = 8 hence |2𝐴| = 82 = 64
5 3 −4 𝑥 10
𝑆𝑜𝑙𝑣𝑒 [ ] [ ] = [ ] , 𝑓𝑜𝑟 𝑥 𝑎𝑛𝑑 𝑦.
9 2 𝑦 2
3 −4 𝑥 10
Solution : [ ][ ] = [ ]
9 2 𝑦 2

𝑥 3 −4 −1 10
⇒[𝑦] = [ ] [ ]
9 2 2
𝑥 1 2 4 10
⇒[𝑦] = 6+36 [ ][ ]
−9 3 2
𝑥 1 20 + 8
⇒[𝑦] = 42 [ ]
−90 + 6
28 84
⇒𝑥 = 𝑎𝑛𝑑 𝑦 = −
42 42

2 84
⇒𝑥 = 𝑎𝑛𝑑 𝑦 = −
3 42

05 MARKS QUESTIONS
1
−4 4 4 1 −1 1
Determine the product of [−7 1 3 ] 𝑎𝑛𝑑 [1 −2 −2],
5 −3 −1 2 1 3

And then solve the system of equations


𝑥−𝑦+𝑧 =4
𝑥 − 2𝑦 − 2𝑧 = 9
2𝑥 + 𝑦 + 3𝑧 = 1

−4 4 4 1 −1 1
Solution : 𝐿𝑒𝑡 𝐴 = [−7 1 3 ] 𝑎𝑛𝑑 𝐵 = [1 −2 −2]
5 −3 −1 2 1 3

−4 4 4 1 −1 1
𝑡ℎ𝑒𝑛 𝐴𝐵 = [−7 1 3 ] [1 −2 −2]
5 −3 −1 2 1 3

−4 + 4 + 8 4−8+4 −4 − 8 + 12
= [−7 + 1 + 6 7−2+3 7−2+9 ]
5−3−2 −5 + 6 − 1 5+6−3

8 0 0
= [0 8 0]
0 0 8

Page 35 of 199
⇒ 𝐴𝐵 = 8 𝐼

−1
𝐴 1 4 4 4
⇒ 𝐵 = = [−7 1 3 ] … … . . 𝑃𝑎𝑟𝑡(𝑖) 𝑜𝑓 𝑠𝑜𝑙𝑢𝑡𝑖𝑜𝑛.
8 8
5 −3 −1

Now the given system of equations are


𝑥−𝑦+𝑧 =4
𝑥 − 2𝑦 − 2𝑧 = 9
2𝑥 + 𝑦 + 3𝑧 = 1

⇒ 𝐵𝑋 = 𝐶

1 −1 1 𝑥 4
𝑤ℎ𝑒𝑟𝑒 𝐵 = [1 −2 −2] , 𝑋 = [𝑦] 𝑎𝑛𝑑 𝐶 = [9]
2 1 3 𝑧 1

𝑁𝑜𝑤 𝑋 = 𝐵 −1 𝐶
1 4 4 4 4
⇒ 𝑋 = [−7 1 3 ] [9]
8
5 −3 −1 1
𝑥 3
[𝑦] = [−2]
𝑧 −1

On comparing the corresponding elements, we get

𝑥 = 3, 𝑦 = −2, 𝑧 = −1
2
Use matrix method to solve the following system of equations:
2 3 10
+ + =4
𝑥 𝑦 𝑧

4 6 5
− + =1
𝑥 𝑦 𝑧

6 9 20
+ − = 2, 𝑥, 𝑦, 𝑧 ≠ 0
𝑥 𝑦 𝑧

Solution: The given system of equations are

2 3 10
+ + =4
𝑥 𝑦 𝑧

4 6 5
− + =1
𝑥 𝑦 𝑧

6 9 20
+ − =2
𝑥 𝑦 𝑧

Page 36 of 199
1⁄
2 3 10 𝑥 4
⇒ [4 −6 1
5 ] [ ⁄𝑦] = [1] … . . (𝑖)
6 9 −20 1⁄ 2
𝑧

1⁄
2 3 10 𝑥 4
𝐶𝑜𝑛𝑠𝑖𝑑𝑒𝑟, 𝐴 = [4 −6 1
5 ] , 𝑋 = [ ⁄𝑦] , 𝐵 = [1]
6 9 −20 1⁄ 2
𝑧

𝑠𝑜 𝑒𝑞𝑢𝑎𝑡𝑖𝑜𝑛 (𝑖)𝑏𝑒𝑐𝑜𝑚𝑒𝑠

𝐴𝑋 = 𝐵

⇒ 𝑋 = 𝐴−1 𝐵 … . . (𝑖𝑖)

2 3 10
𝑁𝑜𝑤, |𝐴| = |4 −6 5 |
6 9 −20

= 2 (120-45)-3(-80-30)+10(36+36)= 150+330+720=1200 ≠ 0, 𝑠𝑜 𝐴−1 𝑒𝑥𝑖𝑠𝑡𝑠

Cofactors of an element of determinant A are :

𝐴11 = 75, 𝐴12 = 110, 𝐴13 = 72

𝐴21 = 150, 𝐴22 = −100, 𝐴23 = 0

𝐴31 = 75, 𝐴32 = 30, 𝐴33 = −24

𝐴11 𝐴21 𝐴31


𝑁𝑜𝑤, 𝑎𝑑𝑗 𝐴 = [𝐴12 𝐴22 𝐴32 ]
𝐴13 𝐴23 𝐴33

75 150 75
= [110 −100 30 ]
72 0 −24
𝑎𝑑𝑗 𝐴
𝑁𝑜𝑤, 𝐴−1 =
|𝐴|
𝑎𝑛𝑑, 𝑋 = 𝐴−1 𝐵
1⁄
𝑥 75 150 75 4
1 1
⇒ [ ⁄𝑦 ] = 1200 [110 −100 30 ] [1]
1⁄ 72 0 −24 2
𝑧

1⁄ 1⁄
𝑥 600 2
1 1 1
⇒ [ ⁄𝑦 ] = 1200 [400] = ⁄3
1⁄ 240 1
𝑧 [ ⁄5]

1 1 1 1 1 1
⇒𝑥= , = 3, 𝑧 =
2 𝑦 5

Page 37 of 199
⇒ 𝑥 = 2, 𝑦 = 3, 𝑧 = 5

CASE STUDY 1
Three friends, Ravi, Sanjay and Ramesh came to Delhi to visit Qutab – Minar. They are carrying a
number of visiting cards but they do not want to reveal the actual number of it. They start playing with
each other in the garden nearby the monument. Total number of visiting cards they have is 18. Twice
the number of visiting cards Ramesh has when added to Ravi’s number of cards gives 21.On adding
Sanjay’s and Ramesh’s number of visiting cards to thrice the Ravi’s number of cards, they give 36.
( Assume the number of cards Ravi, Sanjay and Ramesh has are x, y and z respectively)
Based on this information answer the following questions:
(A) The system of linear equations formed from the given situation , is
(a)x+y+z=18, x+2y=21, x+3y+z = 36
(b) x+y+z=18, x+2z=21,3x+y+z=36
(c) x+y+z=18, y+2z=21, x+y+z=36
(d) x+y+z=18, 2x+z=21, x+3y+3z=36

Solution : Correct option is - (b) x+y+z=18, x+2z=21,3x+y+z=36


(B) The correct matrix form AX=B of the system of linear equations formed from the given situation,
is
1 1 1 𝑥 18
(a)[2 0 1] [𝑦] = [21]
1 3 3 𝑧 36
1 1 1 𝑥 18
𝑦
(b)[1 2 0] [ ] = [21]
1 3 1 𝑧 36
1 1 1 𝑥 18
(c)[0 1 2] [𝑦] = [21]
1 1 3 𝑧 36
1 1 1 𝑥 18
𝑦
(d)[1 0 2] [ ] = [21]
3 1 1 𝑧 36
1 1 1 𝑥 18
Solution : Correct option is - (d)[1 0 2] [𝑦] = [21]
3 1 1 𝑧 36

(C )
𝑻𝒉𝒆 𝒄𝒐𝒓𝒓𝒆𝒄𝒕 𝒗𝒂𝒍𝒖𝒆 𝒐𝒇 𝐝𝐞𝐭(𝑨)𝒊𝒔 :
( a) 4 (b) -2 (c) 2 (d) -1

(
D) 𝑻𝒉𝒆 𝒄𝒐𝒇𝒂𝒄𝒕𝒐𝒓𝒔 𝒐𝒇 𝒂𝟑𝟐 𝒊𝒏 𝑨 𝒊𝒔 ∶
(a)1 (b) -2 (c) 2 (d) -1

( E)
𝑻𝒉𝒆 𝒏𝒖𝒎𝒃𝒆𝒓 𝒐𝒇 𝒗𝒊𝒔𝒊𝒕𝒊𝒏𝒈 𝒄𝒂𝒓𝒅𝒔 𝒘𝒊𝒕𝒉 𝑹𝒂𝒗𝒊, 𝑺𝒂𝒏𝒋𝒂𝒚 𝒂𝒏𝒅 𝑹𝒂𝒎𝒆𝒔𝒉
𝒓𝒆𝒔𝒑𝒆𝒄𝒕𝒊𝒗𝒆𝒍𝒚 𝒂𝒓𝒆
(a) 3,6,9 (b) 3,9,6 (c) 9,3,6 (d)9,6,3

CASE STUDY 2
A trust having a fund of Rs.30000 wishes to invest into two different types of bonds. The first bond
pays 5% interest per annum which will be given to an orphanage and the second bond pays 7%

Page 38 of 199
interest per annum which will be given to “CANCER AID SOCIETY” – an NGO. Determine how the
trust should divide Rs.30000 among two types of bonds to obtain an annual total interest of Rs.1800.
(i)Form the equations in terms of numbers of bonds.
(ii) Find values of both types of bonds.

Solution : 𝑰𝒇, 𝑹𝒔 𝒙 𝒂𝒓𝒆 𝒊𝒏𝒗𝒆𝒔𝒕𝒆𝒅 𝒊𝒏 𝒇𝒊𝒓𝒔𝒕 𝒃𝒐𝒏𝒅 𝒂𝒏𝒅 𝑹𝒔. 𝒚 𝒂𝒓𝒆 𝒊𝒏𝒗𝒆𝒔𝒕𝒆𝒅 𝒊𝒏 𝒔𝒆𝒄𝒐𝒏𝒅 𝒃𝒐𝒏𝒅
𝒕𝒉𝒆𝒏, 𝒙 + 𝒚 = 𝟑𝟎𝟎𝟎𝟎 & 5𝒙 + 𝟕𝒚 = 𝟏𝟖𝟎𝟎𝟎𝟎

1 1 𝑥 30000
⇒[ ] [𝑦 ] = [ ]
5 7 180000
𝐴𝑋 = 𝐵

⇒ 𝑋 = 𝐴−1 𝐵

1 1
| | = 7 − 5 = 2 ≠ 0, 𝑠𝑜 𝐴−1 𝑒𝑥𝑖𝑠𝑡𝑠
5 7

𝑥 1 7 −1 30000 15000
⇒ [𝑦] = [ ][ ]= [ ]
2 −5 1 180000 15000

⇒ 𝑥 = 15000, 𝑦 = 15000

Page 39 of 199
5.CONTINUITY AND DIFFERENTIABILITY
* A function is said to be continuous at x  a if
Left hand limit  Right hand limit  functional value
i.e. lim f x   lim f x   f a 
xa  xa 
i.e. lim f (a  h)  lim f (a  h)  f (a)
h0 h0
* A function is said to be differenti able at x  a if
f a  h   f a  f a  h   f a 
Lf ` a   Rf ` a  i.e. lim  lim
h0 h h0 h

dx
d 
i  x  nx n1 ,  n    n1 ,
d n 1
dx  x 
 n
dx
 
ii  d x  1 ,
2 x
x
iii  d  1   21 and d x  1, iv  d c   0, c  R
dx  x  x dx dx
d x
dx
 
v  a  a log a, e  e
x d x
dx
 
x
vi  log a x   1 , a  0, a  1, x  0
d
dx x log a
vii  log | x | , x  0
d 1
viii  sin x   cos x, x  R
d
dx x dx
ix  cos x    sin x , x  R,
d
x  d tan x   sec 2 x, x  R
dx dx
xi cot x    cos ec x, x  R
d 2
xii  d sec x   sec x tanx , x  R
dx dx
xiii  cos ecx   cos ecx cotx, x  R
d
dx
 
xiv  d sin 1 x  1 , x   1,1
dx 1  x2
 
xv d cos 1 x   1 , x   1,1
dx
 
xvi d tan 1 x  1 2 , x  R
dx 1 x
1 x 2

dx
 
xvii  d cot 1 x   1 2 , x  R
1 x

xviii  d sec 1 x 
dx
 1
| x | x2 1
xix  d
dx

cos ec1x 
1
 xx d
dx
| x |  x , x  0
|x|
| x | x2 1
xxi d kf (x)   k d f (x) xxiid
u  v   du  dv
dx dx dx dx dx
du dv
v u
d u
xxiii  d
u  v   u dv  v du xxiv    dx 2 dx
dx dx dx dx  v  v

MCQ TYPE OF QUESTIONS

Q.1 The derivative of x 2 w.r.to x 3 is


2x 2 3x 3
(a) (b) (c) (d)
3 3x 2 2x
Q.2  x 1  x 1  dy
If y  sec 1    sin 1 
 
 ,then
 is
 x 1   x 1 dx

a  1 2 b1 c 0 d  21
1 x x 1

Page 40 of 199
Q.3 d2y
If y  t 10  1 & x  t 8  1 then is
dx 2
a  5
b 166 c 1 d   1
16t 6 5t
Q.4  sin x 
 g x  then g x  is
dy
If y  x sin x and  x sin x 
dx  x 
a sin xlog x  bcos xlog x  c x log sin x  d x log cos x 
Q.5  
If f x  
sec x - 1
, find f `  is
sec x  1 3
a  3 b  3 c  2 d  2
2 2 3 3

ASSERTION & REASONING

Below are given two statements – one labeled Assertion (A) and other labeled Reason (R). Select the
correct answer to these questions from the codes (i), (ii), (iii) and (iv) as given below:
(i) Both A and R are true and R is the correct explanation of the assertion
(ii) Both A and R are true but R is not the correct explanation of the assertion
(iii) A is true, but R is false
(iv) A is false, but R is true

Q.1 

sin 2x
, if x  0
Assertion : f x    x is continuous function at x  0.

 x  2 , if x  0
Reason : lim  f x   lim  f x   f 0 and lim
sin x
1
x0 x0 x0 x

Q.2 Assertion : If y  logx x  x log x , then logy  x log logx   log x 2 .


Reason : If a  b  c then loga  logb  logc.

VERY SHORT ANSWER TYPE (VSA)

2 MARKS

Q.1 Find the interval in which f x  x 2  2x  15 is  or  .


Q.2 dy
If x 3  y 3  3axy , find .
dx
Q.3 If y  tanx , prove that y 2  2yy1
CBSE 2017.
Q.4
1  cos x
 , when x  0
Find the value of k for which f x    8x 2 is continuous at x  0
 k , when x  0
CBSE 2017.
Q.5
If y  sin x cos x find
dy
.
dx

Page 41 of 199
SHORT ANSWER TYPE (SA)

3 MARKS EACH

Q.1 Find the values of a &b so that the function f given by


 1 , if x  3

f x   ax  b , if 3  x  5. is continuous at x  3 and x  5.CBSE 2013
 7 , if x  5

Q.2  3a 2 x  x 3 
Differenti ate tan 1  3  w.r.to x. CBSE 2015
 a  3ax 2 
 
Q.3
dy sin 2 a  y 
If siny  x sina  y  , prove that  . CBSE 2009,11,12
dx sin a

Q.4 If y  3e 2x  2e 3x , prove that


d2y
 6y  0 CBSE 2007,2009
dy
 5
dx 2 dx

Q.5  1  x 2  1  x 2 
Differenti ate tan 1   w.r.to cos x CBSE 2019
1 2

 1  x 2  1  x 2 

LONG ANSWER TYPE OF QUESTIONS (5 marks each)

Q.1 Find the intervals in which f x   x  13 x  33 is increasing   or decreasing  .


CBSE2011

Q.2 1  sin 3 x 
 2
, if x 
 3 cos x 2
  π
Let f x    a , if x  . If f x  is continuous at x  , find a &b.
 2 2
 b 1  sin x  , if x 

   2 x 2 2
CBSE2008,2016

CASE BASED QUESTIONS

Q.1
Let 𝑥 = (𝑡) and 𝑦 = (𝑡) be parametric forms with as a parameter, then ,
where 𝑓′(𝑡) ≠ 0.

On the basis of above information, answer the following questions.

1(i) The derivative of 𝑓(tan 𝑥) w.r.t. , where 𝑓′(1) = 2 and

, is

Page 42 of 199
(b) -√2 (c) (d)

The derivative of 𝑒𝑥3 with respect to log 𝑥 is


1(ii)
(a) 𝑒𝑥3 (b) 3𝑥22𝑒𝑥3 (c) 3𝑥3𝑒𝑥3 (d) 3𝑥2𝑒𝑥3 + 3𝑥

1(iii) The derivative of with respect to is


(a) −1 (b) (c) (d)

1(OR iii) The derivative of cos−1(2𝑥2 − 1) w.r.t. cos−1 𝑥 is

(a) (d) 1 − 𝑥2

Q.2 If 𝑦 = (𝑢) is a differentiable function of and 𝑢 = (𝑥) is a differentiable function of , then 𝑦 =

𝑓(𝑔(𝑥)] is a differentiable function of and . This rule is also known as CHAIN


RULE.

Based on the above information, find the derivative of functions w.r.t. in the following questions.

cos x w.r.to x is

2(i) a  sin x b   sin x csin x d   sin x


x 2 x
1
x
7 x w.r.to x is
1 1 1 1
 2  x  2  x  2  x  2  x
a x 2 1 .7 x . log 7 b  2x .7 x . log 7 c x 2 1 .7 x . log 7 d  2x .7 x . log 7
2(ii)  x   x 1  x   x 1
1  cos x
w.r.to x
1  cos x
2(iii) a 1 sec 2 x b  1 c  1 sec 2 x d both (a) & (b)
2 2 1  cos x 2 2
1 1 x
sec x  cosec w.r.to x is
x2 1
a 2 b   2 c 2
d  1
2(OR iii) x2 1 x2 1 | x | x2 1 | x | x2 1

Answers
Ch. 5
Q.1(b),2(c),3(b),4(b),5(c)
MCQ
VSA Q.1-Decreasing in(-1,∞),Increasing (-∞,-1)

Page 43 of 199
x 2  ay
Q.2
ax  y 2

Q.4 1/16

Q.5 sin xcos x cos x cot x  sin x log sinx 

SA Q.1 a=3,b=-8
3a
Q.2
x  a2
2

Q.5 -1/2
LA Q.1 f x  is  in 1,3  3,   and  in  ,1   1,1
1
Q.2 a  , b  4
2

CBQ 1(i) a,1(ii) c,1(iii) b,1(OR iii) a

2(i) b,2(ii) a,2(iii) d,2(OR iii) c

Page 44 of 199
6. APPLICATION OF DERIVATIVES
Concept :-

Increasing or Decreasing function:- Let I be an open interval contained in the

domain of a real valued function f . Then f is said to be

(i) Increasing on I if in I

(ii) Strictly increasing on I if in I

(iii) Decreasing on I if in I
(iv) Strictly decreasing on I if in I

Theorem ;- Let f be continuous on and differentiable on the open interval then

(i) f is increasing in if for each

(ii) f is decreasing in if for each

(iii) f is a constant function in if for each

Theorem ;- Let f be continuous on and differentiable on the open interval then

(i) f is strictly increasing in if for each

(ii) f is decreasing in if for each

(iii) f is a constant function in if for each

** Let f be a function defined on an interval I.Then

(a) f is said to have a maximum/minimum value(extremum value ) in I ,if there exists a point c in I such that
f(c)≥f(x) OR f(c)≤f(x)for all x∈ 𝐼 respectively.

** Absolute maxima and minima

Let f be a function defined on the interval I and c∈ 𝐼.Then

(a) f(c) is absolute minimum if f(x)≥f(c) for all x∈ 𝐼

(b) f(c) is maximum or f(x)≤f(c) x∈ 𝐼.

(c) c∈ 𝐼 is called the critical point iff f`(c)=0

(d) Absolute maximum or minimum value of a continuous function f on [a,b] occurs

Page 45 of 199
at a or b or at critical points (where f`(x)=0).i.e. if c1, c2,… cn are critical points lying in
[a,b] then absolute maximum/minimum value of f={f(a),f(c1 ),f(c2),…f(cn),f(b)}

** Local maxima and minima

(a) A function f is said to have local maxima or simply a maximum value at x=a if
f(a±h)≤f(a) for sufficiently small h.

(b) A function f is said to have local minima or simply a minimum value at x=a if
f(a±h)≤f(a) for sufficiently small h.

**(c )First derivative test :A function f has a maximum value at x=a if

(i) f`(a)=0

(ii) f`(x) changes sign from +ive to –ive in the nbd of 'a',

However ,f has local minimum value at x=a if

(i) f`(a)=0

(ii) f`(x) changes sign from -ive to +ive in the nbd of 'a'.

If f`(a)=0 and f`(x) does not change sign ,then f(x) has neither max nor mini and the
point a is called point of inflexion.

The points where f`(x)=0 are called stationary or critical points .The stationary points
at which the function attains either maximum or minimum values are called extreme
points.

** Second derivative test

(i) A function has a maxima at x=a if f`(x)=0 and f``(a)<0

(ii) A function has a minima at x=a if f`(x)=0 and f``(a)>0


MCQ TYPE

Q.1 If the radius of a circle is increasing @ of 1 2 cm/sec, then the rate


increase of its circumfere nce is
a  2cm / sec b cm/min c   cm/sec d   2cm / sec
Q.2 1
The function g(x)  x  , x ≠ 0 decreases in the closed interval
x
a  - 1.1 b- 2,2 c- 1,1 d - 2,2
Q.3 The sides of an equilatera l triangle are increasing @ of 2 cm/sec. CBSE 2015
The rate at which t he area increases , when the side is 20 cm, is
a 20 3 cm 2 / sec b10 3 cm 2 /sec c 40 3cm 2 / sec d 30 3 cm 2 /sec

Page 46 of 199
Q.4 The function f x   x 3 - 3x 2  4x is
a strictly ↑ in R bstrictly ↓ in R cneither ↑ nor ↓ in in R d None
Q.5 The maximum value of f x   sin x  cos x is
a  2  1 b  2 1 c  2 d 2 2
Assertion & Reasoning

Below are given two statements – one labeled Assertion (A) and other labeled Reason (R). Select the
correct answer to these questions from the codes (i), (ii), (iii) and (iv) as given below:
(i)Both A and R are true and R is the correct explanation of the assertion
(ii) Both A and R are true but R is not the correct explanation of the assertion
(iii) A is true, but R is false
(iv) A is false, but R is true

Q.1
Assertion : The least valu e of f x   x 
1
, when x  0 is 2.
x
Reason : To have least valu e of f x , f `x   0 & f ``x   0.
Q.2 Assertion : f x   tanx - x is increasing in its domain.
Reason : f `x  ≥ 0.
Very short answer type of answers (VSA) 2 marks each

Q.1 Find the intervals in which f x   x 2 - 2x  15 is ↑ or ↓ .


Q.2 The amount of pollut ion conten t added in air in a city due t o x diesel vehicles is
given by P x   0.005 x 3  0.02x 2  30x  5000 Find the m arg inal incre ase in pol lution
content wh en 3diesel veh icles are added and write whic h value is indicated in the
above que stions. CBSE 2018
Q.3 Find the point of inflexion s for the function f x   x 3 - 6x 2  12x - 8.
Q.4 Find the max & mini value of f x   2x 3 - 24x  107 in the interval 1,3.
Q.5 The volume of a sphere is increasing at 3 cm 3 / sec .Find the rate of
increase of the radius , when the radius is 2 cm.
Short answer type of questions (SA) 3 marks each

Q.1 A ladder 5 m long is leaning against a wall .The bottom of the ladder is pulled along
the ground away from the wall @ of 2 cm/sec.How fast its height on the wall
decrea sin g (↓) when the foot of the ladder is 4m away from the wall ?CBSE2015,19
Q.2 Find the points of local maxima or minima , if any , of the following function.
Also find local maximum or minimum value , as the case may be :

f x   sin x  cos x, where 0  x  CBSE 2015
2
Q.3  
Pr ove f x   - x is an increasing function ↑ in 0, .CBSE 2011
4 sin x
2  cos x  2
Q.4 Amongest all open from the top  right circular cylindrica l boxes of volume 125 cm 3 ,
find the dimensions of the box which has the least surface area. CBSE 2020
Q.5 Find the intervals in which t he f x   x 3 - 6x 2  9x  15 is ↑ or ↓ .CBSE 2000,04
Long answer type of questions (LA) 5 marks each

Page 47 of 199
Q.1 Pr ove that the radius of the right circular cylinder of greatest curved surface which
can be inscribed in a given cone is half of that of the cone. CBSE 2010,12,13,20
Q.2 A tank wit h rectangula r base and rectangula r sides , open at the top is be constructe d
so that its depth is 2m and volume is 8 m 3 .If building of tank cost Rs 70 per square
meter for the base and Rs 45 per square meter for sides , what is the cost of least
expensive tank? CBSE 2009,2019
CASE BASED QUESTIONS

Q.1  x  x 
A manufactur er can sell x items at a price of ₹   (5 -   each.The cost price is ₹   500 .
 100   5 
Based on the above information, answer the following questions.
Show steps to support your answers. (CBSE 2009)
Q1i  The profit function is given by 1

a x x +500    5  x  bx 5 - x  c x 5  x    x +500  d  x x +500 


5   100   100   100   5  5 
Q.1ii  The number of items sold to earn maximum profit is 1
a 120 b50 c240 d 100
Q.1iii  The maximum profit earned is 2
a  240 b 76 c 624 d  79
OR
2
iii OR The value of d P2 is
dx
1
a  b  1
c 24 d   24
50 50 5 5

Q.2 A telephon e company in a town has 500 subscriber s on its list and collectsfi xed charges of rupess
300 per subscriber .The company proposes to increase the annual subscripti on and it is believed that
every increase of rupees 1 one subscriber will discontinu e the service.
Based upon above informatio n answer the following questions
Q.2i  The revnue function R x  is
a 500  x 300  x  b 500  x 300  x  c500  x 300  x  d 500  x 300  x 
Q 2ii  If revenue R(x) is maximum then x is
a  200 b100 c400 d None
Q 2iii  The maximum revenue R x  collected is in rupees 
a  80000 b90000 c160000 d 150000
OR
Q.2OR iii  If rupees 50 is increased , then under above condition the revenue collected i s
a 157,500 b 160000 c167,500 d 150,000
Answers Chapter 6 (Application of derivatives )

MCQ 1(b),2(a),3(a),4(a),5(c)

A.R. 1(iii),2(i)

Page 48 of 199
VSA Q.1  ,1  ,1
Q.2 30.02
Q.3 x2
Q.4 mini  75, max 89
3
Q.5
16
SA 1 8
3
2 2
4 r  5, h  5
5 1,0  2,  
LA (2) ₹1000

CBQ 1(i) c
1(ii) 240
1(iii) b
1(OR iii) a
2(i) b ,2(ii) b, 2(iii) c, 2(OR iii) a

Page 49 of 199
7. INTEGRALS
Key Concepts
Integration is the reverse process of differentiation

Page 50 of 199
Page 51 of 199
d
(C) Integration byparts  f ( x) g ( x)dx  f ( x)  g ( x)dx   [ f ( x)  g ( x)dx]dx (Here f(x) is considered
dx
as first function and g(x)is considered as second function)
Note:
(1) We can use the order ILATE for sequencing the first function and second function, where
I=Inverse Trigonometric functions, L = Logarithmic functions, A= Algebraic functions,
T = Trigonometric functions, E= Exponential functions

(2) If the Integrant contains only one function, we take that function as the first function

and 1as the second function.

SECTION -A (Level-I)
Q1. Evaluate: dx

1  cot x
Q2. Evaluate  dx
x  log sin x

Q3. dx is equal to……

Q4. dx is equal to………

Q5 = 8, Then the value of a is….

Q6. , x 2 is equal to……  2

Q7. Evaluate : .

Page 52 of 199

2

 sin
7
Q8. Write the value of xdx

2

Q9.Write the valueof sec x(sec x tanx)dx


2 x tan 1 x 2
Q10.Integrate:  dx
1 x4


e cos x
Q11.Find the Integral:  cos x dx
0
e  e cos x

3
Q12.Evaluate:  [ x]dx
0

1
Q13.The value of  x dx  .......
1

ax
Q14. Evaluate:
 ax
dx

1 x 
1

Q15.Evaluate:
 log 
0
x 
dx

1 x2  1

 cos  dx 1
2 
Q16. Find the Integral for 0  1  x 

 /3
1
Q17.Evaluate:

 1/6 tan x
dx

Q18.Evaluate : ∫ 𝐞𝟐𝐱𝐬𝐢𝐧𝐱𝐝𝐱

 5x x 5  1dx
4
Q19. Evaluate:
1

 xe
x2
Q20.Evaluate: dx
0

ASSERTION & REASONING:-

DIRECTION: IN THE FOLLOWING QUESTIONS, A STATEMENT OF ASSERTION (A) IS FOLLOWED BY A


STATEMENT OF REASON (R). MARK THE CORRECT CHOICE AS:

Page 53 of 199
1 1 x 1
x 2
 2x  3
dx 
2
tan 1
2
c
Q1. Assertion (A):

1 1 x
x 2
a 2
dx  tan 1
a a
c
Reason (B) :

A. Both A and R are true and R is the correct explanation of A

B. Both A and R are true but R is NOT the correct explanation of A

C. A is true but R is false.

D. A is false and R is True.

x ( x  log x)dx  x x  c
x

Q2. Assertion (A):

d x
x  x x (1  log x)
Reason (B) : dx

A. Both A and R are true and R is the correct explanation of A

B. Both A and R are true but R is NOT the correct explanation of A

C. A is true but R is false.

D. A is false and R is True

e (sin x. cos x)dx  e x sin x  c


x

Q3. Assertion (A):

e ( f ( x)  f ' ( x)dx  e x f ( x)  c
x

Reason (B) :

A. Both A and R are true and R is the correct explanation of A

B. Both A and R are true but R is NOT the correct explanation of A

C. A is true but R is false.


D. A is false and R is True

Page 54 of 199
SECTION -B (Level-II)
x3  x 2  x 1
Q1.Evaluate:  dx
x 1

Q2.Evaluate:  (cos 4 x  sin 4 x)dx

e 5 log x  e 4 log x
Q3. Find the integral :  dx
e 3 log x  e 2 log x

sin 3 x  cos 3 x
Q4. Evaluate :  dx
sin 2 x cos 2 x

sec 2 x
Q5
 cos ec 2 xdx .

tan 4 x sec 2 x
Q6.
 x
dx

e2x  1
Q7. Evaluate :  dx
e2x  1

( x  1)( x  log x) 2
Q8. Evaluate:  x
dx

Q9. Find the integral

Q10. Evaluate:

Q11.Evaluate

 /2
 3  5 sin x 
Q12. Evaluate:  log  3  5 cos x dx
0 0

Q13. Evaluate:  cos 3 x.e log sin x dx

cos 2 x  2 sin 2 x
Q14. Evaluate:  dx tan x  c
cos 2 x

 2x  1 
1

 tan
1
 2 
dx
Q15.Find the definite integral 0 1 x  x 

Page 55 of 199
SECTION - (Level-III)
1 x
 tan
1
Q1. Find the integral of dx
1 x

Q2. Evaluate

Q3.Integrate:  tan x  cot x dx

Q4.Integrate:  cos 2 x cos 4 x cos 6 xdx

Q5.Integrate:  x 2  4 x  3dx

Q6. Integrate:  ( x  3) / 5  4 x  x 2 dx

 /2 cos x
Q7.Evaluate:  dx
0 (1  sin x)( 2  sin x)

Q8.Evaluate: 
3/ 2
x sin x dx
1

 /4 1
Q9.Evaluate:  3
dx
0
cos x 2 sin 2 x

 x
Q10.Evaluate:  dx
0 a cos x  b 2 sin 2 x
2 2

 /4 sin x  cos x
Q11.Evaluate:  dx
0 9  16 sin 2 x

Q12.Integrate:   x  1  x  2  x  3 dx
4

SECTION -D (Level-IV)
1
Q 1.Integrate:  cos( x  a) cos( x  b)dx
x3
Q2. Integrate:  dx
x  2x  5
2

5x  3
Q3.Integrate:  dx
x 2  4 x  10

Page 56 of 199
1 x2
Q4.Evaluate:  x(1  2 x)dx
sin x cos x
Q5. . Evaluate:  sin 4
x  cos 4 x
dx

( x  3)e x
Q6.Evaluate:  ( x  1) 3
dx

( x 2  1)( x 2  2)
Q7. Integrate:  ( x 2  3)( x 2  4)dx
 /4 
Q8. Prove that 0
log( 1  tan x)dx 
8
log 2



Q9. Prove that 0
2 log sin xdx = 
2
log 2


x tan x
Q10. Evaluate:  tan x  sec x dx
0

 /2
Q11. Evaluate:  (2 log sin x  log sin 2x)dx
0

 /4
1
Q12.Find definite integral : 
0 cos 3 x 2 sin 2 x
dx

1

Q13.Integrate: log log x 
(log x) 2
dx


Q14. Sketch the graph of y  x  3 and evaluate x  3 dx
6

1
 sin3 x . sin(x  a )
dx
Q15.Evaluate:

*Case Study Questions:

Q 1.Read the following text and answer the following questions on the basis of the same:

Page 57 of 199
1

x
99
(i) Find the value of dx
1


(ii) Find the value of  x cos xdx

 /2

 sin
3
(iii) Find the value of xdx
 /2

 tan x.sec x dx
2
(iv) Find the value of


(v) Find the value of  xsin xdx


Q2.The given integral ∫ f(x) dx can be transformed into another form by changing the independent
variable x to t by substituting x=g(t)

Consider I = ∫ f(x) dx
Put x = g(t) so that dx/dt = g′(t)
We write dx = g′ (t)dt
Thus I = ∫ f(x) dx= ∫ f(g(t)) g′(t) dt

This change of variable formula is one of the important tools available to us in the name of integration by
substitution.

Page 58 of 199
sin(tan 1 )
1
(i)  1  x 2 dx
1 is ……..
(ii) ∫ tanx dx is equal to =……..

(iii) ∫ sin (ax+b) cos(ax+b) dx is ………….

1
(iv)  x  x log x dx
is equal to …………...

ANSWERS
SECTION-A:-
2x 1 
1.  cos ecx  c 2. log( x  log sin x)  c 3. 4. c 4. e  x .  c 5. a = 2, 6. -2, 7. 8. 0,
log 2 , x , 2

9. sec x tanx+c 10.



tan 1 x 2  2

 c 11.  12. 3, 13. 1 14. log sec x  c 15. 0


2 ,
1
e 2 x sin( x  tan 1 )
  2  c 19. 4 2 (e  1)
16.  log 2 17. 18. 20.
2 12 5 3 2

SECTION-B :-
x3 sin 2 x x3 2
1.  x  c 2. c 3.  c 4. sec x  cos ecx  c 5. tan x  x  c 6. tan 5 x  c
3 2 3 5

7. log( e x  e  x )  c 8.
x  log x 3  c 9. 64/231 10.

11. 16/3 12.0 13.
3 2
 cos 4 x  c 14. tan x  c 15. 0,

SECTION-C :-

1.
1
2
x cos 1 x  1  x 2  c  1
3
2
3
x
2
 tan x  1 
2.  tan  x  tan 1  c 3. 2 tan 1 
 2 tan x 
  c

11 1 1  ( x  2) 2 1
4.  sin 12 x  x  sin 8 x  sin 4 x   c 5. x  4 x  3  log x  x 2  4 x  3  c 6.
4 12 8 4  2 2
x2 4 3 1 10 2 2
 5  4 x  x 2  sin 1  c 7. log 8.  9. 10.
3 3  2 9 2ab

1 19
11. log 9 12. ,
40 2

SECTION-D :-

Page 59 of 199
1 cos( x  a) 1 2 x 1  6
1. log c 2. log x 2 2 x  5  log c
sin( a  b) cos( x  b) 2 6 x 1  6
x 3
3. 5 x 2  4 x  10  7 log x  2  x 2 4 x  10  c 4.  log x  log 1  2 x  c
2 4

 1 1 1 2 x x
5. tan (cos 2 x)  c 6. ex  c 7. x  tan 1  3 tan 1  c 8.proof 9. proof
4 ( x  1) 2
3 3 2
  1 10 2  1 
10. (  2) 11. 12. 13. x log log x  c
log x 
log
2 2 2 9 

2 sin( x   )
14. 9 15. c
sin  sin x

Page 60 of 199
8. APPLICATIONS OF INTEGRALS
(A) Multiple Choice Questions

1 The area bounded by the curves y2 = 4x and y = x is equal to:


1/3 b. 8/3 c. 35/6 d. None of these
2 The area of the region bounded by the curve x² = 4y and the straight line x = 4y – 2 is:
⅜ sq. units b. ⅝ sq. units c. ⅞ sq. units d. 9/8 sq. units
3 The area enclosed between the graph of y = x and the lines x = 0, y = 1, y = 8 is
3

7 b. 14 c. 45/4 d. None of these


4 Area bounded by the line x = 2y + 3, the y-axis and between y = -1 and y = 1 is
6 sq. units b. 4 sq. units c.8 sq. units d. 3/2 sq. units
5 The area of the region bounded by the circle x² + y² = 16 is
2π sq. units b. π sq. units c. 4π sq. units d. 16π sq. units
6 Area bounded by the curve y = sin x and the x-axis between x = 0 and x = 2π is
2 sq. units b. 3 sq. units c. 4 sq. units d. None of these

ANSWERS:- (1) b (2) d (3) c (4) a (5) d (6) c

(B) Assertion and Reasoning Questions:


In the following questions two statements Assertion (A) and reason (R) are given. Read the statements
carefully and answer as instructed:

(i) Option A, if Both A and R are true and R be a correct explanation of A.


(ii) Option B, if Both A and R are true but R does not a correct explanation of A.
(iii) Option C, if A is true but R is false.
(iv) Option D, if A is false and R is true
1.
Assertion(A): The area bounded by 𝒚𝟐 = 𝟒𝒙 and y = x is 8/3 sq. units.
𝟖𝒂𝟐
Reason(R): The area bounded by 𝒚𝟐 = 4ax and y = mx is 𝟑𝒎𝟑 sq. units.

2. Assertion (A): The area bounded by circle 𝑥 2 + 𝑦 2 = 𝑎2 in first quadrant is ∫a √𝑎2 − 𝑥 2 dx


0
a
Reason (R): the same area can be found by ∫0 √𝑎2 − 𝑦 2 dy

3 Assertion ( A ) : Area bounded by the curve y = cosx in first quadrant is 1 square unit.
𝜋⁄
Reasoning ( R ) : ∫0 2 cosxdx =1

Answers: (1) A (2) B (3) A

(B)Very Short Answer Type Questions (2 Marks)

1. Find the area under the parabola y2 = 4ax, above X-axis, from x = 0 to x = 4a
Sol. Equation of the given curves is :
y2 = 4ax
 Y = 2√𝑎𝑥

Page 61 of 199
∴ the required area under the curve from x= 0 and x = 4a
𝑥2
𝐴 = ∫𝑥1 2√𝑎𝑥dx
4𝑎
𝐴 = ∫0 2√𝑎𝑥dx
2 3 4a
= [2 √ax 2 ]
3 0
32 2
= 3 a units
2. Find the area bounded by the line y = x, x-axis and from x = – 1 to x = 2.
Sol. We have, y = x, a line
Required Area = Area of shaded region
0 2
0 2 x2 x2
= |∫−1 x dx| + |∫0 x dx| = | | + | |
2 −1 2 0
1 2 1 5
= |− 2| + |1| = 2 + 2 = 2
sq. units

3. Find the area of the curve y = sin x between 0 and π.


Solution: Given equation of curve is y = sin x

Area of OAB

= – [cos π – cos 0]
= -(-1 -1)
= 2 sq. units

Short Answer Type Questions ( 3 / 5 Marks)

1. Find the area of the region enclosed by the parabola x2 = y, the line y = x + 2 and the x-axis,
Sol. From the given equation
x2 = y and y = x + 2
 x2 = x + 2
 x2 - x – 2 = 0
 (x-2)(x+1) = 0
 x = 2, x = -1
For the parabola with vertex (0,0) and the axis of parabola is y-axis
A O B C
X -1 0 1 2
Y 1 0 1 4
For the line y = x+2
A D E C
X -1 0 1 2
Y 1 2 3 4
2 2
So the Required area = ∫−1(x + 2)dx - ∫−1 x 2 dx

Page 62 of 199
(x+2)2 2 x3
2
=[ ] - [2]
2 −1 −1
1 8 1 15 9
= [16 − 1] [ + ] = −3 = units
2 3 3 3 2
𝒙𝟐 𝒚𝟐
2. Using integration find the area of the region bounded by the ellipse 𝒂𝟐
+ 𝒃𝟐 = 𝟏
Soln: Given equation of the ellipse is:

Now, Area of ellipse = 4 × Area of AOB

= 2ab × sin-1(1)
= 2ab × π/2
= πab sq. units
3. Find the area of the region bounded by y2 = 9x, x = 2, x = 4 and the x-axis in the first quadrant.
Solution: We can draw the figure of y2 = 9x; x = 2, x = 4 and the x-axis in the first curve as below.

y2 = 9x
y = ±√(9x)
y = ±3√x
We can consider the positive value of y since the required area is in the first quadrant.

Page 63 of 199
The required area is the shaded region enclosed by ABCD.

= 2 [(2)3 – (√2)3]
= 2[8 – 2√2]
= 16 – 4√2
Hence, the required area is 16 – 4√2 sq. units.

Important Questions for Practice (3 and 5 marks)

1. Using the integrals find the area bounded by the circle 𝑥 2 + 𝑦 2 = 4 𝑎𝑛𝑑 𝑡ℎ𝑒 𝑙𝑖𝑛𝑒 𝑥 = √3𝑦
above the x-axis.
2. Find the area of the region bounded by the curve 𝑥 2 = 4𝑦, 𝑦 = 2 𝑎𝑛𝑑 𝑦 = 4 and the y-axis in the first
quadrant.
3. Find the area bounded between the circles 𝑥 2 + 𝑦 2 = 4 𝑎𝑛𝑑 and (𝑥 − 2)2 + 𝑦 2 = 4, 𝑎𝑏𝑜𝑣𝑒 𝑡ℎ𝑒 𝑥 𝑎𝑥𝑖𝑠.
4. Using the integrals find the area bounded by the circle 𝑥 2 + 𝑦 2 = 1 𝑎𝑛𝑑 𝑡ℎ𝑒 𝑙𝑖𝑛𝑒 𝑥 = 𝑦
above the x-axis.

CASE STUDY BASED QUESTIONS

Case Study-01 In a classroom, teacher explains the properties of a particular curve by saying that this
particular curve has beautiful up and downs. It starts at 1 and heads down until 𝜋 radian, and then heads up
𝜋
again and closely related to sine function and both follow each other, exactly 2 radians apart as shown in figure.

Page 64 of 199
Based on the above information, answer the following questions:
(i) Name the curve, about which teacher explained in the classroom

(a) cosine (b) sine (c) tangent (d) cotangent


𝜋
(ii) Area enclosed by the curve explained in the passage from 0 to 2 above X-axis is:
(a) 13sq. unit (b) 12sq. unit (c) 1 sq. unit (d) 2 sq. units
𝜋 3𝜋
(iii) Area of curve discussed in classroom from to is:
2 2
(a) -2 sq. units (b) 2 sq. units (c) 3 sq. units (d) -3 sq. units

(iv)Area of explained curve from 0 to 2π is:


(a) 1 sq. unit (b) 2 sq. units (c) 3 sq. units (d) 4 sq. units

Answers: (i)(a) cosine (ii) (c) 1 sq. unit (iii) (b) 2 sq. units (iv) (d) 4 sq. units

Case Study-02 Boundary of a park is in the shape of parabola, whose equation is given by 𝑥 2 = 𝑦, A cross
road intersects the boundary at two distinct points A and B. The equation of the road is y = x.
On the basis of above information answer the following questions:

(i) Co-ordinates of the point(s) of intersection of boundary and the cross road are:
(a) (0,0), (2, 2) (b) (0,0), (1, 1) (c) (0,0), (-1, -1) (d) (0,0), (-2, -2)
(ii) Which of the following figure represent the given situation?

1
(iii) The value of the integral ∫0 𝑥𝑑𝑥 is:
(a) 1/4 (b) 1/3 (c) ½ (d) 1
1
(iv)The value of the integral∫0 𝑥 2 𝑑𝑥
is:
(a) 1/4 (b) 1/3 (c) ½ (d) 1
(v) The area bounded by the curves 𝑥 2 = 𝑦 and x = y is:
𝟏 𝟏 𝟏
(a) 𝟔 sq. unit (b) 𝟑 sq. unit (c) 𝟐 sq. unit (d) 1 sq. unit

Answers: (i) (b) (0, 0), (1, 1) (ii) figure a (iii) (c) ½ (iv) (a)

Page 65 of 199
9. DIFFERENTIAL EQUATIONS

(A) MCQ
1 What is the order of differential equation y’’ + 5y’ + 6 = 0?
A. 0 B. 1 C. 2 D. 3

2 What is the sum of the degree and order of differential equation (y’’’)2 + (y’’)3 + (y’)4 + y5 = 0?
A. 2 B. 3 C. 4 D. 5
3 Solution of differential equation x.dy – y.dx = 0 represents:
A. a rectangular hyperbola B. parabola whose vertex is at the origin
C. straight line passing through the origin D. a circle whose centre is at the origin

4 The curve for which the slope of the tangent at any point is equal to the ratio of
the abscissa to the ordinate of the point is:
A. an ellipse B. parabola C. circle D. hyperbola
5 𝒅𝒚
What is the general solution of the differential equation 𝒅𝒙 = √𝟒 − 𝒚𝟐 (−𝟐 < 𝒚 < 𝟐 is:
. A. sin-1 y = x + c B. sin-1 y/2 = x + c C. sin-1 y2 = x + c D. None of these
Answer: B. sin-1 y/2 = x + c

ANSWERS:- (1) C (2) D (3) C (4) D (5) B

(B) Assertion and Reasoning Questions:


In the following questions two statements Assertion (A) and reason (R) are given. Read the statements
carefully and answer as instructed:

(v) Option A, if Both A and R are true and R be a correct explanation of A.


(vi) Option B, if Both A and R are true but R does not a correct explanation of A.
(vii) Option C, if A is true but R is false.
(viii) Option D, if A is false and R is true

1. Assertion (A): Order of a differential equation represents number of arbitrary constants in the general

solution.
Reason (R): Degree of a differential equation represents number of family of curves.
2. Assertion (A): if a homogeneous differential equation is of the form 𝑑𝑦 = 𝑓(𝑥, 𝑦)then the substitution y=vx
𝑑𝑥
reduces it to a differential equation in variable separable form.
𝑑𝑦
Reason (R): a homogeneous differential equation𝑑𝑥 = 𝑔(𝑥, 𝑦) reduces to the variable separable form by
making the substitution x = vy.
3 dy
Assertion ( A ) : The Integrating Factor of the differential equation dx – y = Cosx is ex
y
Reasoning ( R ) : A function of the form f ( x , y ) = xn g ( x ) is called homogeneous function .

Answers: (1) B (2) B (3) D

(C)Very Short Answer Type Questions (2 Marks)

Page 66 of 199
d2 y
1. Verify that the function y = acosx + bsinx is solution of the differential equation dx2+ y = 0
Solution: We have given
y = acosx + bsinx
Differentiating both sides w.r.t x
dy
= - asinx + bcosx
dx
Again differentiating both sides w.r.t x
d2 y
= - acosx + ( -bsinx ) = - ( acosx + bsinx )
dx2
Now L.H.S. = - ( acosx + bsinx ) + acosx + bsinx = 0
= R.H.S.
Hence it is verified that the given function is a solution of given differential equation.

2. Solve 𝐬𝐞𝐜 𝟐 𝐱. 𝐭𝐚𝐧𝐲 𝐝𝐱 + 𝐬𝐞𝐜 𝟐 𝐲. 𝐭𝐚𝐧𝐱 𝐝𝐲 = 𝟎.


Soln. Given equation is:
sec 2 x. tany dx + sec 2 y. tanx dy =0
𝑡𝑎𝑛𝑦 𝑡𝑎𝑛𝑥
=> 2
=− 𝑑𝑥
sec y sec 2 x
𝑖𝑛𝑡𝑒𝑔𝑟𝑎𝑡𝑖𝑛𝑔 𝑏𝑜𝑡ℎ 𝑠𝑖𝑑𝑒𝑠:
𝑡𝑎𝑛𝑦 𝑡𝑎𝑛𝑥
∫ sec2 ydy = ∫ − sec2x 𝑑𝑥
=>log(sec 2 y) = -log (sec 2 x) + logC
=> sec 2 ysec 2 x = C
Ans. 𝐬𝐞𝐜 𝟐 𝐲𝐬𝐞𝐜 𝟐 𝐱 = C
3. Solve: 𝐞𝐱 𝐭𝐚𝐧 𝐲 𝐝𝐱 + (𝟏 − 𝐞𝐱 )𝐬𝐞𝐜 𝟐 𝐲 𝐝𝐲 = 𝟎
Solution : The equation is :
ex tany dx + (1 − ex )sec 2 y dy = 0
dy ex tany
=> =−
dx (1 − ex )sec 2 y
dy ex tany
=> = −( )( )
dx 1 − ex sec 2 y
sec 2 y ex
=> dy = x dx
tany e −1
Integrating both sides we get,
sec 2 y ex
∫ dy = ∫ x dx
tany e −1
=> log ( tan y ) =log(ex − 1) + log c
=> log ( tan y ) =log(ex − 1)c by using (log a + log b = log ab )
Ans: tan y = (𝐞𝐱 − 𝟏)c .
4. Solve (𝟏 + 𝐲 𝟐 )𝐝𝐱 − 𝐲(𝟏 + 𝐱 𝟐 )𝐝𝐲 = 𝟎.
Given equation is:
(1 + y 2 )dx − y(1 + x 2 )dy = 0
=> (1 + y 2 )dx = y(1 + x 2 )dy
𝑦
=> 𝑑𝑦 = (1 + 𝑥 2 )𝑑𝑥
1 + 𝑦2
𝑖𝑛𝑡𝑒𝑔𝑟𝑎𝑡𝑖𝑛𝑔 𝑏𝑜𝑡ℎ 𝑠𝑖𝑑𝑒𝑠 𝑤𝑒 𝑔𝑒𝑡:
𝑦
∫ 1+𝑦 2 𝑑𝑦 = ∫(1 + 𝑥 2 )𝑑𝑥
 𝑖𝑛 𝐿𝐻𝑆 𝑃𝑢𝑡 1 + 𝑦 2 = t
𝑑𝑡
 𝑦𝑑𝑦 = 2
ℎ𝑒𝑛𝑐𝑒, 𝑜𝑛 𝑖𝑛𝑡𝑒𝑔𝑟𝑎𝑡𝑖𝑛𝑔 𝑏𝑜𝑡ℎ 𝑠𝑖𝑑𝑒𝑠 𝑤𝑒 𝑔𝑒𝑡:
𝟏 𝒙𝟑
𝐥𝐨𝐠(𝟏 + 𝒚𝟐 ) = 𝒙 + + c Ans.
𝟐 𝟑

Page 67 of 199
5. Find the general solution of the following differential equation dy/dx = (1 + x²) ( 1+ y²)

Solution: The given differential equation is dy/dx = (1 + x²) ( 1+ y²)


𝑑𝑦/( 1 + 𝑦²) = (1 + 𝑥²)𝑑𝑥
By integrating both sides of the above equation, we get
∫ 𝑑𝑦/( 1 + 𝑦²) = ∫ (1 + 𝑥²)𝑑𝑥
𝑡𝑎𝑛−1 𝑦 = 𝑥 + 𝑥³/3 + 𝐶
The above equation is the required general solution of the differential equation.

𝑺𝒐𝒎𝒆 𝑸𝒖𝒆𝒔𝒕𝒊𝒐𝒏𝒔 𝒇𝒐𝒓 𝑷𝒓𝒂𝒄𝒕𝒊𝒄𝒆:


dy
1. Solve cos 2 x dx + y = tanx.
dy y 1
2. Solve dx + 2( x ) = x2
3. Solve ydx + (x − y 2 )dy = 0.
dy y
4. Solve dx − x = 2x 2
dy
5. Find the general solution of the differential equation dx – y = cosx
(D) Short Answer Type Questions (3 Marks)
dx
1. Solve the differential equation (1 + x + y)= dy
Solution: Given equation is:
dx
x+y+1 = dy
dx
−x= y+1
dy
dx
Comparing with dy + Px = Q where P and Q are functions of y.
Here P = −1 and Q = 1 + y
Integrating factor = e∫ −1 dy = e−y
Solution of the DE is:-
X(I. F.) =∫(𝐼. 𝐹 )𝑄 𝑑𝑦 + 𝐶
=> x.e−y = ∫(1 + y) e−y dy +c
=> ∫ e−y dy + ∫ y. e−y dy + c = −e−y − y. e−y − e−y + c
=> x = -y - 2 + c 𝐞𝐲 Ans.

2. Find the general solution of the differential equation ( x3 + y3 ) dy = x2y dx.


Solution: Given equation is:
( x3 + y3 ) dy = x2y dx.
dy x2y
=> = ……..(i)
dx x3 + y3
𝑐𝑙𝑒𝑎𝑟𝑙𝑦 𝑡ℎ𝑒 𝑔𝑖𝑣𝑒𝑛 𝑒𝑞𝑢𝑎𝑡𝑖𝑜𝑛 𝑖𝑠 𝑎 ℎ𝑜𝑚𝑜𝑔𝑒𝑛𝑒𝑜𝑢𝑠 𝑒𝑞𝑢𝑎𝑡𝑖𝑜𝑛 𝑖𝑛 𝑥 𝑎𝑛𝑑 𝑦. 𝑠𝑜,
𝑝𝑢𝑡 𝑦 = 𝑣𝑥
dy dv
∴ dx = v +𝑥 dx
dv 𝑥 2 𝑣𝑥
𝑝𝑢𝑡 𝑡ℎ𝑒𝑠𝑒 𝑣𝑎𝑙𝑢𝑒𝑠 𝑖𝑛 𝑒𝑞𝑢𝑎𝑡𝑖𝑜𝑛 (𝑖)𝑤𝑒 𝑔𝑒𝑡, v +𝑥 dx = 3
𝑥 3 +𝑣 3𝑥
dv 𝑣
∴ 𝑥 dx = 1+𝑣3 − 𝑣
dv −𝑣 4
∴ 𝑥 dx = 1+𝑣3
1+𝑣 3 𝑑𝑥
∴ 𝑑𝑣 = − 𝑥
𝑣4
∴integrating both sides we get,
1+𝑣 3 𝑑𝑥
∫ 𝑣4 𝑑𝑣 = - ∫ 𝑥
On solving we get:

Page 68 of 199
−1
+ log|v| = - log|x| +C
3𝑣 3
1
∴ logvx =3𝑣3 +C
𝑥3
Or, log y = 3𝑦 3 + 𝐶
Questions for Practice
𝑑𝑦 𝑦2
3. Find the solution of differential equation =
𝑑𝑥 1−𝑥𝑦
4. Solve the differential equation ( y – sin2x ) dx + tanx dy = 0
dy x 2 + y2
5.Solve the differential equation : dx = 2xy

(E) Long Answer Type Questions (5 Marks)


dy π
1. Find the particular solution of dx + ycotx = 2x + x 2 cotx given that y = 0 when x = 2
Solution: Given differentiasl equation is:
dy
+ ycotx = 2x + x 2 cotx
dx
dy
Comparing with linear differential equation dx + Py = Q, we get:
P = cot x , Q = 2x + x 2 cotx
Now integrating factor is;
I F = 𝑒 ∫ 𝑝𝑑𝑥
= 𝑒 ∫ 𝑐𝑜𝑡𝑥𝑑𝑥
= 𝑒 𝑙𝑜𝑔𝑠𝑖𝑛𝑥
= sinx
Now the general solution of the given linear differential equation is
Y (I F) = ∫( 𝐼 𝐹). 𝑄𝑑𝑥
 y.sinx = ∫ 𝑠𝑖𝑛𝑥(2x + x 2 cotx)𝑑𝑥
 y.sinx = ∫(2x. sinx + x 2 cosx)𝑑𝑥
 ysinx =∫(2x. sinx 𝑑𝑥 + x 2 sinx − ∫(2x. sinx 𝑑𝑥 + C
 ysinx = 𝐱 𝟐 𝐬𝐢𝐧𝐱 + C
π
Again we have given: y = 0 when x = 2
𝝅𝟐
 0= 𝟒
+C
𝝅𝟐
 C= − 𝟒
𝝅𝟐
Hence solution of the given equation is: ysinx = 𝐱 𝟐 𝐬𝐢𝐧𝐱 −
𝟒
𝑑𝑦 𝑦
2. Find the particular solution of the differential equation x(𝑑𝑥 ) – 𝑦 + 𝑥𝑠𝑖𝑛 (𝑥 ) = 0
𝑔𝑖𝑣𝑒𝑛 𝑡ℎ𝑎𝑡 𝑤ℎ𝑒𝑛 𝑥 = 2 , 𝑦 = 𝜋
Solution: Given differential equation is:
𝑑𝑦 𝑦
𝑥 ( ) – 𝑦 + 𝑥𝑠𝑖𝑛 ( ) = 0
𝑑𝑥 𝑥
𝑑𝑦 𝑦 𝑦
=> (𝑑𝑥 ) = 𝑥 − 𝑠𝑖𝑛 (𝑥 ) … … … (i)
𝑤ℎ𝑖𝑐ℎ 𝑎 ℎ𝑜𝑚𝑜𝑔𝑒𝑛𝑒𝑜𝑢𝑠 𝑑𝑖𝑓𝑓𝑒𝑟𝑒𝑛𝑡𝑖𝑎𝑙 𝑒𝑞𝑢𝑎𝑡𝑖𝑜𝑛 𝑜𝑓 𝑑𝑒𝑔𝑟𝑒𝑒 0.
𝑦
∴ 𝑝𝑢𝑡 𝑥 = 𝑣 or y = vx
𝑑𝑦 𝑑𝑣
∴ 𝑑𝑥 = 𝑣 + 𝑥 𝑑𝑥
𝑠𝑢𝑏𝑠𝑡𝑖𝑡𝑢𝑡𝑖𝑛𝑔 𝑡ℎ𝑒𝑠𝑒 𝑣𝑎𝑙𝑢𝑒𝑠 𝑖𝑛 𝑒𝑞𝑢𝑎𝑡𝑖𝑜𝑛 (𝑖) we get,
𝑑𝑣
∴ 𝑣 + 𝑥 𝑑𝑥 = 𝑣 − 𝑠𝑖𝑛𝑣

Page 69 of 199
𝑑𝑣
∴ 𝑥 𝑑𝑥 = −𝑠𝑖𝑛𝑣
𝑑𝑣 𝑑𝑥
∴ =−
𝑠𝑖𝑛𝑣 𝑥
Integrating both sides we get:
𝑑𝑣 𝑑𝑥
∫ 𝑠𝑖𝑛𝑣 = − ∫ 𝑥
=>𝑙𝑜𝑔(𝑠𝑖𝑛𝑣) = −𝑙𝑜𝑔𝑥 + logc
=>𝑙𝑜𝑔(𝑠𝑖𝑛𝑣) + 𝑙𝑜𝑔𝑥 = logc
=>𝑙𝑜𝑔𝑥(𝑠𝑖𝑛𝑣) = logc
=> 𝑥(𝑠𝑖𝑛𝑣) = c
𝒚
Or 𝒙 (𝒔𝒊𝒏 ) = c
𝒙
Questions for Practice (3 and 5 Marks)
𝑑𝑦
1. Find the particular solution of the given differential equation 𝑑𝑥 = 1 + 𝑥 2 + 𝑦 2 + 𝑥 2 𝑦 2
given that y= 1 when x = 0
dy
2. Solve the initial value problem: (x3+ x2 + x + 1)dx = 2x2 + x when y (0) = 1
dy
3. Show that the differential equation (x – y)dx = x + 2y is homogeneous equation and solve it.
ydx−xdy
4. Find the general solution of the differential equation =0
𝑥

(F) CASE STUDY BASED QUESTIONS


Case Study-01 It is known that, if the interest is compounded continuously, the principal changes at the
rate equal to the product' of the rate of bank interest per annum and the principal. Let P denotes the
principal at any time t and rate of interest be r % per annum.

Based on the above information, answer the following questions.

(i) Find the value of dP/dt ?


𝑃𝑟 𝑃𝑟 𝑃𝑟
(a) 100 (b) 1000 (c) 10 (d) None of these
(ii) The differential equation corresponding to the given situation is:
𝑑𝑃 𝑑𝑃 𝑑𝑃 𝑃𝑟
=𝑃 (b) = 𝑃𝑟 (c) = (d) None of these
𝑑𝑡 𝑑𝑡 𝑑𝑡 100
(iii) If Po be the initial principal, then find the solution of differential equation formed in
given situation.
(iV)
1. At what interest rate will Rs.100 double itself in 10 years? (loge2 = 0.6931).
(a) 9.66% (b) 8.239% (c) 7.341% (d) 6.931%

Page 70 of 199
Answers: (i) a (ii) c (iii) solve the D E (variable separable form) (iv) d

Case Study – 02 A rumor on whatsapp spreads in a population of 5000 people at a rate proportional to the
product of the number of people who have heard it and the number of people who have not. Also, it is given
that 100 people initiate the rumor and a total of 500 people know the rumor after 2 days.

Based on the above information, answer the following questions

(i) f y(t) denotes the number of people who know the rumour at an instant t, then maximum value f y(t) is:
(a) 500 (b) 100 (c) 5000 (d) none of these

ii) 𝑑𝑌
The value of 𝑑𝑡
is proportional to:
(y - 5000) (b) y(y - 500) (c) 𝐲(𝟓𝟎𝟎 − 𝐲) (d) y(5000 - y)
(iii) The value of y(0) is:
(a) 100 (b) 500 (c) 600 (d) 200
(iV)2. The value of y(2) is:
(a) 100 (b) 500 (c) 600 (d) 200

Answers: (i) (c) : Since, size of population is 5000.


∴ Maximum value of y(t) is 5000.
(ii) (d) : Clearly, according to given information
𝑑𝑌
𝑑𝑡
= 𝑘𝑦(5000 − 𝑦)
(iii) (a): Since, rumour is initiated with 100 people.
∴ When t = 0, then y = 100
Thus y(0) = 100
(iv) (b) : Since, rumour is spread in 500 people, after 2 days.
∴ When t = 2, then y = 500.
Thus, y(2) = 500
--------------------------------------------------------------------------------------------------------------------------------------

Page 71 of 199
10. VECTOR ALGEBRA

________________________________________________________
(MCQs)
(1) The value of x for which 𝑥(𝑖̂ + 𝑗̂ + 𝑘̂) is a unit vector.
1
(a) 0 (b) ± (c) 1 (d) ±√3
√3
(2) If 𝜃 is the angle between any two vectors 𝑎 ⃗⃗⃗ and | 𝑎
⃗⃗⃗ & 𝑏 ⃗⃗⃗ | = |𝑎
⃗⃗⃗ . 𝑏 ⃗⃗⃗ | then value of 𝜃 is
⃗⃗⃗ × 𝑏
𝜋 𝜋 𝜋 𝜋
(a) (b) 4 (c) 6 (d) 2
3
(3) The Projection of 𝑎 = 2𝑖̂ − 𝑗̂ + 𝑘̂ 𝑜𝑛 𝑏⃗ = 𝑖̂ + 2𝑗̂ + 2𝑘̂ is
2 1
(a) (b) 3 (c) 2 (d) √6
3
⃗⃗⃗ , ⃗⃗⃗
(4) If 𝑎 ⃗⃗⃗ + ⃗⃗⃗
𝑏 & 𝑐⃗⃗ are three unit vectors such that 𝑎 ⃗⃗⃗ . ⃗⃗⃗
𝑏 + 𝑐⃗⃗ = 0 then the value of 𝑎 𝑏 + ⃗⃗⃗⃗
𝑏 . 𝑐⃗⃗ + 𝑐⃗⃗ . 𝑎
⃗⃗⃗
is
3
(a) 1 (b) 3 (c) − 2 (d) None of these
(5) The value of 𝜆 for which two vectors 2𝑖̂ − 𝑗̂ + 2𝑘̂ 𝑜𝑛 𝑏⃗ = 3𝑖̂ + 𝜆𝑗̂ + 𝑘̂ are perpendicular
(a) 2 (b) 4 (c) 6 (d) 8
____________________________________________________________________________________
(ASSERTION REASONING)
Each of the following questions contains two statements: Assertion (A) and Reason (R). Choose the correct
options out of the choices.
(a). Both A and R are true and R is the correct explanation of ‘A’
(b). Both A and R are true and R is not the correct explanation of ‘A’
(c). A is true but R is false
(d). A is false but R is true

(6) Assertion(A): Two vectors are said to be like vectors if they have the same direction but different
magnitude.
Reason(R): Vector quantities do not have a specific direction.

(7) Assertion(A): The area of the triangle whose two sides are represented by the vectors 2𝑖̂ 𝑎𝑛𝑑 − 3𝑗̂ is 3
sq. unit
1
Reason(R): Area of triangle is given by 2 |𝑎 + 𝑏⃗|.

_______________________________________________________________________________________
(VSA- 2 MARKS)
 
(8) Write a unit vector perpendicular to both the vectors a  î  ĵ  k̂ and b  î  ĵ
 
(9) Find the angle between the vectors a  î  ĵ  k̂ and b  î  ĵ  k̂

(10) Find the projection of the vector î  3 ĵ  7k̂ on the vector 2î  3 ĵ  6k̂
ˆ ˆ
(11) Find the value of  for which the vectors 2 î – 3 j + 4 k̂ and – 4 î + 6 j –  k̂ are collinear.
 
ˆ ˆ
(12) Find the value of p for which a = 3 î + 2 j + 9 k̂ and b = î + p j + 3 k̂ are parallel vectors.

(13) Find the unit vector in the direction of vector PQ , where P and Q are the points(2, 3, 4) and (5, 6, 7),
respectively.

Page 72 of 199
     
(14) If a is a unit vector and ( x – a ). ( x + a ) = 80, then find | x | .
ˆ ˆ
(15) Write the value of ( î  j ). k̂ + ( j  k̂ ) . î
_______________________________________________________________________________________
(SA- 3 MARKS)

⃗⃗⃗ − ⃗⃗⃗
(16) Find| 𝑎 ⃗⃗⃗ & ⃗⃗⃗
𝑏 |, if two vectors 𝑎 𝑏 are such that| 𝑎 ⃗⃗⃗ | = 2, | 𝑏⃗ | = 3 & 𝑎
⃗⃗⃗ . ⃗⃗⃗
𝑏 =4.
(17) Show that the vectors 2𝑖̂ − 𝑗̂ + 𝑘̂ , 𝑖̂ − 3𝑗̂ − 5𝑘̂ & 3𝑖̂ − 4𝑗̂ − 4𝑘̂ form the vertices of a right angled
triangle.
(18) Find the position vector of a point R which divides the line joining two points P and Q whose
position vectors are 𝑖̂ + 2𝑗̂ − 𝑘̂ 𝑎𝑛𝑑 − 𝑖̂ + 𝑗̂ + 𝑘̂ respectively, in the ratio 2 : 1 (i) internally (ii)
externally.
(19) Find the area of a triangle having the points A(1, 1, 1), B(1, 2, 3) and C(2, 3, 1) as its vertices.
(20) Vectors 𝑎 ⃗⃗⃗ & 𝑐⃗⃗ are such that 𝑎
⃗⃗⃗ , 𝑏 ⃗⃗⃗ + 𝑐⃗⃗ = 0 and| 𝑎
⃗⃗⃗ + 𝑏 ⃗⃗⃗ | = 3, | 𝑏⃗ | = 5 𝑎𝑛𝑑 | 𝑐 | = 7 . Find the
angle between 𝑎 ⃗⃗⃗ & ⃗⃗⃗
𝑏.
_______________________________________________________________________________________
(LA- 5 MARKS)
(21) Let 𝑎 = 𝑖̂ + 4𝑗̂ + 2𝑘̂, 𝑏⃗ = 3𝑖̂ − 2𝑗̂ + 7𝑘̂ 𝑎𝑛𝑑 𝑐 = 2𝑖̂ − 𝑗̂ + 4𝑘̂ . Find a vector 𝑑 which is
perpendicular to both 𝑎 𝑎𝑛𝑑 𝑏⃗ 𝑎𝑛𝑑 ⃗⃗⃗ 𝑐 . 𝑑 = 15.
(22) If 𝑎 = 2𝑖̂ − 𝑗̂ − 2𝑘̂, 𝑎𝑛𝑑 𝑏⃗ = 7𝑖̂ + 2𝑗̂ − 3𝑘̂ then express 𝑏⃗ in the form of 𝑏⃗ = ⃗⃗⃗
𝑏1 + ⃗⃗⃗⃗
𝑏2 , where ⃗⃗⃗
𝑏1 is
parallel to 𝑎 and ⃗⃗⃗⃗
𝑏2 is perpendicular to 𝑎 .
_______________________________________________________________________________________
(CASE BASED QUESTIONS)
(23) Solar panels have to be installed carefully so that the tilt of the roof, and the direction to the Sun,
produce the largest possible electrical power in the solar panels. A survey uses his instrument to
determine the coordinates of the four corners of a roof where solar panels are to be mounted. In the
picture, suppose the points are labeled counter clockwise from the roof corner nearest to the camera in
units of meters 𝑃1 (6, 8, 4), 𝑃2 (21, 8, 4), 𝑃3 (21, 16, 10) 𝑎𝑛𝑑 𝑃4 (6, 16, 10).

Page 73 of 199
On the basis of above information answer the following questions.
(1) Find the component of two edge vectors defined by
𝐴 = 𝑝𝑜𝑠𝑖𝑡𝑖𝑜𝑛 𝑣𝑒𝑐𝑡𝑜𝑟 𝑜𝑓 𝑃2 − 𝑝𝑜𝑠𝑖𝑡𝑖𝑜𝑛 𝑣𝑒𝑐𝑡𝑜𝑟 𝑜𝑓 𝑃1 and
⃗ = 𝑝𝑜𝑠𝑖𝑡𝑖𝑜𝑛 𝑣𝑒𝑐𝑡𝑜𝑟 𝑜𝑓 𝑃4 − 𝑝𝑜𝑠𝑖𝑡𝑖𝑜𝑛 𝑣𝑒𝑐𝑡𝑜𝑟 𝑜𝑓 𝑃3
𝐵
(2) Find the magnitude of vectors 𝐴 𝑎𝑛𝑑 𝐵⃗
(3) Find the component of 𝑁⃗ , perpendicular to 𝐴 and 𝐵
⃗ and the surface of the roof.
(24) The slogan on chart papers are to be placed on school display board at the points A, B and C
displaying A(Hub of Learning), B(Creating a better world for tomorrow) and C(Education comes
first). The coordinates of these points are (1, 4, 2), (3, -3, -2) and (-2, 2, 6) respectively.

On the basis of above information answer the following questions.


⃗⃗⃗ , ⃗⃗⃗
(1) If 𝑎 ⃗⃗⃗ + ⃗⃗⃗
𝑏 & 𝑐⃗⃗ be the position vectors of points A, B, C respectively, then find |𝑎 𝑏 + 𝑐⃗⃗ |.
(2) Find the area of the triangle ∆ 𝐴𝐵𝐶
(3) Find the unit vector in the direction of ⃗⃗⃗⃗⃗⃗
𝐴𝐶 .

_______________________________________________________________________________________
(ANSWERS)
1
Ans1: (b) ±
√3
𝜋
Ans2: (b) 4
2
Ans3: (a) 3
3
Ans4: (c) − 2
Ans5: (d) 8
Ans6: (c) A is true but R is false
Ans7: (c) A is true but R is false
î ĵ
Ans8:  
2 2

Page 74 of 199
 1
Ans9: cos1  
 3
 
 a.b
Projection of a on b  
=5
Ans10: b

Ans11: 8
2
Ans12: p 
3
1
3

î  ĵ  k̂ 
Ans13:

| x |=9
Ans14:
Ans15: 2
Ans16: | 𝑎 ⃗⃗⃗ | = √5
⃗⃗⃗ − 𝑏
2 2 2
⃗⃗⃗⃗⃗ | = |𝐵
Ans17:|𝐴𝐵 ⃗ 𝐶| + |𝐴𝐶 ⃗⃗⃗⃗⃗ | ⇒ (√41)2 = (√6)2 + (35)2 Vectors are the vertices of a right
angled triangle.
−𝑖̂ +4𝑗̂
Ans18: (i) ̂
(ii) −3𝑖̂ + 4𝑘
3
1 √21
Ans19: Area = |𝐴𝐵 ⃗⃗⃗⃗⃗ × ⃗⃗⃗⃗⃗
𝐴𝐶 | = 𝑠𝑞. 𝑢𝑛𝑖𝑡
2 2
𝜋
Ans20: 𝜃 =
3
̂
160𝑖̂−5𝑗̂ +70𝑘
Ans21: 𝑑 =
3
Ans22: 𝑏1 = 4𝑖̂ − 2𝑗̂ − 4𝑘̂ 𝑎𝑛𝑑 ⃗⃗⃗⃗
⃗⃗⃗ 𝑏2 = 3𝑖̂ + 4𝑗̂ + 𝑘̂
Ans23: (1) 𝐴 = 15𝑖̂ 𝑎𝑛𝑑 𝐵 ⃗ = 8𝑗̂ + 6𝑘̂
(2) 15, 10
⃗ = −90𝑗̂ + 120𝑘̂,
(3) 𝑁 𝑐𝑜𝑚𝑝𝑜𝑛𝑒𝑛𝑡𝑠 𝑎𝑟𝑒 − 90, 120
⃗⃗⃗ + ⃗⃗⃗
Ans24: (1) |𝑎 𝑏 + 𝑐⃗⃗ | = √29
1 √1937
⃗⃗⃗⃗⃗ × 𝐴𝐶
(2) Area =2 |𝐴𝐵 ⃗⃗⃗⃗⃗ | = 𝑠𝑞. 𝑢𝑛𝑖𝑡
2
−3𝑖̂−2𝑗̂ +4𝑘̂
(3) unit vector in the direction of AC is =
√29

Page 75 of 199
11. THREE-DIMENSIONAL GEOMETRY
(MCQs)
(1) The coordinate of the foot of perpendicular drawn from the point (2, 5, 7) on the x- axis is
(a) (2, 0, 0) (b) (0, 5, 0) (c) (0, 0, 7) (d) (0, 5, 7)
(2) The equations of x-axis in space are
(a) x = 0, y = 0 (b) x = 0, z = 0 (c) x = 0 (d) y = 0, z = 0
(3) A line makes equal angles with co-ordinate axis. Direction cosines of this line are
1 1 1 1 −1 −1 1 1 1
(a) ±(1, 1, 1) (b) ±( , , ) (c) ±( , , ) (d) ±(3 , , )
√3 √3 √3 √3 √3 √3 3 3
𝑥−2 𝑦+4 𝑧−3 𝑥 𝑦−1 𝑧+3
(4) The lines = = and = = are
1 2 3 2 4 6
(a) Parallel (b) Skew (c) Perpendicular (d) intersecting
𝑥−5 𝑦+7 𝑧−2
(5) The vector equation of the line = = is
3 −4 −1
(a) 𝑟 = (5𝑖̂ + 7𝑗̂ + 2𝑘̂ ) + 𝜆(3𝑖̂ − 4𝑗̂ − 𝑘̂) (c) 𝑟 = (3𝑖̂ − 4𝑗̂ − 𝑘̂) + 𝜆(5𝑖̂ + 7𝑗̂ + 2𝑘̂)
(b) 𝑟 = (5𝑖̂ − 7𝑗̂ + 2𝑘̂ ) + 𝜆(3𝑖̂ − 4𝑗̂ − 𝑘̂) (d) 𝑟 = (5𝑖̂ + 7𝑗̂ + 2𝑘̂) + 𝜆(3𝑖̂ + 4𝑗̂ + 𝑘̂)

__________________________________________________________________________________

(ASSERTION REASONING)
Each of the following questions contains two statements: Assertion (A) and Reason (R). Choose
the correct options out of the choices.
(a). Both A and R are true and R is the correct explanation of ‘A’
(b). Both A and R are true and R is not the correct explanation of ‘A’
(c). A is true but R is false
(d). A is false but R is true

(6) Assertion(A): Equation of line Passing through the point (1,2,3) and ( 2,-1,5) is
𝑥−1 𝑦−2 𝑧−3
= =
1 −3 2
Reason(R): Equation of line Passing through the point (𝑥1, 𝑦1, , 𝑧1 ) 𝑎𝑛𝑑 (𝑥2, 𝑦2, , 𝑧2 ) is
𝑥 − 𝑥1 𝑦−𝑦1 𝑧−𝑧1
= = .
𝑥 −𝑥 2𝑦 −𝑦
1 𝑧 −𝑧2 1 2 1
(7) Assertion(A): For every point 𝑃(𝑥, 𝑦, 𝑧) on XY plane, Z-component is zero.
Reason(R): For every point 𝑄(𝑥, 𝑦, 𝑧) on XZ plane, Y- component is zero.
__________________________________________________________________________________

(VSA- 2 MARKS)
(8) Find the direction cosines of x, y and z-axis.
(9) Find the vector and Cartesian equations for the line passing through the points (1, 2, –1) and
(2, 1, 1).
(10)Find the vector equation of a line passing through the point (-2, 3, 2) and parallel to the

line r  (2î  3 ĵ)  (2î  3 ĵ  6k̂ ) .
(11)Show that the points (2,3,4) ,(-!,-2,1) and (5,8,7) are collinear.
2𝑥−1 𝑦+2 𝑧−3
(12) The Cartesian equation of a line AB is = 2 = 3 . Find the direction cosines of a line
√3
parallel to AB.
__________________________________________________________________________________
(SA- 3 MARKS)

Page 76 of 199
x 1 y 1 6  z
(13) Find the value of  so that the lines 1  x  y  2  z  3 and   are perpendicular
3 2 2 3 1 7
to each other.

(14) Find the angle between the lines r  (2 ĵ  3k̂ )  (î  2 ĵ  2k̂ ) and

r  (2î  6 ĵ  3k̂ )  (2î  3 ĵ  6k̂ ) .
(15) Find the equation of the line passing through the points P(-1,3,-2) and perpendicular to the lines
𝑥 𝑦 𝑧 𝑥+2 𝑦−1 𝑧+1
= 2 = 3 and −3 = 2 = 5
1
x  2 y 1 z  3
(16) Find the point on the line   at a distance 5 units from the point P(1, 3, 3).
3 2 2
x 1 y  3 z  5 x2 y4 z6
(17) Show that the lines   and   intersect. Find their point of
3 5 7 1 3 5
intersection.
_________________________________________________________________________________
(LA- 5 MARKS)
x y 1 z  2
(18) Find the image of the point (1, 6, 3) in the line   .
1 2 3

(19) Find the shortest distance between the lines r =( î  2 ĵ  k̂ )   (î  ĵ  k̂ ) and

r =2 î  ĵ  k̂)  (2î  ĵ  2k̂)
_____________________________________________________________________________________
(CASE BASED QUESTIONS)
(20) The equation of motion of a Missile are x = 3t, y = -4t, z = t, where the time‘t’ is given in seconds,
and the distance is measured in kilometres.
Based on the above answer the following:

Q 1.What is the path of the Missile?


Q 2.At what distance will the rocket be from the starting point (0, 0, 0) in 5 seconds?
Q3. If the position of rocket at a certain instant of time is (5, -8, 10), then what will be the height of the rocket
from the ground? ( The ground is considered as XY-plane)
(21) Rohan wants to prepare a model for the science exhibition. He wanted to show something about the forces. He
prepared a model for picking heavy object as shown in figure below, where the forces in the cable are given.
Based on the above answer the following:

Page 77 of 199
Q1. Write the coordinates of the points A and E.
Q2. Find the Cartesian equation of line along EA.
Q3. Find the position vector of ED.
(22) A motor cycle race was organised in a town, where the maximum speed limit was set by the
organizers. No participant are allowed to cross the specified speed limit, but two motorcycles A and
B are running at the speed more than allowed speed on the road along the lines 𝑟 = 𝜆(𝑖̂ + 2𝑗̂ − 𝑘̂)
and 𝑟 = (3𝑖̂ + 3𝑗̂) + 𝜇(2𝑖̂ + 𝑗̂ + 𝑘̂), respectively.

Based on the above answer the following:

Q1. Find the Cartesian equation of line along which motorcycle A is running.
Q2. Find the shortest distance between the lines.
Q3. Find the point, where the line joining the points (0, 5, 4) and (1, 3, 6) meets XY-plane.

Page 78 of 199
(ANSWERS)

Ans1: (a) (2, 0, 0)


Ans2: (d) y = 0, z = 0
1 1 1
Ans3: (b) ±( , , )
√3 √3 √3
Ans4: (a) Parallel
Ans5: (b) 𝑟 = (5𝑖̂ − 7𝑗̂ + 2𝑘̂) + 𝜆(3𝑖̂ − 4𝑗̂ − 𝑘̂)
Ans6: (a) Both A and R are true and R is the correct explanation of ‘A’
Ans7: (b) Both A and R are true and R is not the correct explanation of ‘A’
Ans8: (1, 0, 0) (0, 1, 0) and (0, 0, 1)
𝑥−1 𝑦−2 𝑧+1
Ans9: Eq. of line = =
1 −1 2
𝑟 = (−2𝑖̂ + 3𝑗̂ + 2𝑘̂) + 𝜇(2𝑖̂ − 3𝑗̂ + 6𝑘̂)
Ans10:
Ans11: : Let P(2,3,4), Q (-!,-2,1) and R(5,8,7) be the given points.
He direction ratios of PQ are(-1-2,-2-3,1-4) direction ratios of QR are
−3 −5 −3
(5+1,8+2,7-1) ie(-3,-5,-3) and (6,10,6) so 6 = 10 = 6 lines are parallel
the points are collinear.

Ans12: (√𝟑/√𝟓𝟓 , 𝟒/√𝟓𝟓 , 𝟔/√𝟓𝟓)


Ans13: 𝜆 = −2
4
Ans14: 𝜃 = cos −1 21
Ans15: 𝑟 = (−𝑖̂ + 3𝑗̂ − 2𝑘̂) + 𝜇(2𝑖̂ + 7𝑗̂ + 4𝑘̂ )
Ans16: (-2,-1,3) or (4,3,7)
1 1 3
Ans17:  ,  , 
2 2 2
Ans18: image 1, 0, 7
3 2
Ans19: unit
2
Ans20: (1) Straight line
(2) √650 = 5√26 𝑘𝑚
(3) Height of the rocket from the ground (i. e. XY-plane)= 10 km
Ans21: (1) A(8, -6, 0) and B(0, 0, 24)
𝑥 𝑦 𝑧−24
(2) line = =
−4 3 12
(3) (−8𝑖̂ − 6𝑗̂ − 24𝑘̂)
𝑥 𝑦 𝑧
Ans22: (1) = = −1
1 2
(2) 0
(3) Point is (-2, 9, 0)

Page 79 of 199
12. LINEAR PROGRAMMING PROBLEM
CH-12 LINEAR PROGRAMMING PROBLEM
Q. MCQs Marks
1 Feasible region is the set of points which satisfy 1
(a) the objective functions (b) some of the given constraints
(c) all of the given constraints (d) none of these
ANS: (c)
2 Solution of LPP To maximise Z = 4x + 8ysubject to constraints : 2x + y ≤ 30, x + 1
2y ≤ 24, x ≥ 3, y ≤ 9, y ≥ 0 is
(a) x = 12, y = 6 (b) x = 6, y = 12 (c) x = 9, y = 6 (d) none of these
ANS: (a)
3 Of all the points of the feasible region, for maximum or minimum of objective 1
function, the point lies
(a) inside the feasible region
(b) at the boundary line of the feasible region
(c) vertex point of the boundary of the feasible region
(d) none of these
ANS: (c)
4 The feasible region of the inequality x+y≤1 and x–y≤1 lies in .......................... quadrants. 1
(a) Only I and II (b)Only I and III
(c)Only II and III (d)All the four
Ans: c
5 The region represented by thhe inequation x–y≤–1, x–y≥0, x≥0 ,y≥0 is.......... 1
...........
(a) bounded (b)unbounded
(c)do not exist (d)triangular region
Ans: c
Q. ASSERTION AND REASON BASED QUESTIONS
choose the correct option.
a) Both the assertion and reason are true, and the reason correctly explains the
assertion.
b) Both the assertion and reason are true, but the reason does not correctly explain
the assertion.
c) The assertion is true, but the reason is false.
d) The assertion is false, but the reason is true.
e) Both the assertion and reason are false.
6 Assertion: "The optimal solution to a linear programming problem always occurs at a 1
vertex of the feasible region."

Reason: "This is because the objective function is linear, and the extreme points
(vertices) of the feasible region can be tested systematically for optimality."
7 Assertion: "In linear programming, if the feasible region is empty, it means that there is 1
no solution to the problem."

Reason: "An empty feasible region indicates that the constraints are inconsistent and
cannot be satisfied simultaneously."
Q. SA-I QUESTIONS Marks
8 Solve the following system of inequalities graphically: 2
2x+y ≥4, x+ y ≤3, 2x–3y ≤6

Page 80 of 199
9 Solve the following problem graphically: Minimise Z = 3x + 2y subject to the 2
constraints: x + y ≥ 8, 3x + 5y ≤ 15, x ≥ 0, y ≥ 0

ANS.
Plotting the inequations x + y ≥ 8, 3x + 5y ≤ 15, x ≥ 0, y ≥ 0, we notice there is
no common shaded portion. Hence, no feasible solution; so no minimum Z.

10 The corner points of the feasible region determined by the following system of 2
linear inequalities:
2x + y ≤ 10, x + 3y ≤ 15, x, y ≥ 0 are (0, 0), (5, 0), (3, 4) and (0, 5)
Let Z = px + qy, where p, q > 0. What is the condition on p, q. that maximum Z
occurs at both (3, 4) and (0, 5)?

ANS: If maximum Z occurs at both (3, 4) and (0, 5), where Z = px + qy with p,
q>0
Then 3p + 4q = 0p + 5q 3p = q
11 Solve the following system of inequalities graphically: 2
3x+2y ≤150, x+4y≤80, x≤15, y≥0, x ≥ 0

12 Solve the following Linear Programming Problems graphically: 2


MinimiseZ=x+2y
subject to2x +y≥3, x+2y≥6,x,y≥ 0.
Ans:
The value of Z is minimum at every point of the line x+2y=6

Page 81 of 199
Q. SA-II QUESTIONS Marks
13 Solve graphically 3
Maximise Z = 2.5x + y
subject to constraints: x + 3y ≤ 12, 3x + y ≤ 12, x, y ≥ 0

ANS. Plotting the inequations x + 3y ≤ 12, 3x + y ≤ 12


x ≥ 0, y ≥ 0
To Maximise z = 2.5x + y
The shaded region is feasible solution. Possible points for maximum Z are A(4,
0), B(3, 3), C(0, 4).

Z is maximum for B(3, 3), i.e. x = 3, y = 3

14 Solve the following Linear Programming Problem graphically: 3


Minimise Z = 3x + 5y such that x + 3y ≥ 3, x + y ≥ 2, x, y ≥ 0

ANS. Draw graph of inequalities


x + 3y ≥ 3, x + y ≥ 2, x ≥ 0, y ≥ 0
The feasible region determined by the system of constraints, x + 3y ≥ 3, x + y ≥
2 and x, y ≥ 0 is shown;
It can be seen that the feasible region is unbounded.

The corner points of the feasible region are A(3, 0), B , and C(0, 2).
The values of Z at these corner points are as follows:

As the feasible region is unbounded, therefore, 7 may or may not be the


minimum value of Z. Now, we draw the graph of the inequality, 3x + 5y < 7 and
check whether the resulting half plane has points in common with the feasible
region or not.
It can be seen that the feasible region has no common point with

Page 82 of 199
3x + 5y < 7. Therefore, the minimum value of Z is 7 at .

15 Solve the following linear programming problem graphically: 3


Minimise Z = x + 2y subject to 2x + y ≥ 3, x + 2y ≥ 6, x, y ≥ 0.

ANS. The feasible region determined by the constraints, 2x + y ≥ 3, x + 2y ≥ 6, x


≥ 0 and y ≥ 0 is as shown.
The corner points of the feasible region are A(6, 0) and B(0, 3).
The values of Z at these corner points are as follows.

As feasible region is unbounded, we draw the solution region for x + 2y ≥ 6 and


notice no point of the solution region (dotted) lies in the feasible region.
Also, it can be seen that the value of Z is same at points A and B. If we take any
other point such as (2, 2) and (4, 1) on line x + 2y = 6, then
Z=6
Thus, the minimum value of Z occurs for more than 2 points.
Therefore, the value of Z is minimum at every point on the line,
x + 2y = 6

16 Show that the minimum of Z occurs at more than two points. Minimise and 3
Maximise Z = x + 2y subject to x + 2y ≥ 100, 2x – y ≤ 0, 2x + y ≤ 200; x, y ≥ 0.
ANS.
Draw graph of inequalities
x + 2y ≥ 100, 2x – y ≤ 0,
2x + y ≤ 200; x, y ≥ 0.
The feasible region determined by the constraints, x + 2y ≥ 100, 2x – y ≤ 0, 2x +
y ≤ 200, x ≥ 0 and y ≥ 0 is shown; ABCDA is the feasible region.
The corner points of the feasible region are A(0, 50), B(20, 40), C(50, 100) and
D(0, 200).

Page 83 of 199
The values of Z at these corner points are as follows:

The maximum value of Z is 400 at (0, 200) and the minimum value of Z is 100 at
all the points on the line segment joining the points (0, 50) and (20, 40).

17 Find graphically, the maximum value of Z = 2x + 5y, subject to constraints given 3


below: 2x + 4y ≤ 8; 3x + y ≤ 6; x + y ≤ 4; x ≥ 0, y ≥ 0
ANS.
Given inequations are 2x + 4y ≤ 8 or x + 2y ≤ 4 3x + y ≤ 6, x + y ≤ 4, x ≥ 0, y ≥ 0,
Maximise Z = 2x + 5y on plotting the graph of the inequations we notice shaded
portion as feasible solution

Possible points for maximum Z are A(2, 0), B , C(0, 2)

Z is maximum at C(0, 2), i.e. x = 0, y = 2, maximum value = 10

Page 84 of 199
Q. LA QUESTIONS Marks
18 Maximise Z = 8x + 9y subject to the constraints given below: 2x + 3y ≤ 6; 3x – 2y 5
≤ 6; y ≤ 1; x, y ≥ 0
ANS.
Given inequations are
2x + 3y ≤ 6, 3x – 2y ≤ 6, y ≤ 1, x, y ≥ 0
Plotting the inequations we notice we have shaded region as feasible solution
possible points

for maximum Z are A(2, 0), B ,C , D(0, 1)

Z is maximum at B , i.e. for x = and y = , maximum value =


22.62.

Page 85 of 199
19 Minimise and Maximise Z = 5x + 2y subject to the following constraints: 5
x – 2y ≤ 2
3x + 2y ≤ 12
–3x + 2y ≤ 3
x ≥ 0, y ≥ 0

ANS.
To minimise and maximise Z = 5x + 2y,
subject to the constraints.
x – 2y ≤ 2
3x + 2y ≤ 12
–3x + 2y ≤ 3
x ≥ 0, y ≥ 0
Plotting the inequations we get shaded portion as feasible solution. Possible
points for

maximum and minimum Z are A(2, 0), B ,C ,D .

Minimum value = 0 at x = 0, y =

Maximum value = 19 at x = ,y=

Page 86 of 199
20 Solve the following linear programming problem graphically.
Minimise Z = 3x + 5y
subject to the constraints
x + 2y ≥ 10
x+y≥6
3x + y ≥ 8
x, y ≥ 0
ANS.
On plotting the inequations we notice shaded portion is feasible solution.
Possible points for minimum Z are A(10, 0), B(2, 4), C(1, 5) and D(0, 8)

As region is unbounded, we notice there is no point of the region common to


the region of 3x + 5y ≤ 26
Z is minimum at B(2, 4), i.e. x = 2, y = 4

Page 87 of 199
*
Q. CASE BASED QUESTIONS Marks
(Word problems on LPP are now deleted from syllabus)
21 A manufacturing company makes two models A and B of a product. Each piece 5
of model A requires 9 labour hours for fabricating and 1 labour hour for
finishing. Each piece of model B requires 12 hours for fabricating and 3 hours
for finishing. For fabricating and finishing the maximum hours available are 180
and 30 respectively. The company makes a profit of ₹ 8000 on each piece of
model A and ₹ 12000 on each piece of model B. Company wants to get
maximum profit. On the basis of above information answer the following:
(a) If x and y are the number of pieces of model A and model B respectively
then the objective function is
(i) Maximise Z = 8000x + 1200y
(ii) Maximise Z = 9x + y
(iii) Maximise Z = 180x + 30y
(iv) Maximise Z = 12x + 3y
(b) The constraint for fabrication is
(i) 9x + 12y < 180
(ii) 9x + 2y > 180
(iii) 12x + ay ≤ 180
(iv) 9x + 12y ≤ 180
(c) The constraint for finishing is
(i) 3x + 4y ≥ 60
(ii) x + 3y ≤ 30
(iii) x + 3y ≥ 180
(iv) x + 3y < 30
(d) The maximum profit the company makes is
(i) ₹ 160000 (ii) ₹ 168000 (iii) ₹ 120000 (iv) None of these (e) The
number of pieces of model A and model B are
(i) 6, 12 (ii) 20, 0 (iii) 12, 6 (iv) 0, 10

ANS.
(a) (i) Maximise Z = 8000x + 12000y
(b) (iv) 9x + 12y ≤ 180
(c) (ii) x + 3y ≤ 30
(d) (ii), ₹ 16800 Z(20, 0) = 160000 + 0 = 160000 Z(12, 6) = 96000 + 72000 = 168000
←Maximum Z(0, 10) = 0 + 120000 = 120000
(e) (iii), 12, 6 [∵ maximum Z in at (12, 6)]

22 For the celebration of birthday party Ms. Shukla plans to make two types of 5
cake. She has 5 kg of flour and 1 kg of fat and she has all other ingredients with
no shortage. I type of cake requires 200 g of flour and 25g of fat and II type of
cake require 100 g of flour and 50 g of fat on the basis of above answer the
following.
(a) If x be the number of cake of I type and y be the number of cake of II type
then constraint for available flour is
(i) 200 x + 100 y ≤ 5 (ii) 100x + 200y ≤ 5

Page 88 of 199
(iii) 100 x + 200 y ≤ 5000 (iv) 0.2x + 0.1y ≤ 5
(b) Constrain for available fat is

(i) 25x + 50 y ≥ 1 (ii) 5x + 10y ≤


(iii) 25x + 50 y ≤ 1000 (iv) x + 2y ≥ 40
(c) The maximum number of cakes of I type
(i) 20 (ii) 50 (iii) 30 (iv) 35
(d) The maximum number of cakes of II type
(i) 20 (ii) 10 (iii) 30 (iv) 0
(e) If she starts selling cake at a price of ₹ 50 for type and ₹ 120 for II type,
then maximum profit is
(i) 1000 (ii) 2200 (iii) 2400 (iv) 1700

ANS: (a) (iv), x = number of cake of I type y = number of cake of II


type 200x + 100y ≤ 5000 On dividing by 1000, we
get 0.2 x + 0.1 y ≤ 5
(b) (iii), 25x + 50y ≤ 1000

Maximum number of cakes = x + y


(d) (ii), 10
(e) (iii), If C optimal function for profit Maximise Z = 50x + 120y

Page 89 of 199
13. PROBABILITY
Q. MCQs Marks
1 Let A and B be two given independent events such that P(A) = p and P(B) = q 1

and P(exactly one of A, B) = , then value of 3p + 3q – 6pq is (a) 2 (b)


–2 (c) 4 (d) –4

ANS: (a), as P(A) P( ) + P( ) P(B) = ⇒p · (1 – q) + (1 – p)q = ⇒p – pq + q –

pq = ⇒3p + 3q – 6pq = 2
2 If P(A ∩ B) = 70% and P(B) = 85%, then P(A/B) is equal to

(a) (b) (c) (d)

ANS: (a), as P(A/B) =


3 Two dice are thrown once. If it is known that the sum of the numbers on the
dice was less than 6 the probability of getting a sum 3 is

(a) (b) (c) (d)


ANS: (c), as favourable cases for sum less than 6 are 10 and favourable for a
total of 3 is 2.
4 The probability distribution of the discrete variable X is given as:
X 2 3 4 5
P(X)

The value of k is
(a) 8 (b) 16 (c) 32 (d) 48

ANS: (c), as ∑P(X) = 1 ⇒ = 1 ⇒k = 32

5
The probability of A, B and C solving a problem are and respectively.
Then the probability that the problem will be solved is

(a) (b) (c) (d) none of these


ANS: (b) as P(problem solved) = 1 – P(more solves) = 1 –

Q. ASSERTION AND REASON BASED QUESTIONS


choose the correct option.
a) Both the assertion and reason are true, and the reason correctly explains the
assertion.
b) Both the assertion and reason are true, but the reason does not correctly explain
the assertion.
c) The assertion is true, but the reason is false.
d) The assertion is false, but the reason is true.
e) Both the assertion and reason are false.

Page 90 of 199
6 Assertion: If two events, A and B, are mutually exclusive, the conditional probability of A
given B (P(A|B)) is equal to zero.
Reason: Mutually exclusive events cannot occur simultaneously. Therefore, if event B has
occurred, the probability of A happening is zero.

7 Assertion: The conditional probability of rolling a 6 on a fair six-sided die given that the
roll is an odd number is zero.
Reason: Odd numbers on a six-sided die are 1, 3, and 5. None of these are equal to 6, so
the conditional probability of rolling a 6 given that the roll is odd is zero.
Q. SA-I QUESTIONS Marks
8 Given that the two numbers appearing on throwing two dice are different. Find 2
the probability of the event ‘the sum of numbers on the dice is 4’.
ANS: A : two numbers appearing on throwing the dice are different
A = {(1, 2), (1, 3), (1, 4), (1, 5), (1, 6), (2, 1), (2, 3), (2, 4), (2, 5), (2, 6)
(3, 1), (3, 2), (3, 4), (3, 5), (3, 6), (4, 1), (4, 2), (4, 3), (4, 5), (4, 6)
(5, 1), (5, 2), (5, 3), (5, 4), (5, 6), (6, 1), (6, 2), (6, 3), (6, 4), (6, 5)}
B : sum of number is 4 = {(1, 3), (2, 2), (3, 1)}
A ∩ B : {(1, 3), (3, 1)}
Required probability, P(B/A) =

9 In a class, 40% students study Statistics, 25% Mathematics and 15% both
Mathematics and Statistics. One student is selected at random. Find the
probability that
(i) he studies Statistics, if it is known that he studies Mathematics.
(ii) he studies Mathematics, if it is known that he studies Statistics.
ANS.

10 Consider the experiment of tossing a coin. If the coin shows head, toss it again 2
but if it shows tail then throw a die. Find the conditional probability of the
event ‘the die shows a number greater than 4’ given that ‘there is at least one
tail’.
ANS: Sample space
S = {HH, HT, T1, T2, T3, T4, T5, T6}
A: die shows number > 4 = {T5, T6};
B: there is at least one tail = {HT, T1, T2, T3, T4, T5, T6}
A ∩ B = {T5, T6}

Required probability P(A/B) = =

Page 91 of 199
11 A speaks truth in 60% of the cases, while B in 90% of the cases. In what per cent
of cases are they likely to contradict each other in stating the same fact? In the
cases of contradiction do you think, the statement of B will carry more weight
as he speaks truth in more number of cases than A?
Ans.

in 42% cases they are likely to contradict each other.

May be, it is only probability of B speaking truth as but during discussion


both are treated equally.
12
Probabilities of solving a specific problem independently by A and B are and

respectively. If both try to solve the problem independently, find the


probability that (i) the problem is solved (ii) exactly one of them solves the
problem.
Ans.

Q. SA-II QUESTIONS Marks


13 A family has 2 children. Find the probability that both are boys, if it is known 3
that (i) at least one of the children is a boy. (ii) the elder child is a boy.
ANS: S = {BB, BG, GB, GG}
(i) A: at least one of the children is a boy = BB, BG, GB
B: both are boys = BB
A ∩ B : BB
Probability of both boys when at least one of the children is a boy.

Page 92 of 199
(ii) A: the elder child is a boy = BB, BG
B: both are boys = BB
A ∩ B : BB
Probability of the elder child is a boy.

14 A husband and a wife appear in an interview for two vacancies for the same 3

post. The probability of husband’s selection is and that of wife’s selection is

. What is the probability that (i) both will be selected? (ii) only one of them
will be selected? (iii) none will be selected?

ANS: Probability of husband’s selection P(H) = ;

Probability of wife’s selection P(W) =

(i) P(both selected) = P(H and W) =


(ii) P(only one selected) =

(iii) P(none selected) =

15 Let A and B be independent events with P(A) = 0.3 and P(B) = 0.4. Find 3
(i) P(A ∩ B) (ii) P(A ∪ B) (iii) P(A/B)
ANS: For independent events,P(A ∩ B) = P(A) . P(B)
(i) P(A ∩ B) = P(A) . P(B) = 0.3 × 0.4 = 0.12
(ii) P(A ∪ B) = P(A) + P(B) – P(A ∩ B) = 0.3 + 0.4 – 0.12 = 0.58

(iii) P(A/B) = = 0.3


16 A and B throw a die alternatively till one of them gets a number greater than 3
four and wins the game. If A starts the game, what is the probability of B
winning?
ANS.

Page 93 of 199
17 In the following probability distribution 3

ANS: As this represents a probability distributions


ΣP(X) = 1 0 + 2p + 2p + 3p + p2 + 2p2 + 7p2 + 2p = 1
10p2 + 9p – 1 = 0 10p2 + 10p – p – 1 = 0
10p(p + 1) – 1(p + 1) = 0 (10p – 1) (p + 1) = 0
10p – 1 = 0 or p + 1 = 0

p= or p = –1 (rejected)

p= .

Q. LA QUESTIONS Marks
18 Suppose a television manufacturing company imports its picture tubes from two 5
different countries A and B importing 75% from country A and 25% from country
B. Suppose further that 95% of the picture tubes supplied by country A and 80%
supplied by country B perform according to specifications. Find the probability
that a picture tube which performs according to specifications came from
country A.
ANS.

Page 94 of 199
19 5

Ans. (i) For probability distribution, ΣP(X) = 1


0 + k + 2k + 2k + 3k + k2 + 2k2 + 7k2 + k = 1 10k2 + 9k – 1 = 0
(10k – 1) (k + 1) = 0 10k – 1 = 0 or k + 1 = 0

k= or k = –1 (rejected)

(ii) P(X < 3) = P(0) + P(1) + P(2) = 0 + k + 2k = 3k = 3 × .

(iii) P(X > 6) = P(7) = 7k2 + k = .

(iv) P(0 < X < 3) = P(1) + P(2) = k + 2k = 3k = 3 × .

Q. CASE BASED QUESTIONS Marks


20 A The reliability of a COVID PCR test is specified as follows: 5
Of people having COVID, 90% of the test detects the disease but 10% goes
undetected. Of people free of COVID, 99% of the test is judged COVID negative
but 1% are diagnosed as showing COVID positive. From a large population of
which only 0.1% have COVID, one person is selected at random, given the COVID
PCR test, and the pathologist reports him/her as COVID positive.

Based on the above information, answer the following:


(a) What is the probability of the ‘person to be tested as COVID positive’ given
that ‘he is actually having COVID?
(b) What is the probability of the ‘person to be tested as COVID positive’ given
that ‘he is actually not having COVID’?
(c) What is the probability that the ‘person is actually not having COVID? (d)
What is the probability that the ‘person is actually having COVID given that ‘he
is tested as COVID positive’?
(e) What is the probability that the ‘person selected will be diagnosed as COVID
positive’?
ANS: (a) 0.9, (b) 0.01, (c) 0.999, (d) 0.083, (e) 0.01089
21 In answering a question on a multiple choice test for class XII, a student either 5
knows the answer or guesses. Let 3/5 be the probability that he knows the

Page 95 of 199
answer and 2/5 be the probability that he guesses. Assume that a student who
guesses at the answer will be correct with probability 1/3. Let E1, E2, E be the
events that the student knows the answer, guesses the answer and answers

correctly respectively.

ANS: (a) (iv), (b) (ii), (c) (i), (d) (iii), (e) (iii)
22 A coach is training 3 players. He observes that the player A can hit a target 4 5
times in 5 shots, player B can hit 3 times in 4 shots and the player C can hit 2
times in 3 shots.
From this situation answer the following

Page 96 of 199
(a) Let the target is hit by A, B: the target is hit by B and, C: the target is hit by
A and C. Then, the probability that A, B and, C all will hit, is

(b) Referring to (a), what is the probability that B, C will hit and A will lose?

(c) With reference to the events mentioned in (a), what is the probability

that ‘any two of A, B and C will hit?

(d) What is the probability that ‘none of them will hit the target’?

(e) What is the probability that at least one of A, B or C will hit the target?

ANS.

Page 97 of 199
SAMPLE PAPERS

1. PREVIOUS YEAR QUESTION PAPERS (2023)

CBSE QUESTION PAPER 65/3/1 SET 1 (Page 99-110)

CBSE QUESTION PAPER 65/3/1 SET 2 (Page 111-122)

CBSE QUESTION PAPER 65/3/1 SET 3 (Page 123-134)

2. CBSE SAMPLE QUESTION PAPER 2023-24


(Page 135-142)

3. CBSE SAMPLE QUESTION PAPER MS 2023-24


(Page 143-161)
4. CBSE ADDITIONAL QUESTION PAPER 2023-24
(Page 162-180)
5. CBSE ADDITIONAL QUESTION PAPER MS 2023-24

(Page 181-199)

Page 98 of 199
Page 99 of 199
Page 100 of 199
Page 101 of 199
Page 102 of 199
Page 103 of 199
Page 104 of 199
Page 105 of 199
Page 106 of 199
Page 107 of 199
Page 108 of 199
Page 109 of 199
Page 110 of 199
Page 111 of 199
Page 112 of 199
Page 113 of 199
Page 114 of 199
Page 115 of 199
Page 116 of 199
Page 117 of 199
Page 118 of 199
Page 119 of 199
Page 120 of 199
Page 121 of 199
Page 122 of 199
Page 123 of 199
Page 124 of 199
Page 125 of 199
Page 126 of 199
Page 127 of 199
Page 128 of 199
Page 129 of 199
Page 130 of 199
Page 131 of 199
Page 132 of 199
Page 133 of 199
Page 134 of 199
SAMPLE QUESTION PAPER
Class:-XII

Session 2023-24

Mathematics (Code-041)

Time: 3 hours Maximum marks: 80

General Instructions:

1. This Question paper contains - five sections A, B, C, D and E. Each section is compulsory. However, there are
internal choices in some questions.

2. Section A has 18 MCQ’s and 02 Assertion-Reason based questions of 1 mark each.

3. Section B has 5 Very Short Answer (VSA)-type questions of 2 marks each.

4. Section C has 6 Short Answer (SA)-type questions of 3 marks each.

5. Section D has 4 Long Answer (LA)-type questions of 5 marks each.

6. Section E has 3 source based/case based/passage based/integrated units of assessment of 4 marks each with

sub-parts.
___________________________________________________________________________________________

Section –A
(Multiple Choice Questions)
Each question carries 1 mark

1, when i  j
Q1. If A   aij  is a square matrix of order 2 such that aij   , then A2 is
 0, when i  j
1 0  1 1 1 1  1 0
(a)   (b)   (c)   (d)  
1 0  22  0 0  22 1 0  22  0 1  22
Q2. If A and B are invertible square matrices of the same order, then which of the following is not correct?
1
 AB
|A| 1
(a) AB
-1
 (b) 
|B| | A | |B|

(c)  AB   B 1 A1 (d)  A  B   B 1  A 1


1 1

Q3. If the area of the triangle with vertices   3 , 0  ,  3, 0  and  0, k  is 9 sq units, then the value/s of k will
be
(a) 9 (b)  3 (c) -9 (d) 6
 kx
 , if x  0
Q4. If f  x    x is continuous at x  0 , then the value of k is
 3, if x  0

(a) −3 (b) 0 (c) 3 (d) any real number
Page 1 of 8

Page 135 of 199


 
   
Q5. The lines r  i  j  k   2i  3 j  6 k and r  2i  j  k   6i  9 j  18 k ; (where  &  are

scalars) are
(a) coincident (b) skew (c) intersecting (d) parallel
3 2
  dy  2   d 2 y 
Q6. The degree of the differential equation 1       2  is
  dx    dx 
3
(a) 4 (b) (c) 2 (d) Not defined
2
Q7. The corner points of the bounded feasible region determined by a system of linear constraints are
 0, 3  ,  1,1  and  3, 0  . Let Z  px  qy , where p , q  0 . The condition on p and q so that the
minimum of Z occurs at  3, 0  and  1,1  is

q
(a) p  2q (b) p  (c) p  3q (d) p  q
2
   
Q8. ABCD is a rhombus whose diagonals intersect at E . Then EA  EB  EC  ED equals to
   
(a) 0 (b) AD (c) 2BD (d) 2AD

 e
cos 2 x
Q9. For any integer n , the value of Sin 3 (2n + 1) x dx is

(a) -1 (b) 0 (c) 1 (d) 2

 0 2x  1 x 
 
Q10. The value of A , if A  1  2 x 0 2 x  , where x    , is
 
  x 2 x 0 

(a)  2 x  1 (c)  2 x  1 (d)  2 x  1


2 3 2
(b) 0

Q11. The feasible region corresponding to the linear constraints of a Linear Programming Problem is given
below.

Which of the following is not a constraint to the given Linear Programming Problem?
(a) x  y  2 (b) x  2 y  10 (c) x  y  1 (d) x  y  1

Page 2 of 8

Page 136 of 199


   
Q12. If a  4i  6 j and b  3 j  4k , then the vector form of the component of a along b is

(a)
5

18  
3i  4k  (b)
18 
25

3 j  4k  (c)
5

18  
3i  4k  (d)
25

18  
4i  6 j 
Q13. Given that A is a square matrix of order 3 and A  2, then adj  2 A  is equal to
(a) 26 (b)  4 (c) 28 (d) 28
1 1 1
Q14. A problem in Mathematics is given to three students whose chances of solving it are , ,
2 3 4
respectively. If the events of their solving the problem are independent then the probability that the
problem will be solved, is
1 1 1 3
(a) (b) (c) (d)
4 3 2 4
Q15. The general solution of the differential equation ydx – xdy  0;  Given x , y  0  , is of the form

(a) xy  c (b) x  c y 2 (c) y  c x (d) y  cx 2 ;


(Where ' c ' is an arbitrary positive constant of integration)

Q16. The value of  for which two vectors 2 i  j  2 k and 3i   j  k are perpendicular is
(a) 2 (b) 4 (c) 6 (d) 8
Q17. The set of all points where the function f  x   x  x is differentiable, is

(a)  0,   (b)   , 0  (c)   , 0    0,   (d)   ,  

1 1 1
Q18. If the direction cosines of a line are  , ,  , then
c c c

(a) 0  c  1 (b) c  2 (c) c   2 (d) c   3

ASSERTION-REASON BASED QUESTIONS

In the following questions, a statement of Assertion (A) is followed by a statement of Reason (R).
Choose the correct answer out of the following choices.

(a) Both (A) and (R) are true and (R) is the correct explanation of (A).
(b) Both (A) and (R) are true but (R) is not the correct explanation of (A).
(c) (A) is true but (R) is false.
(d) (A) is false but (R) is true.
d
Q19. Let f  x  be a polynomial function of degree 6 such that
dx
 f  x     x  13  x  3 2 , then
ASSERTION (A): f  x  has a minimum at x  1.

d
REASON (R): When
dx
 f  x    0,  x   a  h,a  and dxd  f  x    0,  x   a, a  h ; where
' h ' is an infinitesimally small positive quantity, then f  x  has a minimum at x  a,
provided f  x  is continuous at x  a .

Page 3 of 8

Page 137 of 199


Q20. ASSERTION (A): The relation f : 1, 2,3,4   x , y, z , p defined by f   1, x  ,  2, y  ,  3, z  is a
bijective function.
REASON (R): The function f : 1,2,3   x , y, z , p such that f   1, x  ,  2, y  ,  3, z  is one-one.

Section –B

[This section comprises of very short answer type questions (VSA) of 2 marks each]

  33 
Q21. Find the value of sin 1  cos  .
  5 
OR
Find the domain of sin1 x 2  4 .  
Q22. Find the interval/s in which the function f :    defined by f  x   x e x , is increasing.

1
Q23. If f  x   2
; x   , then find the maximum value of f  x  .
4x  2x  1
OR
Find the maximum profit that a company can make, if the profit function is given by
P  x   72  42 x  x 2 , where x is the number of units and P is the profit in rupees.

1  2 x 
Q24. Evaluate :  1
log 
 2 x
 dx .

Q25. Check whether the function f :    defined by f  x   x 3  x, has any critical point/s or not ?
If yes, then find the point/s.

Section – C
[This section comprises of short answer type questions (SA) of 3 marks each]

2x2  3
Q26. Find :  x x
2 2
9 
dx ; x  0.

Q27. The random variable X has a probability distribution P  X  of the following form, where ' k ' is some
real number:

 k , if x  0
 2k , if x  1

P X   
 3k , if x  2
0, otherwise

(i) Determine the value of k .

(ii) Find P  X  2  .

Page 4 of 8

Page 138 of 199


(iii) Find P  X  2 .

x
Q28. Find :  dx; x   0,1 .
1  x3
OR

Evaluate:  4
log  1  tan x  dx .
0

x
 x 
Q29. Solve the differential equation: ye y dx   xe y  y 2  dy ,
   y  0 .
 
OR

dy  
Solve the differential equation:  cos 2 x   y  tan x ;  0  x  2 .
dx  

Q30. Solve the following Linear Programming Problem graphically:


Minimize: z  x  2 y ,
subject to the constraints: x  2 y  100, 2 x  y  0, 2 x  y  200, x , y  0.
OR
Solve the following Linear Programming Problem graphically:
Maximize: z   x  2 y ,
subject to the constraints: x  3, x  y  5, x  2 y  6, y  0.
y 2
d2 y  a 
Q31. If  a  bx  e x  x then prove that x   .
dx 2  a  bx 

Section –D
[This section comprises of long answer type questions (LA) of 5 marks each]

Q32. Make a rough sketch of the region  x, y  : 0  y  x 2



 1, 0  y  x  1, 0  x  2 and find the

area of the region, using the method of integration.


Q33. Let  be the set of all natural numbers and R be a relation on    defined by

 a , b  R  c, d   ad  bc for all  a , b  ,  c , d      . Show that R is an equivalence relation on

   . Also, find the equivalence class of  2,6  , i.e.,  2, 6   .

OR
x
Show that the function f :    x   :  1  x  1 defined by f  x   , x   is one-one and
1 x
onto function.
Q34. Using the matrix method, solve the following system of linear equations :

Page 5 of 8

Page 139 of 199


2 3 10 4 6 5 6 9 20
   4,    1,    2.
x y z x y z x y z

Q35. Find the coordinates of the image of the point  1, 6 , 3  with respect to the line

   
r  j  2k   i  2 j  3k ; where '  ' is a scalar. Also, find the distance of the image from the
y  axis.
OR

 
An aeroplane is flying along the line r   i  j  k ; where '  ' is a scalar and another aeroplane is flying


 
along the line r  i  j   2 j  k ; where '  ' is a scalar. At what points on the lines should they reach, so

that the distance between them is the shortest? Find the shortest possible distance between them.

Section –E
[This section comprises of 3 case- study/passage based questions of 4 marks each with sub parts.
The first two case study questions have three sub parts (i), (ii), (iii) of marks 1,1,2 respectively.
The third case study question has two sub parts of 2 marks each.)
Q36. Read the following passage and answer the questions given below:
In an Office three employees Jayant, Sonia and Oliver process incoming copies of a certain form. Jayant
processes 50% of the forms, Sonia processes 20% and Oliver the remaining 30% of the forms. Jayant
has an error rate of 0.06 , Sonia has an error rate of 0.04 and Oliver has an error rate of 0.03 .
Based on the above information, answer the following questions.

(i) Find the probability that Sonia processed the form and committed an error.
(ii) Find the total probability of committing an error in processing the form.
(iii) The manager of the Company wants to do a quality check. During inspection, he selects a form at
random from the days output of processed form. If the form selected at random has an error, find the
probability that the form is not processed by Jayant.
OR

Page 6 of 8

Page 140 of 199


(iii) Let E be the event of committing an error in processing the form and let E1 , E2 and E 3 be the
3
events that Jayant, Sonia and Oliver processed the form. Find the value of  P  E E .
i 1
i

Q37. Read the following passage and answer the questions given below:
Teams A , B , C went for playing a tug of war game. Teams A, B , C have attached a rope to a metal ring
and is trying to pull the ring into their own area.

Team A pulls with force F1  6iˆ  0 ˆj kN ,

Team B pulls with force F2  4iˆ  4 ˆj kN ,

Team C pulls with force F3   3 iˆ  3 ˆj kN ,

(i) What is the magnitude of the force of Team A ?


(ii) Which team will win the game?
(iii) Find the magnitude of the resultant force exerted by the teams.
OR
(iii) In what direction is the ring getting pulled?

Q38. Read the following passage and answer the questions given below:

The relation between the height of the plant  ' y ' in cm  with respect to its exposure to the sunlight

1 2
is governed by the following equation y  4 x  x , where ' x ' is the number of days exposed to the
2

sunlight, for x  3.

(i) Find the rate of growth of the plant with respect to the number of days exposed to the sunlight.

Page 7 of 8

Page 141 of 199


(ii) Does the rate of growth of the plant increase or decrease in the first three days?
What will be the height of the plant after 2 days?
*************************************************************************************

Page 8 of 8

Page 142 of 199


SAMPLE QUESTION PAPER
MARKING SCHEME
CLASS XII
MATHEMATICS (CODE-041)
SECTION: A (Solution of MCQs of 1 Mark each)
Q no. ANS HINTS/SOLUTION

1 (d)  0 1  2  1 0
A   , A   0 1 .
1 0  
2 (d)
 A  B
1
 B1  A1 .
3 (b) 3 0 1
1
Area  3 0 1 , given that the area  9 sq unit .
2
0 k 1

3 0 1
1
  9  3 0 1 ; expanding along C 2 , we get  k  3.
2
0 k 1

4 (a) Since, f is continuous at x  0 ,

therefore, L. H . L  R. H . L  f  0   a finite quantity .

lim f  x   lim f  x   f  0
x0 x0

 kx
 lim  lim 3  3  k  3.
x 0 x x 0

5 (d) Vectors 2i  3 j  6k &6i  9 j  18k are parallel and the fixed point i  j  k on the

 
line r  i  j  k   2i  3 j  6k does not satisfy the other line

 
r  2i  j  k   6i  9 j  18k ; where  &  are scalars.

6 (c)   dy  2   d 2 y 
3 2

The degree of the differential equation 1       2  is 2


  dx    dx 

7 (b) Z  px  qy     i 

At  3,0  , Z  3 p     ii  and at  1,1 , Z  p  q      iii 

From  ii  &  iii  , 3 p  p  q  2 p  q .

Page 1 of 19

Page 143 of 199


 
8 (a) Given, ABCD is a rhombus whose diagonals bisect each other. EA  EC and
 
EB  ED but since they are opposite to each other so they are of opposite signs
   
 EA   EC and EB   ED .

     


 EA  EC  O .....  i  and EB  ED  O ....  ii 
    
Adding (i) and (ii), we get EA  EB  EC  ED  O .

9 (b) f  x   e cos x sin 3  2n  1 x


2

f   x  e sin3  2n  1  x 
cos2   x 

2
f (  x )   e cos x sin 3 (2n  1) x
 f ( x )   f ( x )

So,  ecos x sin3 (2n  1) x dx  0
2



10 (b) Matrix A is a skew symmetric matrix of odd order.  A  0.


11 (c) We observe,  0,0  does not satisfy the inequality x  y  1

So, the half plane represented by the above inequality will not contain origin
therefore, it will not contain the shaded feasible region.
12 (b) 
   a .b   18
Vector component of a along b   2 b 

 b 
 25
3 j  4k .  
 
13 (d) adj  2 A    2 A  23 A    26 A  26   2   28 .
2 2 2 2

14 (d) Method 1:
1 1
Let A, B , C be the respective events of solving the problem. Then, P  A  , P  B  
2 3
1
and P  C   . Here, A, B , C are independent events.
4
Problem is solved if at least one of them solves the problem.
Required probability is  P  A  B  C   1  P A P B P C      
Page 2 of 19

Page 144 of 199


 1  1  1 1 3
 1   1    1   1    1   .
 2  3  4 4 4
Method 2:
The problem will be solved if one or more of them can solve the problem. The probability is
        
P ABC  P ABC  P ABC  P ABC  P ABC  P ABC  P  ABC    
1 2 3 1 1 3 1 2 1 1 1 3 1 2 1 1 1 1 1 1 1 3
 . .  . .  . .  . .  . .  . .  . .  .
2 3 4 2 3 4 2 3 4 2 3 4 2 3 4 2 3 4 2 3 4 4
Method 3:
Let us think quantitively. Let us assume that there are 100 questions given to A. A
1
solves  100  50 questions then remaining 50 questions is given to B and B solves
2
1 2
50   16.67 questions . Remaining 50  questions is given to C and C solves
3 3
2 1
50    8.33 questions.
3 4
Therefore, number of questions solved is 50  16.67  8.33  75 .
75 3
So, required probability is  .
100 4
15 (c) Method 1:
ydx  xdy  x 1
ydx  xdy  0  2
 0  d    0  x  y  y  cx.
y  y c
Method 2:
dy dx dy dx
ydx  xdy  0  ydx  xdy 
y

x
; on integrating  y
  x
log e y  log e x  log e c
since x , y , c  0 , we write log e y  log e x  log e c  y  cx .
16 (d) Dot product of two mutually perpendicular vectors is zero.
 2  3   1   2  1  0    8.
17 (c) Method 1:
2 x, x  0
f  x  x  x  
 0 ,x  0

There is a sharp corner at x  0 , so f  x  is not differentiable at x  0 .

Method 2:
Page 3 of 19

Page 145 of 199


Lf '  0   0 & Rf '  0   2 ; so, the function is not differentiable at x  0

For x  0, f  x   2 x (linear function) & when x  0, f  x   0 (constant function)

Hence f  x  is differentiable when x    ,0    0,   .

18 (d)  1  1  1
2 2
1
2 2

We know, l  m  n  1           1  3    1  c   3 .
2 2 2

c c c c


19 (a) d
dx
 f  x     x  1  x  3 
3 2

Assertion : f  x  has a minimum at x  1 is true as

d d
dx
 f  x    0,  x   1  h,1 and
dx
 f  x    0,  x  1,1  h ; where,
' h ' is an infinitesimally small positive quantity , which is in accordance with
the Reason statement.
20 (d) Assertion is false. As element 4 has no image under f , so relation f is not a function.
Reason is true. The given function f : 1, 2, 3   x , y , z , p is one – one, as for each

a  1, 2,3 , there is different image in  x , y, z , p under f .

Section –B
[This section comprises of solution of very short answer type questions (VSA) of 2 marks each]

21   33    3   3    3  1
sin1  cos     sin cos  6 
1 1 1
  sin cos    sin sin   
  5   5   5  2 5 
 3 
   . 1
2 5 10
21 OR  
1  x 2  4  1  3  x 2  5  3  x  5 1

 x    5,  3    3, 5  . So, required domain is   5,  3    3, 5  . 1

22 f  x   x e x  f '  x   e x  x  1 1

1
When x   1,   ,  x  1  0 & e x  0  f '  x   0  f  x  increases in this interval.

or, we can write f  x   x e x  f '  x   e x  x  1 1


2
For f  x  to be increasing, we have f '  x   e x  x  1  0  x  1 as e x  0,  x   1

Hence, the required interval where f  x  increases is  1,   . 1


2
23 1
Method 1 : f  x  
4x  2x  1 ,
2

Page 4 of 19

Page 146 of 199


 2 1 1 3  1 3 3
2
1
Let g  x   4 x  2 x  1  4  x  2 x     4  x     1
2

4 16  4 4 4 4 2
 
1
4
maximum value of f  x   . 2
3
1
Method 2 : f  x   , let g  x   4 x 2  2 x  1
4x  2x  1
2

d 1 d2

dx
 g  x    g '  x   8 x  2 and g '  x   0 at x   also 2  g  x    g"  x   8  0
4 dx
1

1 1 1
 g  x  is minimum when x   so , f  x  is maximum at x  
4 4 2
 1 1 4
maximum value of f  x   f     2
 . 1
 4  1  1 3
4     2    1 2
 4  4
1
Method 3 : f  x  
4x  2x  1
2

  8 x  2 1
On differentiating w.r.t x ,we get f '  x   ....  i 
4 x  2
2
2
 2x  1
1 1
For maxima or minima , we put f '  x   0  8 x  2  0  x   .
4 2
Again, differentiating equation (i) w.r.t. x ,we get


 4 x2  2 x  1   8   8 x  2 2   4 x 
 2 x  1  8 x  2  
2
2

f " x      1
 4 x  2 x  1
4
 2
 2
1  1
At x   , f "     0
4  4
1
f  x  is maximum at x   .
4
1
 maximum value of f  x  is f    
1 1 4
2
 . 2
 4  1  1 3
4    2    1
 4  4
1
Method 4: f  x  
4x  2x  1
2

  8 x  2 1
On differentiating w.r.t x ,we get f '  x   ....  i 
4 x 
2
2
 2x  1 2

1 1
For maxima or minima , we put f '  x   0  8 x  2  0  x   .
4 2
 1 1
When x    h  ,   , where ' h ' is infinitesimally small positive quantity.
 4 4 

4 x  1  8 x  2  8 x  2  0    8 x  2   0 and  4 x 2  2 x  1  0  f '  x   0
2

Page 5 of 19

Page 147 of 199


 1 1 
and when x    ,   h  , 4 x  1  8 x  2  8 x  2  0    8 x  2   0
 4 4  1
1 2
   0  f '  x   0 . This shows that x   is the point of local maxima.
2
and 4 x 2  2 x  1
4
1
maximum value of f  x  is f    
1 1 4
2
 . 2
 4  1  1 3
4    2    1
 4   4
23 OR For maxima and minima, P '  x   0  42  2 x  0 1
2
 x  21 and P "  x   2  0
1
So, P  x  is maximum at x  21 . 2
The maximum value of P  x   72   42  21    21   513 i.e., the maximum profit is 513.
2
1

24 2 x
Let f  x   log  
2 x

2 x 2 x 
We have, f   x   log     log  2  x    f  x  1
2 x   
1 2 x
So, f  x  is an odd function.   log   dx  0. 1
1
2 x
25 f  x   x 3  x , for all x   .
1
d 1
dx
 f  x    f '  x   3 x 2  1; for all x   , x 2  0  f '  x   0 2
1
Hence, no critical point exists.
2

Section –C
[This section comprises of solution short answer type questions (SA) of 3 marks each]

26 2x2  3 1
We have, . Now , let x 2  t

x2 x2  9  2

2t  3 A B 1 5
So,   , we get A  & B  1
t  t  9 t t  9 3 3

2x2  3 1 dx 5 dx 1
 x x2 2
9 
dx  

3 x2 3 x 2
9 2

1 5  x
  tan 1    c , where ' c ' is an arbitrary constant of integration. 1
3x 9 3
27 1 1
We have, (i)  P  X   1  k  2 k  3k  1  k  6 .
i

1
Page 6 of 19

Page 148 of 199


1 1
(ii) P  X  2   P  X  0   P  X  1  k  2k  3k  3   .
6 2 1
(iii) P  X  2   0.

28 3
3 12 1
Let x 2  t  dt  x dx
2 2

x 2 dt 1
 1 x 3
dx  
3 1 t2 2

2 1 1
 sin  t   c
3

2  3 1
 sin1  x 2   c , where ' c ' is an arbitrary constant of integration.
3  
28 OR 
Let I   4 log e  1  tan x  dx ------(i)
0


   a a 1
 I   4 loge  1  tan   x   dx , Using,  f ( x )dx   f (a  x )dx
0
 4  0 0

  
 1  tan x   2 
 I   4 log e  1   dx   4
log e   dx   4
log e 2 dx  I ( Using ------(i) 1
0
 1  tan x  0
 1  tan x  0

1
 
 2I  log e 2  I  log e 2.
4 8

29  xy
x
 x x
 ydx  xdy 
Method 1: ye dx   xe  y  dy  e  ydx  xdy   y dy  e y 
y 2 y 2
  dy 1
   y2 
 
x
 x 1
 e d    dy
y

 y
x x
 x
  e d     dy  e y  y  c , where ' c ' is an arbitrary constant of integration.
y

 y 1
x

dx xe y  y 2
Method 2: We have ,  x
dy
y .e y

dx x y 1
   x ……………. (i) 2
dy y
ey
1
dx dv
Let x  vy   v  y. ; 2
dy dy

Page 7 of 19

Page 149 of 199


dv y 1
So equation (i) becomes v  y v v
dy e 2
dv y 1
y  v
dy e 2
1
 e v dv  dy
2

e dv   dy  e v  y  c  e x / y  y  c 1
v
On integrating we get,
2
where ' c ' is an arbitrary constant of integration.

29 OR The given Differential equation is

dy
 cos x  dx
2
 y  tan x

Dividing both the sides by cos 2 x , we get

dy y tan x
 
dx cos x cos 2 x
2

dy
dx
  
 y sec2 x  tan x sec2 x ........  i  1
2
dy
Comparing with  Py  Q
dx

P  sec 2 x , Q  tan x .sec 2 x


1
The Integrating factor is, IF  e   e
P dx sec2 x dx
 e tan x 2
On multiplying the equation  i  by e tan x , we get

d
dx
     
y .e tan x  e tan x tan x sec2 x  d y .e tan x  e tan x tan x sec2 x dx   1

On integrating we get , y .e tan x   t .e t dt  c1 ; where, t  tan x so that dt  sec2 x dx

 te t  e t  c   tan x  e tan x  e tan x  c

 
 y  tan x  1  c . e  tan x , where ' c1 '& ' c ' are arbitrary constants of integration.
1
30 The feasible region determined by the
constraints, x  2 y  100, 2 x  y  0, 2 x  y  200, x , y  0 , is given below.

Page 8 of 19

Page 150 of 199


1
1
2

A  0, 50  , B  20, 40  , C  50, 100  and D  0, 200  are the corner points of the feasible
region.
1
The values of Z at these corner points are given below.

Corner point Corresponding value of


Z  x  2y

A  0, 50  100 Minimum

B  20, 40  100 Minimum


1
C  50, 100  250
2

30 OR D  0, 200  400

The minimum value of Z is 100 at all the points on the line segment joining the points  0,50 

and  20,40  .

The feasible region determined by the constraints, x  3, x  y  5, x  2 y  6, y  0.


is given below.

1
1
2

Page 9 of 19

Page 151 of 199


Here, it can 1
be seen that
the

feasible region is unbounded.

The values of Z at corner points A  3, 2  , B  4, 1 and C  6, 0  are given below. 1


Corner point Corresponding value of Z   x  2 y 2

A  3, 2  1 ( may or may not be the maximum value)

B  4, 1  -2

C  6, 0  -6

Since the feasible region is unbounded, Z  1 may or may not be the maximum value.

Now, we draw the graph of the inequality, – x  2 y  1 , and we check whether the resulting
open half-plane has any point/s, in common with the feasible region or not.

Here, the resulting open half plane has points in common with the feasible region.

Hence, Z  1 is not the maximum value. We conclude, Z has no maximum value.

31 y  x  1
 log e    log e x  log e  a  bx 
x  a  bx  2

On differentiating with respect to x , we get


dy
x y
dx 1 1 d 1 b
    a  bx    1
x 2
x a  bx dx x a  bx
dy 1 b  ax 1
x  y  x2    
dx  x a  bx  a  bx 2
On differentiating again with respect to x , we get

d 2 y dy dy  a  bx  a  ax  b  1
x   
dx 2 dx dx  a  bx 
2
2

Page 10 of 19

Page 152 of 199


2
d2 y  a 
 x 2   .
dx  a  bx  1
2

Section –D
[This section comprises of solution of long answer type questions (LA) of 5 marks each]

32

To find the point of intersections of the curve y  x 2  1 and the line y  x  1 ,

we write x 2  1  x  1  x  x  1  0  x  0,1.

So, the point of intersections P  0,1  and Q  1,2  . 1

Area of the shaded region OPQRTSO = (Area of the region OSQPO + Area of the region
STRQS )
 
1 2
  x 2  1 dx    x  1 dx 1
0 1
1 2
 x3   x2  1
   x    x
3 0  2 1 2
 1     1 
   1   0   2  2     1  1
 3     2  2
23 23 1
 Hence the required area is sq units.
6 6
33 Let  a , b  be an arbitrary element of    . Then,  a , b      and a, b 

We have, ab  ba ; (As a, b   and multiplication is commutative on  )

  a , b  R  a , b  , according to the definition of the relation R on   

Thus  a , b  R  a , b  ,   a , b      .

So, R is reflexive relation on    . 1


Let  a , b  ,  c , d  be arbitrary elements of    such that  a , b  R  c , d  .

Page 11 of 19

Page 153 of 199


Then,  a , b  R  c , d   ad  bc  bc  ad ; (changing LHS and RHS)

 cb  da; (As a, b, c, d   and multiplication is commutative on  )

  c , d  R  a , b  ; according to the definition of the relation R on   

Thus  a , b  R  c , d    c , d  R  a , b 

So, R is symmetric relation on    . 1


Let  a , b  ,  c , d  ,  e , f  be arbitrary elements of    such that

 a , b  R  c , d  and  c , d  R  e , f  .
 a , b  R  c , d   ad  bc 
Then    ad  cf    bc  de   af  be
 c , d  R  e , f   cf  de 
  a, b  R  e, f ; (according to the definition of the relation R on    )

Thus  a , b  R  c , d  and  c , d  R  e , f    a, b  R  e, f 
So, R is transitive relation on    .
1
As the relation R is reflexive, symmetric and transitive so, it is equivalence relation on    .
1
 2, 6     x , y      :  x , y  R  2, 6  2
1
  x , y      : 3 x  y 2

  x , 3 x  : x     1, 3  ,  2,6 ,  3,9  , .........


1
33 OR
 x
 1  x , if x  0
We have, f  x   
 x , if x  0
 1  x
Now, we consider the following cases
x
Case 1: when x  0 , we have f  x  
1 x
Injectivity: let x, y   0 such that f  x   f  y  , then

x y
   x  xy  y  xy  x  y
1 x 1 y
So, f is injective function. 1
x 1
Surjectivity : when x  0 , we have f  x    0 and f  x   1   1,as x  0
1 x 1 x
y
y 1 y

Let y   0,1  , thus for each y  0,1 there exists x 
1 y
 0 such that f  x  
y
 y.
1
1 y

1
Page 12 of 19

Page 154 of 199


So, f is onto function on  0,   to  0,1 .
x
Case 2: when x  0 , we have f  x  
1 x
Injectivity: Let x, y   i.e., x , y  0 , such that f  x  f  y  , then

x y
   x  xy  y  xy  x  y
1 x 1 y
So, f is injective function.
1
x x 1
Surjectivity : x  0 , we have f  x    0 also, f  x    1   1
1 x 1 x 1 x
1  f  x   0 .
y
Let y    1, 0  be an arbitrary real number and there exists x   0 such that,
1 y
y
 y  1 y
f  x  f    y.
 1 y  1 y
1 y
y
So, for y    1, 0  , there exists x   0 such that f  x   y .
1 y 1
Hence, f is onto function on   , 0  to   1, 0  .
Case 3:
x y
(Injectivity): Let x  0 & y  0 such that f  x   f  y   
1 x 1 y
 x  xy  y  xy  x  y  2 xy , here LHS  0 but RHS  0 , which is inadmissible.
Hence , f  x   f  y  when x  y.
1
Hence f is one-one and onto function.
34 The given system of equations can be written in the form AX  B,

 2 3 10  1 / x   4
     
Where, A   4 6 5  , X  1 / y  and B  1 
 6 9 20   1 / z   2

2 3 10 1
Now, A  4 6 5  2  120  45  3  80  30   10  36  36  2
6 9 20

 2  75   3   110   10  72   150  330  720  1200  0  A1 exists. 1


2
T
 75 110 72   75 150 75 
 adj A  150 100 0   110 100 30 
  
1
1
 75 30 24   72 0 24  2

Page 13 of 19

Page 155 of 199


 75 150 75 
1 1 
Hence, A 
1
 adjA  110 100 30  1
A 1200 
 72 0 24 2

1
 
x  75 150 75   4 
1 1 
1
As, AX  B  X  A B     110 100 30  1  1
y 1200  2
   72 0 24   2
1
z
 
1 1
   
 300  150  150   x  600   2 
1  1 1  1 1
  440  100  60       400     2
1200 y 1200  3
 288  0  48     240   
1 1
z  5 
 

1 1 1 1 1 1
Thus,  ,  ,  Hence, x  2, y  3, z  5 . 1
x 2 y 3 z 5

35 Let P 1,6,3  be the given point, and let ' L ' be the foot of the perpendicular from ' P ' to the

given line AB (as shown in the figure below). The coordinates of a general point on the
given line are given by

x 0 y 1 z  2
    ;  is a scalar, i.e., x   , y  2  1 and z  3  2
1 2 3
Let the coordinates of L be   , 2  1,3  2  .
1
So, direction ratios of PL are   1, 2  1  6 and 3  2  3, i .e .   1, 2  5 and 3  1. 2
1
Direction ratios of the given line are 1, 2 and 3, which is perpendicular to PL . 2

Therefore,    1 1   2  5  2   3  1  3  0  14  14  0    1
1
So, coordinates of L are 1,3,5  .

Page 14 of 19

Page 156 of 199


Let Q  x1 , y1 , z1  be the image of P 1,6,3  in the given line. Then, L is the mid-point of 1
PQ .

Therefore,
 x1  1  1,  y1  6   3 and  z1  3  5  x  1, y1  0 and z1  7
1
2 2 2
Hence, the image of P  1,6,3  in the given line is 1,0,7  .
1
Now, the distance of the point  1,0,7 from the y  axis is 12  72  50 units.

1
35 OR Method 1:

Given that equation of lines are


 
   
r   i  j  k ..............  i  and r  i  j   2 j  k ..............  ii 

The given lines are non-parallel lines as vectors i  j  k and 2 j  k are not parallel. There is a

unique line segment PQ ( P lying on line  i  and Q on the other line  ii  ), which is at right

angles to both the lines. PQ is the shortest distance between the lines. Hence, the shortest possible
distance between the aeroplanes  PQ .

 
Let the position vector of the point P lying on the line r   i  j  k where '  ' is a scalar, is
1
 
 i  j  k , for some  and the position vector of the point Q lying on the line 2

 
r  i  j   2 j  k ; where '  ' is a scalar, is i   1  2  j     k , for some  .
   1
Now, the vector PQ  OQ  OP   1    i   1  2    j       k ; (where ' O ' is the 2

origin), is perpendicular to both the lines, so the vector PQ is perpendicular to both the vectors
i  j  k and 2 j  k .

  1    .1   1  2    .  1       .1  0 &

  1    .0   1  2    .  2        .1  0 1
 2  3  3  0 & 2  5   3  0 2
1
2
Page 15 of 19

Page 157 of 199


2
On solving the above equations , we get   and   0
3
1
So, the position vector of the points, at which they should be so that the distance between them is

the shortest, are


3

2   

i  j  k and i  j . 1

   1 1 2  2 2


1  1  2
2
2
PQ  OQ  OP  i  j  k and PQ             1
3 3 3  3  3  3 3

2
The shortest distance  units.
3
Method 2:

x y z
The equation of two given straight lines in the Cartesian form are   ........ i  and
1 1 1
x1 y1 z
  .........  ii 
0 2 1
The lines are not parallel as direction ratios are not proportional. Let P be a point on straight line 1
i and Q be a point on straight line  ii  such that line PQ is perpendicular to both of the lines. 2

Let the coordinates of P be   ,   ,   and that of Q be  1, 2  1,   ; where '  ' and '  ' are
1
scalars. 2
Then the direction ratios of the line PQ are    1,    2  1,    

Since PQ is perpendicular to straight line  i  , we have,


1
(  1).1  (    2  1).( 1)  (   ).1  0 2

 3  3  2......  iii 
1
Since , PQ is perpendicular to straight line  ii  , we have 2
0.    1      2  1 .( 2)       .1  0  3  5  2........  iv  1

2
Solving  iii  and  iv  , we get   0,   1
3
 2 2 2
Therfore , the Coordinates of P are  ,  ,  and that of Q are  1, 1, 0
 3 3 3 1

Page 16 of 19

Page 158 of 199


2 2 2
 2  2  2 2
So, the required shortest distance is  1  3    1  3    0  3   3
units.
     

Section –E

[This section comprises solution of 3 case- study/passage based questions of 4 marks each with two sub
parts. Solution of the first two case study questions have three sub parts (i),(ii),(iii) of marks 1,1,2
respectively. Solution of the third case study question has two sub parts of 2 marks each.)

36 Let E1 , E2 , E3 be the events that Jayant, Sonia and Oliver processed the form, which are clearly

pairwise mutually exclusive and exhaustive set of events.

50 5 20 1 30 3
Then P  E1    , P  E2    and P  E3    .
100 10 100 5 100 10

Also, let E be the event of committing an error.

We have, P  E | E1   0.06 , P  E | E 2   0.04 , P  E | E 3   0.03.

(i) The probability that Sonia processed the form and committed an error is given by

1
P  E  E2   P  E2  . P  E | E2    0.04  0.008. 1
5

(ii) The total probability of committing an error in processing the form is given by

P  E   P  E1  . P  E | E1   P  E2  . P  E | E2   P  E3  . P  E | E3 

50 20 30
P E   0.06   0.04   0.03  0.047. 1
100 100 100

(iii) The probability that the form is processed by Jayant given that form has an error is given by
P  E | E1   P  E1 
P  E1 | E  
P  E | E1  . P  E1   P  E | E2  . P  E2   P  E | E3  . P  E3 

50
0.06  1
100 30
 
50 20 30 47
0.06   0.04   0.03 
100 100 100
Therefore, the required probability that the form is not processed by Jayant given that form has an
1
 
error  P E1 | E  1  P  E1 | E   1 
30 17
 .
47 47
1
3
(iii) OR  PE E  P E
i 1 | E   P  E 2 | E   P  E3 | E   1
i 1 1
Since, sum of the posterior probabilities is 1.

Page 17 of 19

Page 159 of 199


3
( We have ,  P  E i E   P  E1 | E   P  E 2 | E   P  E 3 | E 
i 1

P  E  E1   P  E  E2   P  E  E3 

P E
P   E  E1    E  E2    E  E3  
 as Ei & E j ; i  j , are mutually exclusive events
P E
P  E  ( E1  E2  E3  PE  S PE
    1; ' S ' being the sample space )
PE PE PE

37 We have ,
  
 4   3    3   18  3 2 kN .
2 2 2
F1  62  02  6 kN , F2   42  32  4 2 kN , F3 
1
(i) Magnitude of force of Team A  6 kN .
   1
(ii) Sin ce a  c  3(i  j )and b  4 (i  j )
  
So, b and a  c are unlike vectors having same intial point 1
  
and b  4 2 & a  c  3 2
      1
Thus F 2  F 1  F 3 also F 2 and F 1  F 3 are unlike
Hence B will win the game
   
(iii) F  F1  F2  F3  6iˆ  0 ˆj  4iˆ  4 ˆj  3iˆ  3 ˆj   iˆ  ˆj
1

 1    1 
2 2
F   2 kN .
1
OR

F   iˆ  ˆj

1  3
     tan 1       ; where' ' is the angle made by the resultant force with the
1 4 4

 ve direction of the x  axis.

38 1
y  4x  x2
2
(i) The rate of growth of the plant with respect to the number of days exposed to sunlight
dy 2
is given by  4  x.
dx
dy
(ii) Let rate of growth be represented by the function g  x   .
dx

Page 18 of 19

Page 160 of 199


d  dy 
Now, g '  x    1  0
dx  dx 

 g  x  decreases. 1

So the rate of growth of the plant decreases for the first three days. 1
1
 2   6 cm .
2
Height of the plant after 2 days is y  4  2 
2

Page 19 of 19

Page 161 of 199


CBSE
Additional Practice Questions
Subject: Mathematics (041)
Class: XII 2023-24
Time Allowed: 3 Hours Maximum Marks: 80
General Instructions:
1. This Question paper contains - five sections A, B, C, D and E. Each section is compulsory.
However, there are internal choices in some questions.
2. Section A has 18 MCQs and 02 Assertion-Reason based questions of 1 mark each.
3. Section B has 5 Very Short Answer (VSA)-type questions of 2 marks each.
4. Section C has 6 Short Answer (SA)-type questions of 3 marks each.
5. Section D has 4 Long Answer (LA)-type questions of 5 marks each.
6. Section E has 3 source based/case based/passage based/integrated units of assessment (4
marks each) with sub parts.

SECTION A
(This section comprises of Multiple-choice questions (MCQ) of 1 mark each.)

Serial
No. Question Marks
1 For any 2 × 2 matrix P, which of the following matrices can be Q such that PQ 1
= QP?

(a)

(b)

(c)

(d)

Page 162 of 199


2 V is a matrix of order 3 such that |adj V| = 7. 1

Which of these could be |V|?

(a)
(b)
(c)
(d)

3 The points D, E and F are the mid-points of AB, BC and CA respectively. 1

(Note: The figure is not to scale.)

What is the area of the shaded region?

(a) 2 sq units
3
(b) 2 sq units
1
(c) 2 sq units
(d) (2√26 - 1) sq units

4 If f(x) = cos-1 √x, 0 < x < 1, which of the following is equal to f'(x)? 1

(a)

Page 163 of 199


(b)

(c)

(d)

5 A function f: R -> R is defined by: 1

Which of the following statements is true about the function at the point
1
x = ln 2 ?

(a) f(x) is not continuous but differentiable.


(b) f(x) is continuous but not differentiable.
(c) f(x) is neither continuous nor differentiable.
(d) f(x) is both continuous as well as differentiable.

6 In which of these intervals is the function f(x) = 3x 2 - 4x strictly decreasing? 1

(a) (-∞, 0)
(b) (0, 2)
2
(c) (3 , ∞)
(d) (-∞, ∞)

7 Which of these is equal to ∫ 𝑒 (𝑥 log 5) 𝑒 𝑥 𝑑𝑥, where C is the constant of 1


integration?

(a)
(b)
(c)
(d)

Page 164 of 199


8 Shown below is the curve defined by the equation y = log (x + 1) for x ≥ 0. 1

Which of these is the area of the shaded region?

(a) 6log(2) - 2
(b) 6log(2) - 6
(c) 6log(2)
(d) 5log(2)

9 In which of the following differential equations is the degree equal to its order? 1

(a)

(b)

(c)

(d)

Page 165 of 199


10 Kapila is trying to find the general solution of the following differential 1
equations.

Which of the above become variable separable by substituting y = b.x, where


b is a variable?

(a) only (i)


(b) only (i) and (ii)
(c) all - (i), (ii) and (iii)
(d) None of the above

11 1

(a) only (i)


(b) only (ii)
(c) only (i) and (ii)
(d) only (ii) and (iii)

12 1

3
(a) λ = , σ = 0
5
5
(b) λ = 3 , σ = 5
(c) λ = 3, σ = 0
(d) (cannot be found as there are two unknowns and only one equation)

Page 166 of 199


13 1

(a) (2, √2, 2)


(b) (√2, 2, √2)
1 1 1
(c) (2 , , )
√2 2
(d) (2√2, 2√2, 2√2)

14 A line m passes through the point (-4, 2, -3) and is parallel to line n, given by: 1

The vector equation of line m is given by:

Which of the following could be the possible values for p, q and r?

(a) p = 4, q = (-2), r = 3
(b) p = (-4), q = (-2), r = 3
(c) p = (-2), q = 3, r = (-6)
(d) p = 8, q = 4, r = (-3)

15 L1 and L2 are two skew lines. 1

Page 167 of 199


How many lines joining L1 and L2 can be drawn such that the line is
perpendicular to both L1 and L2 ?

(a) exactly one


(b) exactly two
(c) infinitely many
(d) (there cannot be a line joining two skew lines such that it is perpendicular
to both)

16 A linear programming problem (LPP) along with the graph of its constraints is 1
shown below. The corresponding objective function is Minimize: Z = 3x + 2y.
The minimum value of the objective function is obtained at the corner point (2,
0).

The optimal solution of the above linear programming problem _________.

(a) does not exist as the feasible region is unbounded.

(b) does not exist as the inequality 3x + 2y < 6 does not have any point in
common with the feasible region.

(c) exists as the inequality 3x + 2y > 6 has infinitely many points in common with
the feasible region.

(d) exists as the inequality 3x + 2y < 6 does not have any point in common with
the feasible region.
17 The feasible region of a linear programming problem is bounded. The 1
corresponding objective function is Z = 6x - 7y.

Page 168 of 199


The objective function attains __________ in the feasible region.

(a) only minimum


(b) only maximum
(c) both maximum and minimum
(d) either maximum or minimum but not both

18 M and N are two events such that P(M ∩ N) = 0. 1

Which of the following is equal to P(M|(M ∪ N))?

(a)

(b)

(c)

(d)

19 X = {0, 2, 4, 6, 8}. 1
P is a relation on X defined by P = {(0, 2), (4, 2), (4, 6), (8, 6), (2, 4), (0, 4)}.

Based on the above information, two statements are given below - one labelled
Assertion (A) and the other labelled Reason (R). Read the statements carefully
and choose the option that correctly describes statements (A) and (R).

Assertion (A): The relation P on set X is a transitive relation.

Reason (R): The relation P has a subset of the form {(a, b), (b, c), (a, c)},
where a, b, c ∈ X.

(a) Both (A) and (R) are true and (R) is the correct explanation for (A).
(b) Both (A) and (R) are true but (R) is not the correct explanation for (A).
(c) (A) is true but (R) is false.
(d) (A) is false but (R) is true.

20 Two statements are given below - one labelled Assertion (A) and the other 1
labelled Reason (R). Read the statements carefully and choose the option that
correctly describes statements (A) and (R).

Page 169 of 199


Assertion (A): The maximum value of the function f(x) = x 5 , x ∈ [-1, 1], is
attained at its critical point, x = 0.

Reason (R): The maximum of a function can only occur at points where
derivative is zero.

(a) Both (A) and (R) are true and (R) is the correct explanation for (A).
(b) Both (A) and (R) are true but (R) is not the correct explanation for (A).
(c) (A) is false but (R) is true.
(d) Both (A) and (R) are false.

SECTION B
(This section comprises of very short answer type-questions (VSA) of 2 marks
each.)

Serial
No. Question Marks
21 Find the domain of the function y = cos-1 (|x - 1|). Show your steps. 2

OR

Draw the graph of the following function: 2

y = 2sin-1 (x), -π ≤ y ≤ π

22 2

23 If x = cot t and y = cosec2 t, find: 2

Page 170 of 199


Show your steps.

24 Iqbal, a data analyst in a social media platform is tracking the number of active 2
users on their site between 5 pm and 6 pm on a particular day.

The user growth function is modelled by N(t) = 1000e0.1t, where N(t)


represents the number of active users at time t minutes during that period.

Find how fast the number of active users are increasing or decreasing at 10
minutes past 5 pm. Show your steps.

OR

The population of rabbits in a forest is modelled by the function below: 2

Determine whether the rabbit population is increasing or not, and justify your
answer.

25 Solve the integral: 2

Show your steps.

SECTION C
(This section comprises of short answer type questions (SA) of 3 marks each)

Serial
No. Question Marks
3
26

Page 171 of 199


Solve the integral:

Show your work.

27 Evaluate the integral: 3

Show your steps.

OR

Using the properties of definite integrals, prove the following: 3

State the property used.

28 When an object is thrown vertically upward, it is under the effect of gravity 3


and air resistance. For small objects, the force due to air resistance is
numerically equal to some constant k times v, where v is the velocity of the
object (in m/s) at time t (s).

This situation can be modelled as the differential equation shown below.

A tennis ball of mass 0.050 kg is hit upwards with a velocity of 10 m/s. An air
resistance numerically equal to 0.4v acts on the ball.

(i) Model the above situation using a differential equation.


(ii) Write an expression for the velocity of the ball in terms of the time.

Show your work.

Page 172 of 199


29 Shown below is a curve. 3

L1 is the tangent to any point (x, y) on the curve.


L2 is the line that connects the point (x, y) to the origin.

The slope of L1 is one third of the slope of L2 .

Find the equation of the curve. Show your work.

OR

(i) Solve the differential equation and show that the solution represents a
family of circles.

(ii) Find the radius of a circle belonging to the above family that passes
through the origin.

Show your work.

30 Each unit of Product A that a company produces, is sold for Rs 100 with a 3
production cost of Rs 60 and each unit of Product B is sold for Rs 150 with
production cost of Rs 90. On a given day, the company has a budget of Rs
8000 to spend on production. The production process makes it such that they
can only produce a maximum of 100 units each day. Also, the number of
product B produced cannot be more than twice as many of Product A.

Frame a linear programming problem to determine how many units of


Product A and B should the company produce in a day in order to maximize
their profit?

Page 173 of 199


(Note: No need to find the feasible region and optimal solution.)

OR

Shown below is the feasible region of a maximisation problem whose 3


objective function is given by Z = 5x + 3y.

i) List all the constraints the problem is subjected to.


ii) Find the optimal solution of the problem.

Show your work.


31 A company follows a model of bifurcating the tasks into the categories shown 3
below.

Page 174 of 199


At the beginning of a financial year, it was noticed that:

♦ 40% of the total tasks were urgent and the rest were not.
♦ half of the urgent tasks were important, and
♦ 30% of the tasks that were not urgent, were not important

What is the probability that a randomly selected task that is not important is
urgent? Use Bayes' theorem and show your steps.

SECTION D
(This section comprises of long answer-type questions (LA) of 5 marks each)

Serial
No. Question Marks
32 The Earth has 24 time zones, defined by dividing the Earth into 24 equal 5
longitudinal segments. These are the regions on Earth that have the same
standard time. For example, USA and India fall in different time zones, but
Sri Lanka and India are in the same time zone.

A relation R is defined on the set U = {All people on the Earth} such that R =
{(x, y)| the time difference between the time zones x and y reside in is 6
hours}.

i) Check whether the relation R is reflexive, symmetric and transitive.


ii) Is relation R an equivalence relation?

Show your work.

Page 175 of 199


OR

A function f : R - {-1, 1} -> R is defined by: 5

i) Check if f is one-one.
ii) Check if f is onto.

Show your work.

33 Abdul threw a basketball in the direction of the basketball hoop which 5


traversed a parabolic path in a vertical plane as shown below.

(Note: The image is for representation purpose only.)

The equation of the path traversed by the ball is y = ax 2 + bx + c with respect


to a xy-coordinate system in the vertical plane. The ball traversed through the
points (10, 16), (20, 22) and (30, 25). The basketball hoop is at a horizontal
distance of 70 feet from Abdul. The height of the basketball hoop is 10 feet
from the floor to the top edge of the rim.

Did the ball successfully go through the hoop? Justify your answer.

(Hint: Consider the point where Abdul is standing as the origin of the
xy-coordinate system.)

34 Shown below are concrete elliptical water pipes, each 10 feet in length. 5

Page 176 of 199


The graph given above represents the inner circumference of the elliptical
pipe, where x and y are in feet. Assume that the water flows uniformly and
fully covers the inner cross-sectional area of the pipe.

Find the volume of water in the pipe at a given instant of time, in terms of π.
Use the integration method and show your steps.

(Note: Volume = Area of the base × Height)

35 i) Find the vector and cartesian equations of the straight line passing through 5
the point (-5, 7, -4) and in the direction of (3, -2, 1).

ii) Find the point where this straight line crosses the xy-plane.

Show your work.


OR

Given below are two lines L1 and L2 : 5

L1 : 2x = 3y = -z
L2 : 6x = -y = -4z

i) Find the angle between the two lines.


ii) Find the shortest distance between the two lines.

Show your work.

Page 177 of 199


SECTION E
(This section comprises of 3 case-study/passage-based questions of 4 marks
each with two sub-questions. First two case study questions have three sub
questions of marks 1, 1, 2 respectively. The third case study question has two
sub questions of 2 marks each.)

Serial
No. Question Marks

36 Answer the questions based on the given information.


The flight path of two airplanes in a flight simulator game are shown below.
The coordinates of the airports P and Q are given.

Airplane 1 flies directly from P to Q.


Airplane 2 has a layover at R and then flies to Q.

(Note: Assume that the flight path is straight and fuel is consumed uniformly
throughout the flight.)
1
i) Find the vector that represents the flight path of Airplane 1. Show your
steps.

ii) Write the vector representing the path of Airplane 2 from R to Q. Show 1
your steps.

Page 178 of 199


iii) What is the angle between the flight paths of Airplane 1 and Airplane 2 2
just after takeoff? Show your work.

OR

iii) Consider that Airplane 1 started the flight with a full fuel tank. 2

Find the position vector of the point where a third of the fuel runs out if the
entire fuel is required for the flight. Show your work.

37 Answer the questions based on the given information.

Rubiya, Thaksh, Shanteri, and Lilly entered a spinning zone for a fun game,
but there is a twist: they don't know which spinner will appear on their
screens until it is their turn to play. They may encounter one of the following
spinners, or perhaps even both:

Different combinations of numbers will lead to exciting prizes. Below are


some of the rewards they can win:

♦ Get the number '5', from Spinner A and '8' from Spinner B, and you'll win a
music player!
♦ You win a photo frame if Spinner A lands on a value greater than that of
Spinner B!

Page 179 of 199


i) Thaksh spun both the spinners, A and B in one of his turns. 1

What is the probability that Thaksh wins a music player in that turn? Show
your steps.

ii) Lilly spun spinner B in one of her turns. 1

What is the probability that the number she got is even given that it is a
multiple of 3? Show your steps.

iii) Rubiya spun both the spinners. 2

What is the probability that she wins a photo frame? Show your work.

OR

iii) As Shanteri steps up to the screen, the game administrator reveals that for 2
her turn, the probability of seeing Spinner A on the screen is 65%, while that
of Spinner B is 35%.

What is the probability that Shanteri gets the number '2'? Show your steps.

38 Answer the questions based on the given information.

Two metal rods, R1 and R2 , of lengths 16 m and 12 m respectively, are


insulated at both the ends. Rod R1 is being heated from a specific point while
rod R2 is being cooled from a specific point.

The temperature (T) in Celsius within both rods fluctuates based on the
distance (x) measured from either end. The temperature at a particular point
along the rod is determined by the equations T = (16 - x)x and T = (x -
12)x for rods R1 and R2 respectively, where the distance x is measured in
meters from one of the ends.

i) Find the rate of change of temperature at the mid point of the rod that is
being heated. Show your steps. 2

ii) Find the minimum temperature attained by the rod that is being cooled. 2
Show your work.

Page 180 of 199


CBSE
Additional Practice Questions-Marking Scheme
Subject: Mathematics (041)
Class: XII 2023-24

Time Allowed: 3 Hours Maximum Marks: 80

SECTION A
Multiple Choice Questions of 1 mark each.
Q
No. Answer/Solution Marks
1 (b) 1

2 (c) 1

3 3 1
(b) 2 sq units

4 (d) 1

5 (b) f(x) is continuous but not differentiable. 1

6 (a) (-∞, 0) 1

7 (a) 1

8 (a) 6log(2) – 2 1

9 (c) 1

Page 181 of 199


10 (a) only (i) 1

11 (d) only (ii) and (iii) 1

12 3 1
(a) λ = 5 , σ = 0

13 (b) (√2, 2, √2) 1


14 (d) p = 8, q = 4, r = (-3) 1

15 (a) exactly one 1

16 (d) exists as the inequality 3x + 2y < 6 does not have any point in common 1
with the feasible region.

17 (c) both maximum and minimum 1

18 (c) 1

19 (d) (A) is false but (R) is true. 1

20 (d) Both (A) and (R) are false. 1

SECTION B
Very short answer questions of 2 marks each.
Q
No. Answer/Solution Marks

Page 182 of 199


21 Since the domain of inverse of cosine function is [-1, 1], finds the domain of the given 1.5
function as follows:

-1 ≤ x - 1 ≤ 1
So, 0 ≤ x ≤ 2

And,

-1 ≤ 1 - x ≤ 1
=> 1 ≥ x - 1 ≥ -1
So, 2 ≥ x ≥ 0

Concludes the domain of cos-1 (|x - 1|) as [0, 2]. 0.5

OR

Draws the graphs of y = 2sin-1 (x) as shown below.

2.0

Page 183 of 199


22

0.5

Obtains x = 3 and w = 2 using the relationship obtained in the previous step as


follows:

2x = 6 => x = 3 0.5
2w = 4 => w = 2

Writes any value of y and z that satisfies the third relationship obtained in the first
step. For example, y = 0 and z = -1. 0.5

0.5

(Award full marks for any other matrix that satisfies the relationship.)
23 0.5

0.5

0.5

0.5

(Award full marks if any alternate method is used.)

Page 184 of 199


24 Writes that the rate at which the number of active users is increasing or decreasing at 0.5
𝑑
a given time is given by 𝑑𝑡 N(t).

Finds the derivative of N(t) as:


𝑑 1.0
N(t) = 1000 (0.1) e0.1t
𝑑𝑡

Finds the rate of change of active users at 10 minutes past 5 pm as:


𝑑
N(10) = 1000(0.1)e(0.1)(10) = 100e
𝑑𝑡

Concludes that the number of active users are increasing at a rate of 100e people per
0.5
minute at 5:10 pm on that day.

OR

Finds the derivative of the given function as:

1.0

Writes that the above quantity is greater than 0 for any value of t. 0.5

Concludes that the rabbit population is increasing. 0.5


25 Substitutes (k - x) by u to get dx = -du and rewrites the given integral as:

0.5

Integrates the expression in above step as:

1.0
where, C is the constant of integration.

Substitutes u = (k - x) back in the above expression and writes:

0.5

where, C is the constant of integration.

Page 185 of 199


SECTION C
Short answer questions of 3 marks each.
Q
No. Answer/Solution Marks
26 Rewrites the numerator of the given integral as:

𝑑
3x + 5 = A 𝑑𝑥 (x 2 + 4x + 7) + B
0.5
=> 3x + 5 = A(2x + 4) + B

Finds the values of A and B by comparing the coefficients of like terms as:
3
2A = 3 => A = 2
1.0
4A + B = 5 => B = -1

Substitutes the values of A and B in the given integral and integrates the same as:

1.5

where, C is the constant of integration.

27 Takes u = (25 + cos θ).

Finds du as:
0.5
du = -sin θ dθ

𝜋
Finds the change in limit when θ = 0 and θ = 2 to u = 26 and u = 25 respectively. 0.5

Rewrites the given integral using the above substitution and integrates the same as:

Page 186 of 199


1.5

0.5

Applies the limit to find the value of the given definite integral as
26 ×26
log .
25 ×27

(Award full marks if the problem is solved correctly by taking u = 26 + cos θ.)
1.0
OR

States the property that is going to be used as:

Takes 2a = π and f(x) = h(sin x).

Finds f(2a - x) as:

f(2a - x) = f(π - x) = h(sin(π - x)) = h(sin x) = f(x) 2.0

Thus confirms that the property listed in the above step can be applied to the given
integral.

Hence concludes that:

28 (i) Models the situation and rearranges terms to form a linear differential equation as
follows:
0.5

Page 187 of 199


𝑑𝑦
(ii) Considering the obtained equation as linear of the form + Py = Q with P = 8 and
𝑑𝑥
hence takes the integrating factor as:
0.5

Multiplies the differential equation by the integrating factor as follows:

0.5

Integrates both sides to obtain the general solution of the differential equation as
follows:

1.0

Uses the initial condition v(0) = 10 m/s to find the value of C as follows:

Hence, writes the expression for the velocity of the ball as a function of time as
follows:

0.5

29 Frames the differential equation using the given conditions as follows:

1.0

Rearranges the terms to separate the variables as follows:


0.5

Integrates both sides to obtain the general solution of the curve. The working may look
as follows:

Page 188 of 199


1.0

Substitutes (8, 2) to obtain the equation of the curve as follows:


0.5

OR

(i) Separates the variables and rearranges the terms of the differential equation as
follows:
0.5

Integrates both sides to obtain the following:


1.0

0.5
Writes that the solution is a general solution of a circle and hence it represents a family
of circles.

(ii) Substitutes x = 0 and y = 0 into the general solution to obtain C 2 = 0 and writes the
particular solution as:
0.5
x2 + y2 - 4x + 2y = 0

Rearranges the terms to rewrite the particular solution as (x - 2)2 + (y + 1)2 = 5 to find
the radius as √5 units. The working may look as follows:

x 2 + y2 - 4x + 2y = 0
Adding 5 to both sides and rearranging terms,
0.5
=> (x 2 - 4x + 4) + (y2 + 2y + 1) = 5
=> (x - 2)2 + (y + 1)2 = 5

Page 189 of 199


30 Finds the profit on selling the products as:

Profit for each unit of Product A sold = 100 - 60 = Rs 40


0.5
Profit for each unit of Product B sold = 150 - 90 = Rs 60.

Takes x and y to be the numbers of Product A and Product B to be produced in a day


respectively and frames the objective function as:

Maximise Z = 40x + 60y 1.0

Writes the constraints of the given linear programming problem as:

60x + 90y ≤ 8000


x + y ≤ 100 1.5
y ≤ 2x or -2x + y ≤ 0
x, y ≥ 0

OR

i) Uses the graph of the feasible region and lists the constraints of the given
maximisation problem as:

3x + 2y ≤ 12
x + 2y ≤ 8 1.5
x, y ≥ 0

ii) Finds the value of the objective function at corner points as:

Corner point z = 5x + 3y
(0, 0) 0
1.0
(0, 4) 12
(2, 3) 19
(4, 0) 20

Concludes that the objective function attains maximum value at (4, 0) and hence (4, 0) 0.5
is the optimal solution.

31 Applies Bayes' theorem and writes:

Page 190 of 199


1.0

Substitutes respective probabilities in the expression obtained above to find the


required probability as follows:

1.5

Simplifies the above expression to get the probability that a randomly selected task that
10 0.5
is not important is urgent as 19 or 52.63%.

SECTION D
Long answer questions of 5 marks each.
Q
No. Answer/Solution Marks

Page 191 of 199


32 i) Writes that for no x ∈ U, (x, x) ∈ R as the difference in time between x & x is
0 hours.

Concludes that R is not reflexive. 1.5

Writes that, whenever the difference in time between x and y is 6 hours, the
difference in time between y and x is also 6 hours. That is, (x, y) ∈ R =>
(y, x) ∈ R.
1.5
Concludes that R is symmetric.

Writes that, if the difference in time between x and y is 6 hours, and the
difference in time between y and z is also 6 hours, then the difference in time
between x and z could be either 0 hours or 12 hours. That is, (x, y) ∈ R &
(y, z) ∈ R but (x, z) ∉ R.

Concludes that R is not transitive. 1.5

ii) From the above steps, concludes that R is not an equivalence relation. 0.5

OR

i) Assumes f(x) = f(y) and evaluates the same as:

1.5

Uses the above step to conclude that x = y or xy = -1. 0.5

Takes a pair of numbers x and y such that xy = -1 to show that f is not one-one.

For example,
1 2 2
for x = and y = -2, f(x) = − and f(y) = − .
2 3 3
1.0

Page 192 of 199


ii) Equates f(x) to y and solves the same to express x in terms of y as:

1.5

Writes that for any y ∈ R (codomain), there exists x ∈ R - {-1, 1} (domain) 0.5
such that f(x) = y. Hence concludes that f is onto.

33 Writes the system of equations as:

100a + 10b + c = 16
400a + 20b + c = 22 0.5
900a + 30b + c = 25

Writes the above system of equations in the form AX = B as:

0.5

Finds |A| as 1(18000 - 12000) - 1(3000 - 9000) + 1(2000 - 4000) = -2000 and 0.5
writes that A-1 exists as |A| ≠ 0.

Finds A-1 as:

1.5

(Award 1 mark if only all the cofactors are found correctly.)

Page 193 of 199


3 21
Finds the values of a, b and c as − 200 , 20 and 7 respectively by solving
X = A-1 B as:

1.0

Finds the equation of the path traversed by the ball as: 0.5
3 21
y = − 200 x 2 + 20 x + 7.

Writes that when x = 70 feet, y = 7 feet. So, the ball went by 7 feet above the
floor that means 3 feet below the basketball hoop. So, the ball did not go 0.5
through the hoop.

34 Finds the equation of the ellipse as:

0.5

Expresses y in terms of x as:


0.5

Integrates the above equation with respect to x from limit 0 to 6, that gives the
area of one quarter of the ellipse. The working may look as follows:
0.5

Applies the formula of integration and simplifies as:


1.0

Applies the limit and solves further as:


0.5

Simplifies the above expression to get the area of one-quarter of the base as 6π 1.0
sq feet.
Finds the area of the whole ellipse as 4 × 6π = 24π sq feet. 0.5
Finds the volume of water as 24π × 10 = 240π cubic feet. 0.5

Page 194 of 199


35 0.5

Writes the vector equation of the given straight line as:


1.0

Writes the cartesian equation of the given straight line as:


1.0

ii) Simplifies the vector form obtained in step 2 as:


1.0

Writes that at the point where the line crosses xy-plane, its z-coordinate is zero
and equates the z-coordinate of the above equation to zero as:

λ-4=0 0.5
=> λ = 4

Substitutes λ = 4 in the vector form to get the required point as (7, -1, 0). 1.0

OR

Rewrites the equation of L1 in cartesian form as:


𝑥 𝑦 𝑧
= 2 = −6 0.5
3

Rewrites the equation of L2 in cartesian form as:


0.5
𝑥 𝑦 𝑧
= −12 = −3
2

i) Identifies the direction cosines of both the lines as (3, 2, -6) and 0.5
(2, -12, -3).

Finds the cosine of the angle between the two lines as:
1.5

(Award 0.5 marks if only the formula of the cosine of the angle between the
two lines is written correctly.)
Concludes that the angle between the two lines is 90°. 0.5

Page 195 of 199


ii) Rewrites the equations of L1 and L2 in vector form as:

1.0

Writes that both the lines pass through the origin hence intersect at the origin.

(Award full marks if the inference about both lines passing through the origin
is drawn without writing the vector forms.)

Writes that since both the lines intersect at the origin, the shortest distance 0.5
between the two lines is 0 units.

SECTION E
Case-based questions of 4 marks each.
Q
No. Answer/Solution Marks
36 Writes the vectors for points P and Q as follows:
i)

0.5

Finds the vector representing the flight path of Airplane 1 as:

0.5

36 Uses vector subtraction to find the vector representing the flight path from R to
ii) Q as:

1.0

Page 196 of 199


36 Finds the cosine of the angle between the vectors representing the flight paths
iii) of Airplane 1 and Airplane 2 as:

1.5

Finds the angle between the flight paths as:

0.5

OR

Considers a point S which divides PQ internally in the ratio 1:2. 0.5

Finds the position vector of point S as:

1.5

(Award 0.5 marks if only the formula is written correctly.)

37 Finds the required probability as:


i)

1.0

37 Uses the conditional probability and finds the required probability as follows:
ii)

1.0

Page 197 of 199


37 Finds the probability of getting 2 from spinner A and getting 1 from spinner B
iii) as:
0.5

Finds the probability of getting 5 from spinner A and getting either 1, 2, 3 or 4


from spinner B as:

1.0

Writes that P1 and P2 are mutually exclusive and hence, finds the probability
that she wins a photo frame as:

0.5

OR

Uses the theorem of total probability and writes:

P(getting 2) = [P(Spinner A) × P(Getting 2|Spinner A)] +


[P(Spinner B) × P(Getting 2|Spinner B)] 0.5

Finds the required probability by substituting the required probability as:

1.5

38 Identifies that the rod being heated is R1 and finds the rate of change of
i) temperature at any distance from one end of R1 as:
1.0

0.5
Finds the mid-point of the rod as x = 8 m.

Page 198 of 199


Finds the rate of change of temperature at the mid point of R1 as:

0.5

38 Identifies that the rod being cooled is R2 and finds the rate of change of
ii) temperature at any distance x m as:

𝑑𝑇
Equates to 0 to get the critical point as x = 6. 1.0
𝑑𝑥

Finds the second derivative of T as:

0.5

Finds the minimum temperature attained by the rod R2 as


0.5
T(6) = (6 - 12)6 = -36 °C.

Page 199 of 199

You might also like